You are on page 1of 188

Tax accounting chapter exams Chapter 16:

1 Marks: 1/1 Six possibilities exist for the result after all possible capital gain and loss netting has been completed. Choose one answer. a. True Correct. b. False p. 16-20 Correct Marks for this submission: 1/1. Question 2 Marks: 1/1 There are three holding periods: short-term, mid-term, and long-term. Choose one answer. a. True Correct. b. False There are two holding periods: short-term (one year or less) and long-term (more than one year). p. 16-3 Correct Marks for this submission: 1/1. Question 3 Marks: 1/1 A franchise transfer is generally a sale or exchange of a capital asset. Choose one answer. a. True Correct. b. False Section 1253 provides that a transfer of a franchise, trademark, or trade name is not a transfer of a capital asset when the transferor retains any significant power, right, or continuing interest in the property transferred. p. 16-13 Correct Marks for this submission: 1/1. Question 4 Marks: 1/1 The 0% gain portion of the 0%/15% gain applies when the taxable income before taxing the 0%/15% gain puts the individual taxpayer out of the 15% regular income tax bracket. Choose one answer. a. True b. False Correct.

The 0% gain portion of the 0%/15% gain applies when the taxable income before taxing the 0%/15% gain does not put the individual taxpayer out of the 15% regular income tax bracket. p. 16-21 Correct Marks for this submission: 1/1. Question 5 Marks: 1/1 Tom has owned 20 shares of Burgundy Corporation stock for four years. He sells the stock short for a total of $800. One month later, he closes the short sale by purchasing and delivering 20 shares of Burgundy Corporation stock for a total of $600. Tom has a $200 short-term capital gain. Choose one answer. a. True Correct. b. False pp. 16-18 and 16-19 Correct Marks for this submission: 1/1. Question 6 Marks: 1/1 The tax law does not require capital gains and losses to be separated from other types of gains and losses. Choose one answer. a. True Correct. b. False The tax law requires capital gains and losses to be separated from other types of gains and losses because long-term capital gains may receive beneficial tax treatment as compared to ordinary gains and because restrictions exist with respect to the deductibility of net capital losses. p. 16-2 Correct Marks for this submission: 1/1. Question 7 Marks: 1/1 To compute the holding period, start counting on the day the property was acquired and include the day of disposition. Choose one answer. a. True Correct. b. False A disallowed loss from a sale to a related taxpayer is never usable by the seller. There is also no tacking of the holding period of the property sold to any later acquired property. Uptons basis for the purchased property would equal what he paid for it$45,000. p. 16-17 Correct Marks for this submission: 1/1. Question 8 Marks: 1/1

An alternative tax calculation may be used when taxable income includes net long-term capital gain. Therefore, capital gains and losses must be matched with one another to see if a net longterm capital gain exists. Choose one answer. a. True Correct. b. False pp. 16-2 and 16-3 Correct Marks for this submission: 1/1. Question 9 Marks: 1/1 Personal use assets are capital assets. Choose one answer. a. True Correct. b. False p. 16-4 Correct Marks for this submission: 1/1. Question 10 Marks: 1/1 Lease cancellation payments received by a lessee are treated as an exchange and can result in a capital gain. Choose one answer. a. True Correct. b. False A capital gain will result if the lease is a capital asset to the lessee (i.e., personal use property). p. 16-15 Correct Marks for this submission: 1/1. Question 11 Marks: 1/1 Which of the following events causes the purchaser of an option to add the cost of the option to the basis of the property to which the option relates? Choose one answer. a. The option is exercised. b. The option is sold. c. The option lapses. d. The option is rescinded. e. None of the above. Correct.

If the option is exercised, the cost of the option becomes part of the basis of the property. Otherwise, the lapse of the option is treated as a sale or exchange. p. 16-11 Correct Marks for this submission: 1/1. Question 12 Marks: 1/1 A like-kind exchange of a 1231 asset has occurred. Choose one answer. a. The holding period of property received in the exchange includes the holding Correct. period of the property given up. b. The holding period of property received in the exchange does not include the holding period of the property given up. c. The holding period of property received in the exchange includes six months of the holding period of the property given up. d. The holding period of property received in the exchange includes twelve months of the holding period of the property given up. e. None of the above. If a 1231 asset or a capital asset is the subject of a like-kind exchange, the holding period of the property received includes the holding period of the property given up. p. 16-16 Correct Marks for this submission: 1/1. Question 13 Marks: 1/1 Rea is a songwriter. She wrote a song, copyrighted it, and sold it for $10,000 cash. The song had a zero tax basis. The purchaser was a national song brokerage company. Rea is in the business of songwriting. Unless Rea makes a special election, the $10,000 received by Rea is: Choose one answer. a. Long-term capital gain. b. Short-term capital gain. c. Ordinary gain. d. Excludible from gross income. e. None of the above. The person whose efforts led to the copyright or creative work has an ordinary asset, not a capital asset. If that work is a musical composition, the taxpayer may elect to treat the work as a capital asset. p. 16-4 and 16-6 Correct Marks for this submission: 1/1. Question 14 Marks: 1/1 Tan, Inc., has a 2009 $50,000 long-term capital gain included in its $185,000 taxable income. Which of the following is correct? Correct.

Choose one answer. a. Tan will benefit from an alternative tax on net capital gains computation. b. Tan's regular tax on taxable income will be the same as its tax using an alternative tax on net capital gains approach. c. Tan's $50,000 net capital gain is not taxable. d. Tan's regular tax on taxable income will be greater than its tax using an alternative tax on net capital gain approach. e. None of the above. Although there is an alternative tax on net capital gains for corporations, it yields the same tax result as the regular computation method. This unusual result is caused by the fact that the alternative tax rate and the maximum regular tax rate are both 35%. p. 16-32 Correct Marks for this submission: 1/1. Question 15 Marks: 0/1 Individuals with capital gains and/or losses use Schedule D. Using the 2010 Schedule D as a reference, which of the following is correct? Choose one answer. a. The form is not used unless there is at least $1,000 of capital gain or loss. b. Part I reports long-term gains and losses. c. Part III reports qualified dividend income. d. Part III requires a netting of net short-term capital loss against net long-term capital gain. Correct e. None of the above. There is no minimum amount of capital gain or loss required to use Schedule D (a.). Part I reports short-term gains and losses (b.). Part III requires netting of net short-term and net longterm amounts when one is a loss and the other is gain (d.). Part III is not used to report qualified dividend income (c.). Installment sales are reported in Part I or Part II depending upon whether there is a recognized short-term or long-term capital gain. pp. 16-28 to 16-32 Incorrect Marks for this submission: 0/1. Question 16 Marks: 1/1 Sylvia purchased for $680 a $2,000 bond when it was issued two years ago. Sylvia amortized $200 of the original issue discount and then sold the bond for $1,800. Which of the following statements is correct? Choose one answer. a. Sylvia has $920 of long-term capital gain. b. Sylvia has $1,220 of long-term capital gain. c. Sylvia has no capital gain or loss. Correct. . Correct.

d. Sylvia has $200 of long-term capital loss. e. None of the above. Sylvias original basis of $680 is increased by the $200 of original issue discount amortization. Her basis is $880 when she sells the bond for $1,800; so her gain is $920. p. 16-9 Correct Marks for this submission: 1/1. Question 17 Marks: 1/1 Walnut Company signs a 15-year franchise agreement with Cookie Company. Cookie Company retained significant powers, rights, and a continuing interest. Walnut (the franchisee) makes noncontingent payments of $25,000 per year for the first five years of the franchise. Walnut Company also pays a contingent fee of 2% of gross sales every month. Which of the following statements is correct? Choose one answer. a. Walnut Company may deduct only the $25,000 per year noncontingent payments in full as they are made. b. Walnut Company may deduct only the monthly contingent fee as it is paid. c. Walnut Company may deduct both the noncontingent annual fee and the contingent monthly fees as they are paid. d. Walnut Company may not deduct either the noncontingent annual fee or the contingent monthly fees as they are paid. e. None of the other answers are correct. The contingent payments are deductible as they are made. The noncontingent payments must be capitalized and amortized over 15 years. pp. 16-13 and 16-14 Correct Marks for this submission: 1/1. Question 18 Marks: 1/1 In 2011, Satesh has $4,000 short-term capital loss, $14,000 0%/15% long-term capital gain, and $7,000 qualified dividend income. Satesh is single and has other taxable income of $15,000. Which of the following statements is correct? Choose one answer. a. No more than $14,000 of Satesh's taxable income is taxed at 0%. b. No more than $7,000 of Satesh's taxable income is taxed at 0%. c. No more than $17,000 of Satesh's taxable income is taxed at 0%. d. None of Satesh's taxable income is taxed at 0%. e. All of Satesh's taxable income is taxed at 0%. Correct. Correct.

The net long-term capital gain is $10,000 ($14,000 0%/15% long-term capital gain $4,000 short-term capital loss). The $7,000 qualified dividend income is added to the 0%/15% net long-term capital gain and the $17,000 total is eligible for the 0%/15% alternative tax rate. REF: p. 16-23 | p. 16-24 | Example 37 Correct Marks for this submission: 1/1. Question 19 Marks: 1/1 In 2010, Jenny had a $14,000 net short-term capital loss and deducted $3,000 as a capital loss deduction. In 2011, Jenny has a $18,000 0%/15% long-term capital gain and no other capital gain or loss transactions. Which of the statements below is correct? Choose one answer. a. Jenny has a 2011 $18,000 net capital gain. b. Jenny has a 2011 $7,000 net capital gain. c. Jenny has a 2011 $7,000 net capital loss. d. Jenny has a 2011 $3,000 capital loss deduction. e. Jenny has a 2011 $7,000 capital loss deduction. The 2010 capital loss carryforward is $11,000 ($14,000 2010 net capital loss 2010 $3,000 capital loss deduction). The $11,000 carries forward as a short-term capital loss and is offset against the $18,000 long-term capital gain. p. 16-23 Correct Marks for this submission: 1/1. History of Responses: # Action Response Time Raw score Grade 1 1 1 1 Jenny has a 2011 $7,000 net capital 18:29:37 on gain. 22/04/12 Jenny has a 2011 $7,000 net capital gain. 18:29:37 on 22/04/12 Correct.

1 Grade 2 Close&Grade Question 20 Marks: 1/1

Sophia purchased for $8,700 a $10,000 bond when it was issued two years ago. Sophia amortized $300 of the original issue discount and then sold the bond for $9,500. Which of the following statements is correct? Choose one answer. a. Sophia has $1,100 of long-term capital gain. b. Sophia has $800 of long-term capital gain. c. Sophia has $500 of long-term capital gain. Correct.

d. Sophia has $800 long-term capital loss. e. None of the other answers are correct. Correct Marks for this submission: 1/1. Question 21 Marks: 1/1 Sam is filing as head of household and has 2011 taxable income of $46,750 which includes $23,000 of 0%/15% gain. What is the tax on his taxable income using the alternative tax method? Choose one answer. a. $6,455. b. $2,250. c. $2,865. d. $7,013. e. None of the other answers are correct. Sams taxable income takes him out of the 15% regular tax bracket, but his regular taxable income does not. Therefore, a portion of the 0%/15% gain is taxed at 0% and a portion is taxed at 15%. His total tax is $4,126 $1,215 tax on first $12,500 (Schedule Z) $1,575 tax on income between $12,500 and $23,000 at 15%. $ $ 0 tax on capital gain from $23,000 to $46,250 at 0%. 75 tax on capital gain from $46,250 to $46,750 at 15%. Correct.

$2,865 total alternative tax on taxable income p. 16-25 Correct Marks for this submission: 1/1. Question 22 Marks: 1/1 Which of the following comparisons is correct? Choose one answer. a. Corporations may carryback capital losses; individuals may not. b. Both corporation and individual long-term capital losses carryover as short-term capital losses. Correct.

c. Corporations may carryforward capital losses indefinitely; individuals may only carryforward capital losses for five years. d. Both corporations and individuals may use an alternative tax rate on net capital gains. e. None of the above. Corporations may carryback capital losses; individuals may not (a). Individual long-term capital losses carryover as long-term, not short-term. Corporations have a five-year carryforward limit; individuals may carryforward capital losses indefinitely. Corporations' alternative tax rate on long-term capital gains is 35%, not 20%. REF: p. 16-27 | p. 16-32 Correct Marks for this submission: 1/1. Question 23 Marks: 1/1 The tax law requires that capital gains and losses be separated from other types of gains and losses. Among the reasons for this treatment are: Choose one answer. a. Long-term capital gains may be taxed at a lower rate than ordinary gains. b. Capital losses that are short term are not deductible. c. Net capital losses are deductible only up to $3,000 per year for individual taxpayers. d. a. and c. Correct.

Both a net long-term capital gain and a net capital loss are subject to special rules. Consequently, the capital gains and losses must be netted separately from other types of gains and losses. REF: p. 16-3 Correct Marks for this submission: 1/1. Question 24 Marks: 1/1 Lewis is a cash basis landlord. A tornado destroyed one of his residential rental buildings. Even though the lease required the tenants (all cash basis) to continue to pay rent, Lewis accepts a payment of $1,000 in full cancellation of the lease. As a result of these events: Choose one answer. a. Lewis has $1,000 of capital gain. b. Lewis's tenants have $1,000 of ordinary income. c. Lewis has a $1,000 capital loss.

d. Lewis has $1,000 of ordinary income. e. None of the other answers are correct.

Correct.

Lease cancellation payments are generally capital gain to the lessee recipient if the lease is a capital asset to the lessee. The payments are a deduction for the paying lessor. However, if the landlord is the recipient, the payments are ordinary income. p. 16-15 Correct Marks for this submission: 1/1. History of Responses: # Action Response Lewis has $1,000 of ordinary income. Lewis has $1,000 of ordinary income. Time 18:48:17 on 22/04/12 18:48:17 on 22/04/12 Raw score Grade 1 1 1 1

2 Grade 3 Close&Grade

Question 25 Marks: 1/1 Julia purchased vacant land in 2009 that she subdivided for resale as lots. All 10 of the lots were sold during 2010. The lots had a tax basis of $3,000 each and sold for $45,000 each. Julia made no substantial improvements to the lots. She acted as her own real estate broker; so there were no sales expenses for selling the lots. Which of the following statements is correct? Choose one answer. a. Julia must hold the lots for at least five years before she is eligible for the special Correct. capital gain treatment of 1237. b. Some of the gain from the sale of the ten lots is long-term capital gain. c. All of the gain from the sale of the ten lots is long-term capital gain. d. To be eligible for the special capital gain treatment of 1237, Julia must be a real estate dealer. e. None of the above. Julia must hold the land at least five years to be eligible for 1237 treatment and must not be a dealer in lots. Since Julia does not satisfy this requirement, all of the gain is ordinary income. pp. 16-7 and 16-8 Correct Marks for this submission: 1/1.

Chapter 8:

1 Marks: 1/1 If an automobile is placed in service in 2009, the limitation for cost recovery in 2011 will be based on the cost recovery limits for the year 2011. Choose one answer. a. True Correct. b. False The limits will be based on the limits for automobiles placed in service in 2009, the year placed in service. REF: p. 8-16 Correct Marks for this submission: 1/1. History of Responses: # 1 Grade Action Response False Time 08:01:32 on 15/03/12 1 Raw score 1 Grade

2 Close&Grade False 08:01:32 on 15/03/12 1 1 Question 2 Marks: 1/1 Under the alternative depreciation system (ADS), only the half-year convention is applicable for personalty. Choose one answer. a. True Correct. b. False The mid-quarter convention also is applicable for personalty. p. 8-20 Correct Marks for this submission: 1/1. History of Responses: # 1 Grade Action Response False False Time 08:03:37 on 15/03/12 08:03:37 on 15/03/12 1 1 Raw score 1 1 Grade

2 Close&Grade Question 3 Marks: 1/1

The statutory dollar cost recovery limits under 280F on passenger automobiles are imposed after any reduction for personal use is considered. Choose one answer.

a. True Correct. b. False These limits are imposed before any reduction for personal use. p. 8-15-16 Correct Marks for this submission: 1/1. History of Responses: # 1 Grade Action Response False False Time 08:08:26 on 15/03/12 08:08:26 on 15/03/12 1 1 Raw score 1 1 Grade

2 Close&Grade Question 4 Marks: 1/1 Choose one answer. a. True Correct.

Under the alternative depreciation system (ADS), the salvage value of the asset is ignored.

b. False p. 8-20 Correct Marks for this submission: 1/1. History of Responses: # 2 Grade Action Response True True Time 15:07:46 on 15/03/12 15:07:46 on 15/03/12 1 1 Raw score 1 1 Grade

3 Close&Grade Question 5 Marks: 1/1 Choose one answer. a. True

Personal use property that is subject to wear and tear is eligible for cost recovery.

Correct. b. False Personal use property is not eligible for cost recovery. p. 8-3 Correct Marks for this submission: 1/1. History of Responses: # 1 Grade Action Response False False Time 08:14:34 on 15/03/12 08:14:34 on 15/03/12 1 1 Raw score 1 1 Grade

2 Close&Grade Question 6 Marks: 1/1

Taxpayers may elect to use the straight-line method under MACRS for personal property. Choose one answer. a. True Correct. b. False p. 8-11 Correct Marks for this submission: 1/1. History of Responses: # 1 Grade Action Response True True Time 15:09:48 on 15/03/12 15:09:48 on 15/03/12 1 1 Raw score 1 1 Grade

2 Close&Grade Question 7 Marks: 1/1

Under the MACRS straight-line election for personalty, the mid-quarter convention is not applicable. Choose one answer. a. True Correct. b. False The mid-quarter convention is applicable under the MACRS straight-line election for personalty. p. 8-11 Correct Marks for this submission: 1/1. History of Responses: # 1 Grade Action Response False False Time 08:19:01 on 15/03/12 08:19:01 on 15/03/12 1 1 Raw score 1 1 Grade

2 Close&Grade Question 8 Marks: 1/1 Choose one answer. a. True

The key date for calculating cost recovery is the date the asset is purchased.

Correct. b. False The key date for calculating cost recovery is the date the asset is placed in service. p. 83 Correct Marks for this submission: 1/1.

History of Responses: # 1 Grade Action Response False False Time 08:19:50 on 15/03/12 08:19:50 on 15/03/12 1 1 Raw score 1 1 Grade

2 Close&Grade Question 9 Marks: 1/1

If a new car that is used predominantly in business is placed in service in 2011, the statutory dollar cost recovery limit under 280F will depend on whether the taxpayer takes MARCS or straight-line depreciation. Choose one answer. a. True Correct. b. False The statutory cost recovery limits apply whether MACRS or straight-line apply. pp. 8-16 Correct Marks for this submission: 1/1. History of Responses: # 1 Grade Action Response False False Time 08:21:26 on 15/03/12 08:21:26 on 15/03/12 1 1 Raw score 1 1 Grade

2 Close&Grade Question 10 Marks: 1/1

The 179 deduction can exceed $500,000 in 2011 if the taxpayer had a 179 amount which exceeded the taxable income limitation in the prior year. Choose one answer. a. True b. False Correct.

The 179 amount eligible for expensing in a carryforward year is limited to the lesser of (1) the statutory dollar amount ($500,000 in 2011) reduced by the cost of 179 property placed in service in excess of $2 million in the carryforward year or (2) the business income limitation in the carryforward year. REF: p. 8-13 Correct Marks for this submission: 1/1.

History of Responses: # 1 Grade Action Response False Time 08:22:40 on 15/03/12 1 1 Raw score 1 1 Grade

2 Close&Grade False 08:22:40 on 15/03/12 Question 11 Marks: 1/1 Patents are amortized over what period of time? Choose one answer. a. 60 months or more. b. 15 years. c. 20 years. d. 30 years. e. None of the above. Correct.

Patents are 197 intangibles and thus are amortized over a 15-year period. Correct Marks for this submission: 1/1. History of Responses: # 2 3 Grade Close&Grade Action Response 15 years. 15 years. Time 20:19:32 on 16/03/12 20:19:32 on 16/03/12 1 1 Raw score 1 1 Grade

Question 12 Marks: 0/1 Jonathan purchased a business asset (three-year property) on November 15, 2010, at a cost of $30,000. This is the only asset he purchased during the year. Jonathan did not elect to expense any of the asset under 179, nor did he elect straight-line cost recovery. Jonathan sold the asset on May 13, 2011. Determine the cost recovery deduction for 2011. Choose one answer. a. $3,750. b. $6,875. c. $9,999. d. $18,333. e. None of the other answers are correct. The mid-quarter convention applies in this case. $30,000 X .6111 X 1.5/4 = $6,875. pp. 8-7, 8-8, and Table 8-2 Incorrect.

Incorrect Marks for this submission: 0/1. Question 13 Marks: 1/1 James purchased a new business asset (three-year property) on July 23, 2011, at a cost of $50,000. He did not elect to expense any of the asset under 179, nor did he elect straight-line cost recovery. Determine the cost recovery deduction for 2011. Choose one answer. a. $8,333. b. $33.333 c. $16,665 d. $41,665. e. None of the other answers are correct. Additional first-year depreciation is $25,000 ($50,000 50%). Regular MACRS is $8,333 ($25,000 .3333). So total cost recovery is $33,333 ($25,000 + $8,333). Correct Marks for this submission: 1/1. History of Responses: # 1 Grade Action Response $33.333 Time 10:52:53 on 15/03/12 1 1 Raw score 1 1 Grade Correct.

2 Close&Grade $33.333 10:52:53 on 15/03/12 Question 14 Marks: 1/1 Which of the following statements is true regarding depletion? Choose one answer. a. Cost depletion must be used for tax purposes. b. Percentage depletion must be used for tax purposes

c. A taxpayer will select the greater of cost or percentage depletion for tax purposes. d. A taxpayer will select the lower of cost or percentage depletion for tax purposes.

Correct.

e. None of the above. The taxpayer will select the greater of cost or percentage depletion for tax purposes. pp. 8-25 Correct

Marks for this submission: 1/1. History of Responses: # Action Response Time Raw score Grade 1 1 A taxpayer will select the greater of cost 10:59:22 on or percentage depletion for tax purposes. 15/03/12 10:59:22 on 15/03/12

1 Grade

A taxpayer will select the greater of 2 Close&Grade cost or percentage depletion for tax purposes. Question 15 Marks: 1/1

Pat purchased a used five-year class asset on March 15, 2011, for $60,000. He did not elect 179 expensing. Determine the cost recovery deduction for 2011 for earnings and profits purposes.

Choose one answer. a. $2,000. b. $3,000. c. $6,000. d. $12,000. e. None of the other answers are correct. .10 X $60,000 = $6,000. p. 8-20 and Table 8-5 Correct Marks for this submission: 1/1. History of Responses: # 1 Grade Action Response $6,000. $6,000. Time 11:01:51 on 15/03/12 11:01:51 on 15/03/12 1 1 Raw score 1 1 Grade Correct.

2 Close&Grade Question 16 Marks: 1/1

Lorraine purchased an apartment building on October 16, 2011, for $7,000,000. Determine the cost recovery deduction for 2011. Choose one answer. a. $36,470. b. $37,450.

c. $46,270. d. $53,060. e. None of the other answers are correct. .00758 X $7,000,000 = $53,060. pp. 8-10, and Table 8-6 Correct Marks for this submission: 1/1. History of Responses: # 2 Grade Action Response $53,060. Time 19:46:38 on 22/04/12 1 Raw score 1 Grade Correct.

3 Close&Grade $53,060. 19:46:38 on 22/04/12 1 1 Question 17 Marks: 1/1 For tax purposes, a deduction is allowed for the consumption of the cost of timber through: Choose one answer. a. Cost recovery. b. Amortization. c. Depletion. d. All of the other answers are acceptable. e. None of the other answers are correct. Timber is subject to depletion. p. 8-3 Correct Marks for this submission: 1/1. History of Responses: # 1 Grade Action Response Depletion. Depletion. Time 11:12:19 on 15/03/12 11:12:19 on 15/03/12 1 1 Raw score Grade 1 1 Correct.

2 Close&Grade Question 18 Marks: 1/1

The only asset Bill purchased during 2011 was a new seven-year class asset. The asset, which was listed property, was acquired on June 17 at a cost of $50,000. The asset was used 40% for business, 30% for the production of income, and the rest of the time for personal use. Bill always elects to expense the maximum amount under 179 whenever it is applicable. The net income from the business before the 179 deduction is $100,000. Determine Bills maximum deduction with respect to the property for 2011.

Choose one answer. a. $1,428. b. $2,499. c. $26,749. d. $33,375. e. None of the other answers are correct. The listed property does not pass the predominantly business usage test. Therefore, neither 179 expensing nor additional first-year depreciation can be taken. In addition, only straight-line cost recovery can be used. Maximum deduction ($50,000 X .0714 X 70%) $2,499 REF: p. 8-13 to 8-15 | Table 8-3 Correct Marks for this submission: 1/1. History of Responses: # 1 2 Grade Close&Grade Action Response $2,499. $2,499. Time 13:45:39 on 15/03/12 13:45:39 on 15/03/12 1 1 Raw score 1 1 Grade Correct.

Question 19 Marks: 1/1 On June 1 of the current year, Roman converted a building to rental property. At the time of the conversion, the building was worth $140,000. Five years ago Roman purchased the building for $120,000. The building is still encumbered by a $50,000 mortgage. What is the basis of the building for cost recovery? Choose one answer. a. $70,000. b. $90,000. c. $120,000. d. $140,000. e. None of the above. The basis is $120,000, the lower of the adjusted basis ($120,000) or fair market value ($140,000) at the date of conversion. p. 8-4 and Example 3 Correct Marks for this submission: 1/1. History of Responses: # Action Response Time Raw score Grade Correct.

1 2

Grade Close&Grade

$120,000. $120,000.

13:55:15 on 15/03/12 13:55:15 on 15/03/12

1 1

1 1

Question 20 Marks: 1/1 Birch Company acquires special tools (three-year property) on February 15, 2011, at a cost of $30,000. Birch also acquires a machine (five-year property) on July 15, 2011, at a cost of $40,000. No election is made to use the straight-line method. The company does not make the 179 election or elect bonus depreciation. Determine the total deductions in calculating taxable income related to the machines for 2011. Choose one answer. a. $17,999. b. $19,400 c. $44,000. d. $44,750. e. None of the other answers are correct. 3-year property ($30,000 X .3333)= $9,999 + 5-year property ($40,000 X .20)= $8,000 Total for 2006 $17,999 pp. 8-4 to 8-8 and Table 8-1 Correct Marks for this submission: 1/1. History of Responses: # 2 Grade Action Response $17,999. $17,999. Time 19:46:25 on 22/04/12 19:46:25 on 22/04/12 1 1 Raw score 1 1 Grade Correct.

3 Close&Grade Question 21 Marks: 1/1

In 2010, Gail had a 179 deduction carryover of $15,000. In 2008, she elected 179 for an asset acquired at a cost of $115,000. Gails 179 business income limitation for 2010 is $127,000. Determine Gails 179 deduction for 2011.

Choose one answer. a. $15,000. b. $115,000. c. $127,000. d. $130,000. e. None of the other answers are correct.
$130,000 ($15,000 + $115,000), limited to $127,000.

Correct.

REF: p. 8-13 | p. 8-14

Correct Marks for this submission: 1/1. History of Responses: # 1 Grade Action Response $127,000. $127,000. Time 14:07:37 on 15/03/12 14:07:37 on 15/03/12 1 1 Raw score 1 1 Grade

2 Close&Grade Question 22 Marks: 1/1

On January 15, 2011, Penelope purchased the rights to a mineral interest for $12,000,000. At that time, it was estimated that the recoverable units would be 2,500,000. During the year, 300,000 units were mined and 175,000 units were sold for $3,000,000. Penelope incurred expenses during 2011 of $1,000,000. The percentage depletion rate is 22 percent. Determine Penelope's depletion deduction for 2011. Choose one answer. a. $1,000,000. b. $840,000. c. $660,000. d. $450,000. e. None of the other answers are correct. Cost Basis $12,000,000/2,500,000 = $4.80 per unit; Cost Depletion 175,000 units sold X $4.80 = $840,000 Percentage Depletion 22% X $3,000,000 = $660,000 Percentage limit ($3,000,000 - $1,000,000) X 50% = $1,000,000 Thus, the depletion deduction is $840,000. pp. 8-22 to 8-25 Correct Marks for this submission: 1/1. History of Responses: # 1 Grade Action Response $840,000. $840,000. Time 14:31:54 on 15/03/12 14:31:54 on 15/03/12 1 1 Raw score 1 1 Grade Correct.

2 Close&Grade Question 23 Marks: 1/1

On July 17, 2010, Kevin places in service a new automobile that cost $15,000. The car is used 80% for business and 20% for personal use. In 2011, he used the automobile 40% for business and 60% for personal use. Determine the cost recovery recapture for 2011. Choose one answer. a. $1,200. b. $0. c. $2,400. d. $3,000. e. None of the other answers are correct. MACRS ($15,000 X .20) = $3,000 (limited to $11,060*); $3,000 X 80% = $2,400 . Straight-line ($15,000 X .10) = $1,500 (limited to $11,060*); $1,500 X 80% = $1,200. Cost recovery recapture in 2011 = $1,200 ($2,400-$1,200) *These depreciation limits are indexed annually. REF: Example 30 | Table 8-1 | Table 8-3 Correct Marks for this submission: 1/1. History of Responses: # 2 Grade Action Response $1,200. $1,200. Time 19:46:51 on 22/04/12 19:46:58 on 22/04/12 1 1 Raw score 1 1 Grade Correct.

3 Close&Grade Question 24 Marks: 1/1

On June 1, 2011, James places in service a new automobile that cost $40,000. The car is used 60% for business and 40% for personal use. (Assume this percentage is maintained for the life of the car.) Determine the cost recovery deduction for 2011.

Choose one answer. a. $4,800. b. $1,836. c. $8,000. d. $11,060. e. None of the other answers are correct. Correct.

$40,000 X .20 = $8,000 (limited to $11,060*). $8,000 X 60% = $4,800. *These depreciation limits are indexed annually. REF: Example 24 | Table 8-1 Correct Marks for this submission: 1/1. History of Responses: # 1 Grade Action Response $4,800. Time 14:50:39 on 15/03/12 1 Raw score 1 Grade

2 Close&Grade $4,800. 14:50:39 on 15/03/12 1 1 Question 25 Marks: 1/1 The only asset Harry purchased during 2011 was a new seven-year class asset. The asset, which was listed property, was acquired on June 17 at a cost of $100,000. The asset was used 40% for business, 30% for the production of income, and the rest of the time for personal use. Harry always elects to expense the maximum amount under 179 whenever it is applicable. The net income from the business before the 179 deduction is $100,000. Determine Harrys maximum deduction with respect to the property for 2011. Choose one answer. a. $1,999. b. $4,998. c. $48,574. d. $75,176. e. None of the above. The listed property does not pass the predominantly business usage test. Therefore, 179 expensing cannot be taken. In addition, only straight-line cost recovery can be used. Maximum deduction ($100,000 X .0714 X 70%) =$4,998 pp. 8-4 to 8-15 and Table 8-3 Correct Marks for this submission: 1/1.
Chapter 9:

Correct.

1 Marks: 0/1 The 222 deduction for qualified tuition and related expenses is subject to the 2%-ofAGI floor on employee expenses. Choose one answer. a. True Incorrect. b. False The 222 amount is a deduction for AGI. p. 9-28 Incorrect Marks for this submission: 0/1. Question 2 Marks: 1/1 In determining whether someone is away from their tax home, the key consideration involves the need to duplicate certain living expenses. Choose one answer. a. True Correct. b. False p. 9-8 and Example 12 Correct Marks for this submission: 1/1. Question 3 Marks: 1/1 For tax purposes, "travel" is a broader classification than "transportation." Choose one answer. a. True Correct. b. False Travel expenses include transportation expenses and meals and lodging while away from home in the pursuit of a trade or business. p. 9-7 Correct Marks for this submission: 1/1. Question 4 Marks: 1/1 Mallard Corporation furnishes meals at cost to its employees by means of a cafeteria it maintains. The cost of operating the cafeteria is subject to the cutback adjustment. Choose one answer. a. True b. False
Example 32

Correct.

Correct Marks for this submission: 1/1. Question 5 Marks: 1/1

Daniel just graduated from college. The cost of moving his personal belongings from his parents' home to his first job site can qualify for the moving expense deduction. Choose one answer. a. True Correct. b. False There is no prohibition against claiming moving expenses if the first job is involved. pp. 9-11 and 9-12 Correct Marks for this submission: 1/1. Question 6 Marks: 1/1 After he finishes working at his main job, Martin returns home, has dinner, then drives to his second job. Martin may deduct the mileage between his first job and the second job. Choose one answer. a. True Correct. b. False The deduction is based on the mileage between the first and the second job. p. 9-5 and Example 5 Correct Marks for this submission: 1/1. Question 7 Marks: 1/1 The cost of tolls and parking are not allowed if the automatic mileage method is used. Choose one answer. a. True Correct. b. False Tolls and parking expenses can be claimed in addition to the amount of the automatic mileage allowance. p. 9-6 Correct Marks for this submission: 1/1. Question 8 Marks: 1/1 In terms of qualifying for an office in the home deduction, employees must satisfy the same requirements as self-employed taxpayers. Choose one answer. a. True Correct. b. False Employees must meet an additional test. The use must be for the convenience of the employer rather than being appropriate and helpful. p. 9-20 Correct

Marks for this submission: 1/1. Question 9 Marks: 1/1 Both employees and self-employed individual taxpayers must use Schedule A of Form 1040 to report their business-related expenses. Choose one answer. a. True b. False Correct.

Self-employed persons must use Schedule C of Form 1040 to report their business-related expenses. p. 9-3 Correct Marks for this submission: 1/1. Question 10 Marks: 1/1 A taxpayer who retires and returns to the United States from an overseas job need not satisfy the work requirement rule in order to deduct moving expenses. Choose one answer. a. True Correct. b. False The retired taxpayer need not satisfy the employment requirements on returning to the U.S. See Global Tax Issues on page 9-30. Correct Marks for this submission: 1/1. Question 11 Marks: 1/1 During the year, Matthew travels from Boston to Moscow on business. His time was spent as follows: 2 days travel (one day each way), 3 days business, and 3 days personal. His expenses for the trip were as follows (meals and lodging reflect only the business portion): Air fare Lodging Meals and entertainment $3,200 600 1,200

Presuming no reimbursement, deductible expenses are: Choose one answer. a. $2,400.

b. $3,200. c. $3,800. d. $4,400. e. $5,000 f. None of the other answers are correct. $3,200 + $600 + (50% X $1,200) = $4,400. Since the 7-days-or-less exception applies, the full airfare ($3,200) is allowed. In applying the 7-day test, the departure travel date is not counted. p. 9-11 and Footnote 18 Correct Marks for this submission: 1/1. Question 12 Marks: 1/1 No deduction is allowed for the participant's contribution, but distributions from the plan are not subject to income tax. This statement describes the operation of: Choose one answer. a. Roth IRAs. b. Keogh (H.R. 10) plans. c. Traditional IRAs. d. Both Roth IRAs and Keogh (H.R. 10) plans, but not Traditional IRAs. e. Both Keogh (H.R. 10) plans and Traditional IRAs, but not Roth IRAs. Keogh and traditional IRAs provide an upfront benefit in the form of a deduction (or exclusion). Neither allows a tax-free withdrawal. What is described fits only the Roth IRA. pp. 9-23 and 9-24 Correct Marks for this submission: 1/1. Question 13 Marks: 1/1 Emma, the regional sales director for a lingerie company, pays $1,000 to obtain a skybox for an evening production of "Icearama International." The skybox holds 10 seats and Emma invites 8 clients to the event. Nonluxury seats sell for $40 each. The refreshments served to Emma and her clients cost $300. A substantial business discussion was held before the show and Emma has all necessary substantiation. Emmas deduction is: Choose one answer. a. $310. b. $330. Correct. Correct.

c. $350. d. $650.

Correct.

e. None of the other answers are correct. 50% X ($400 + $300) = $350. The $400 is calculated by multiplying the 10 seats by $40 per seat. Example 37 Correct Marks for this submission: 1/1. Question 14 Marks: 1/1 Sierra holds two jobs. Her main job is with Forest Corporation and her part-time job is with Valley Company. On a typical workday, she drives her car as follows: home to Forest, Forest to Valley, and Valley to home. Applicable mileage is as follows: Home to Forest Forest to Valley Valley to home On a typical day, Sierras deductible mileage is: Choose one answer. a. 10. b. 20. c. 28. d. 32. e. None of the other answers are correct. The deduction is based on the 10 mile distance between the two jobs (Forest to Valley). p. 9-5 and Example 5 Correct Marks for this submission: 1/1. Question 15 Marks: 1/1 Bart made the following gifts during the year: To Shane, a key client ($2 of the amount listed was for $52 gift wrapping) To Joshua, Barts secretary, on Joshuas birthday 29 To Shanna, Barts boss, at Christmas 35 Presuming proper substantiation, Barts deduction is: Choose one answer. a. $50. Correct. Miles 8 10 14

b. $52. c. $77. d. $116.

Correct.

e. None of the above. $27 + $25 = $52. The cost of gift wrapping is allowed. No deduction is available for a gift to a superior. p. 9-20 Correct Marks for this submission: 1/1. Question 16 Marks: 1/1 During the year, Shirley went from Wilmington to Santiago (Chile) on business. She spent three days on business, two days on travel, and five days on vacation. Shirleys total travel expenses expenses are: Air fare to and from Chile $2,000 Business Days Lodging 600 Business Days Meals 500 Business Days Entertainment 300 Shirleys deductible employee business travel expenses are: Choose one answer. a. $3,400. b. $3,000. c. $2,400. d. $2,000. e. None of the other answers are correct. $1,000 [(5 days business/10 day trip) X $2,000 (air fare)] + $600 + $400 [50%($500 + $300)] = $2,000. The air fare has to be allocated as Shirley did not meet either the seven days (or less) or less than 25% personal use exceptions for foreign travel. p. 9-11; Example 24 Correct Marks for this submission: 1/1. Question 17 Correct.

Marks: 1/1 During the year, Hugh went from Cleveland to Fairbanks on business. Preceding a fiveday business meeting, he spent four days vacationing at a dude ranch. Excluding the dude ranch costs, his expenses for the trip are: Air fare Lodging Meals Entertainment $1,800 600 500 300

Presuming no reimbursement, deductible expenses are: Choose one answer. a. $3,200. b. $3,050. c. $2,800. d. $1,900. e. None of the other answers are correct. $1,800 + $600 + $400 [50%($500 + $300)] = $2,800. No allocation is required for domestic transportation costs (i.e., the airfare). Example 20 Correct Marks for this submission: 1/1. Question 18 Marks: 1/1 In which of the following is a deduction allowed for contributions to the plan, and no income tax consequences result from distributions to the participant at retirement. Choose one answer. a. Roth IRAs. b. Traditional IRAs. c. Keogh (H.R. 10) plans. d. Both Roth IRAs and Traditional IRAs, but not Keogh (H.R. 10) plans. e. None of the other answers are correct. f. Both Traditional IRAs and Keogh (H.R. 10) plans, but not Roth IRAs. Correct. Correct.

A deduction is allowed for contributions to traditional IRAs and Keogh plans but not to Roth IRAs. Distributions are free of income tax in the case of Roth IRAs but not Traditional IRAs and Keogh (H.R. 10) plans. Thus, the combination of a deduction and tax-free distributions does not exist. pp. 9-23 and 9-24 Correct Marks for this submission: 1/1. Question 19 Marks: 1/1 Maggie, who holds a bachelor of education degree, is a middle school teacher in Hartford, Connecticut. The school board recently changed its minimum education requirement by prescribing five years of college training. Existing teachers, such as Maggie, are allowed 5 years in which to acquire the additional year of education. Pursuant to this requirement, Maggie spends her 2010 summer break attending University of Connecticut taking education courses. Her expenses are as follows: Books and tuition Meals Lodging Laundry while in travel status Transportation $4,500 800 900 350 950

Her education expense deduction is: Choose one answer. a. $7,500. b. $7,100. c. $6,750. d. $4,500. e. None of the other answers are correct. $4,500 + (50% X $800) + $900 + $350 + $950 = $7,100. p. 9-13 and Example 27 Correct Marks for this submission: 1/1. Question 20 Marks: 1/1 Which, if any, of the following expenses is a miscellaneous deduction not subject to the 2 percent floor? Choose one answer. a. Malpractice insurance premiums. Correct.

b. Union dues. c. Job hunting expenses. d. Hobby expenses. Correct. e. All of the above expenses are subject to the 2% limitation. All of the options are itemized deductions subject to the 2 percent floor. p. 9-27 Correct Marks for this submission: 1/1. Question 21 Marks: 1/1 During the year, Horace went from Sacramento to Portland, Maine. After five days of business meetings, he took four days of vacation to go sightseeing. Horaces expenses for the trip are as follows: Air fare $ 600 720 630 60

Lodging (9 days X $80) Meals (9 days X $70) Airport limo Horaces deduction is: Choose one answer. a. $1,117. b. $1,235. c. $1,950. d. $2,010.

Correct.

e. None of the other answers are correct. $600 + $400 ($80 X 5 days) + $175 [50% X ($70 X 5 days)] + $60 = $1,235. p. 9-9 and Example 20 Correct Marks for this submission: 1/1. Question 22 Marks: 0/1 Carl is the regional manager for a national chain of auto-parts stores and is based in Billings, Montana. When the company opens new stores in Rapid City, South Dakota, Carl is given the task of supervising their initial operation. For three months, he works weekdays in Rapid City and returns home on weekends. He spends $270 returning to

Billings but would have spent $290 had he stayed in Rapid City for the weekend. As to the weekend trips, how much, if any, qualifies as a deduction? Choose one answer. a. $0, since the trips are personal and not work related. b. $0, since Carls tax home has changed from Billings to Rapid City. c. $290. d. $270. e. None of the above. Carls assignment in Rapid City is temporary, so his tax home has not changed (choice b.). Carls deduction is limited to the lesser of what he actually spent and what he would have spent had he not returned home (choice d.). Example 10 Incorrect Marks for this submission: 0/1. Question 23 Marks: 1/1
Due to a merger, Elizabeth transfers from New York City to Albuquerque. Under a new job description, she is reclassified from employee to independent contractor status. Her moving expenses, which are not reimbursed, are as follows: Transportation Meals Lodging Cost of moving household goods Penalty for breaking New York City apartment lease Elizabeths deductible moving expense is: $1,200 300 400 3,000 5,000

Incorrect.

Choose one answer. a. $4,600. b. $4,750. c. $4,900. d. $9,600. e. None of the other answers are correct. $1,200 (transportation) + $400 (lodging) + $3,000 (moving household goods) = $4,600. Meals are not qualified moving expenses nor are penalties for breaking a lease. It is immaterial that Elizabeths status as an employee changed to that of an independent contractor. p. 9-12 and Example 25 Correct Marks for this submission: 1/1. Question 24 Marks: 1/1 Jordan made the following gifts during the year: To Gabby, a key client ($10 of the amount listed was for gift wrapping)$50 Correct.

To Kelly, Jordan's secretary, on Kellys birthday To Bradley, Jordan's boss, at Christmas Presuming proper substantiation, Jordan's deduction is: Choose one answer. a. $60. b. $85. c. $90. d. $160. Correct.

40 70

e. None of the other answers are correct. $35 + $25 = $60. The cost of gift wrapping is allowed. No deduction is available for a gift to a superior. p. 9-20 Correct Marks for this submission: 1/1. Question 25 Marks: 1/1 Kyle is the city sales manager for Country Cooking, a national fast food franchise. Every working day, Kyle drives his car as follows: Miles Home to office 10 Office to Country Cooking No. 1 12 Country Cooking No. 1 to No. 2 14 Country Cooking No. 2 to No. 3 11 Country Cooking No. 3 to home 15 Kyles deductible mileage is:

Choose one answer. a. 0 miles. b. 15 miles. c. 37 miles. d. 52 miles. e. None of the above. 12 miles + 14 miles + 11 miles = 37 miles. The mileage for driving from his home to the office (10 miles) and from the last worksite to home (15 miles) is not deducible. Example 6 Correct Correct.

Tax acc chapt 1 exam:

As it is consistent with the wherewithal to pay concept, the tax law requires a seller to recognize gain in the year the installment sale took place. Answer: True Correct. False

The seller is taxed in the year the installment payments are received. REF: p. 1-30

A provision in the law that compels accrual basis taxpayers to pay a tax on prepaid income in the year received and not when earned is consistent with generally accepted accounting principles. Choose one answer. a. True b. False

It is inconsistent with accounting rules, although it can be justified under the wherewithal to pay concept. p. 1-32 Correct

The annual exclusion, currently $13,000, is available for both gift and estate tax purposes. Answer:

True Correct.

False

The annual exclusion is not available for estate tax purposes. REF: p. 1-13

For individual taxpayers, the interest rate for income tax refunds (overpayments) is not the same as that applicable to assessments (underpayments). Answer: True Correct. False

There exists no difference in the applicable interest rates for individual taxpayers. REF: p. 1-24

The Federal income tax on corporations generates more revenue than the Federal income tax on individuals. Answer: True Correct. False

Just the opposite is the case. REF: Figure 1.1

The Federal estate tax was originally intended to prevent large concentrations of wealth from being kept within a family for many generations. Choose one answer. a. True

b. False

p. 1-11

A major disadvantage of a flat tax type of income tax is its complexity. Answer: True Correct. False

Just the opposite is the case. Advantage is its simplicity REF: p. 1-20

Marks: 1/1 Characteristics of the "Fair Tax" (i.e., national sales tax) include which, if any, of the following:
d. Abolition of all Federal income and payroll taxes as well as the Federal estate and gift taxes.

Equity considerations can be used to justify: Choose one answer.

a. A tax credit allowed for amounts spent to furnish care for cer minor or disabled dependents. b. Tax deductions which encourage home ownership. c. A Federal deduction for state and local income taxes.

d. Allowing farmers to expense (rather than capitalize) certain s

water conservation expenditures. e. None of the other answers are correct.

p. 1-28 Correct Marks for this s

Scott files his tax return 65 days after the due date. Along with the return, Scott remits a check for $50,000 which is the balance of the tax owed. Disregarding the interest element, Scott's total failure to file and to pay penalties are:

choose one answer. a. $750. b. $6,000. c. $7,500. d. $8,250. e. None of the other answers are correct.

Following the procedure set forth in Example 15, the penalty is determined as follows: Failure to pay penalty [0.5% X $50,000 X 3 (two months violation)] Plus: Failure to file penalty [5% ? $50,000 X 3 (three months violation)] Less: Failure to pay penalty Total penalties Correct $7,500 (750) 6,750 $7,500 $ 750

A characteristic of FUTA is that: Choose one answer. a. It is imposed on both employer and employee. b. It is imposed solely on the employee.

c. Compliance requires following guidelines issued by both state and Federal regulatory authorities.

d. It is applicable to spouses of employees but not to any childre under age 18. e. None of the other answers are correct.

FUTA is imposed only on the employer (choices a. and b.). Choice d. refers to FICA. Since the administration of FUTA is shared by Federal and state governments, employers must comply with the rules issued by each (choice c.). REF: p. 1-18 | p. 1-19

Which statement about state income taxes is false: Choose one answer. a. They often piggyback to the Federal version. b. Some states decouple from the Federal version. c. Never apply to visiting athletes. d. States provide occasional amnesty programs. e. Often apply to both individual and corporate taxpayers.

Many states piggyback to the Federal system (choice a.). Some states, due to revenue shortfalls, have decoupled from various provisions of the Federal version (choice b.). The "jock tax," although much criticized, is very much in being (choice c.). Some states even have had more than one amnesty period (choice d.). Invariably, states that have an individual income tax will have one that is applicable to corporations (choice e.). pp. 1-14, 1-15, and p. 1-16

Marks: 1/1

Federal excise taxes that are no longer imposed include: Choose one answer. a. Tax on air travel. b. Tax on wagering. c. Tax on the manufacture of sporting equipment. d. Tax on jewelry. e. None of the above.

REF: p. 1-9

The U.S. (either Federal, state, or local) does not impose: Choose one answer. a. Franchise taxes. b. Severance taxes. c. Occupational fees. d. Customs duties. e. Export duties.

REF: p. 1-11 | p. 1-17 | p. 1-18

Property can be transferred within the family group by gift or at death. One motivation for preferring the gift approach is: Choose one answer. a. To take advantage of the per donee annual exclusion.

b. To avoid a future decline in value of the property transferred

c. To take advantage of the higher unified transfer tax credit av under the gift tax. d. To shift income to higher bracket donees. e. None of the above.

The per donee annual exclusion is only available for gift tax purposes (choice a.). Ideally, gifts should involve property that is expected to appreciate in value (choice b.). A higher unified tax credit is available for estate tax purposes (choice c.). Usually the donor is trying to shift future income to lower bracket donees (choice d.). REF: p. 1-14 Correct

Which, if any, of the following statements best describes the history of the Federal income tax? Choose one answer. a. It existed during the civil war.

b. The Federal income tax on corporations was held by the U. Supreme Court to be contrary to the U.S. Constitution.

c. The Federal income tax on individuals was held by the U.S. Supreme court to be allowable under the original U.S. Constit

d. Both the Federal income tax on individuals and on corporat were held by the U.S. Supreme Court to be contrary to the U.S Constitution. e. None of the other answers are correct.

REF: p. 1-3 Correct Surgical instruments owned and used in a physician's practice are an example of personal use personalty. Choose one answer. a. True b. False

This is an example of business use personalty. p. 1-8 and Footnote 8

Keeping two sets of books might be an indication of fraud on the part of the taxpayer. Choose one answer. a. True b. False

p. 1-23 and Example 11

Equity considerations can be used to justify: Choose one answer.

a. The treatment of gain resulting from an involuntary conver b. The installment sales rules. c. The NOL carryover and carryback rules. d. All of the above. e. None of the above.

The deferral of gain from involuntary conversions, the installment sales rules, and the NOL carryover and carryback rules are all equity considerations (options a., b. and c.). pp. 1-23 to 1-30 Social considerations can be used to justify: Choose one answer. a. The exclusion for accident and health plans. b. The installment sales rules. c. Deduction for research and experimental expenditures. d. The tax treatment of prepaid subscription revenue. e. None of the above.

The exclusion for accident and health plans are social considerations (options a.). The installment sales rules are an equity consideration (option b.). The deduction of research and experimental expenditures is based on economic considerations (option c.). The tax treatment of prepaid subscription revenue is a political consideration (option d). pp. 1-23 to 1-30 Correct Marks for this submission: 1/1.

Which of the following is not characteristic of the IRS audit procedure? Choose one answer.

a. The percentage of individual income tax returns that the IRS au significantly decreased over the last five years. b. An office audit takes place at the office of the IRS.

c. One of the factors that leads to an audit is the information provid informants.

d. IRS special agents are typically involved when fraud is suspecte e. All of the above arecharacteristic of the IRS audit procedure .

The number of returns audited by the IRS has significantly increased recently (choice a.). An office audit takes place at the office of the IRS (choice b.). The participation of a special agent occurs when fraud is suspected-the type of audit involved is immaterial (choice d). pp. 1-21 and 1-22 Political considerations can be used to justify: Choose one answer. a. The foreign tax credit. b. The credit for child and dependent care expenses. c. The deduction for charitable contributions. d. The tax treatment of prepaid subscription revenue. e. None of the other answers are correct.

The foreign tax credit is an equity consideration. The credit for child and dependent care expenses and the deduction for charitable contributions are both social consideration. The tax treatment of prepaid subscription revenue is a political consideration. pp. 1-23 to 1-30

Marks: 1/1

Which, if any, of the following is a typical characteristic of an ad valorem tax? Choose one answer.

a. Taxpayer compliance is greater for personal use property th real property.

b. The tax on automobiles never considers the value of the ve c. Few states impose a tax on real property.

d. The tax on intangibles generates considerable revenue sinc difficult for taxpayers to avoid. e. None of the above.

Taxpayer compliance is greater with real property (choice a.). All states impose a tax on realty (choice c.). Because it is easily avoided and does not generate much revenue, very few states impose a tax on intangibles (choice d.). pp. 1-7 and 1-8

Which of the following is characteristic of the IRS audit procedure? Choose one answer. a. The IRS uses a DIP score to select returns for audit. b. An office audit takes place at the office of the taxpayer.

c. The percentage of individual income tax returns audited h increased over the last ten years. d. Only IRS special agents can conduct field audits. e. None of the other answers are correct.

Although only about 1% of all returns are audited, the percentage of returns audited in the fiscal year ended Sept 30, 2007 was the highest since 1998. REF: p. 1-22 | p. 1-23

Marks: 0/1

Stealth taxes have the effect of generating additional taxes from higher income taxpayers. Answer: True Correct. False

Stealth taxes phase out (or eliminate) certain tax benefits from higher income taxpayers. REF: p. 1-33

Chapter 2 Exam

Review of attempt 1
Finih eie

Started on Wedneda, 1 Feba 2012, 03:49 PM Completed on Thda, 2 Feba 2012, 07:32 PM Time taken 1 da 3 ho Marks 25/25 Grade 50 o of a maimm of 50 (100%)
2/2/12 ACC-3400: Chapter 2 Exam open.uvu.edu/mod/qui/review.php?attempt=392395&showall=true 2/13

Coec. The ciaion i coec. REF: p. 2-18 Coec


Mak fo hi bmiion: 1/1.
Hio of Repone: # Action Response Time Ra score Grade 1 Gade Fale 16:43:57 on 1/02/12 1 1 2 Close&Grade False 16:43:57 on 1/02/12 1 1

3
Mak: 1/1

Chooe one ane. a. Te b. Fale Coec. The Inenal Reene Code nee conol hen hee i a conflicing a ea beeen he Unied Sae and a foeign con. Secion 7872(d) indicae ha neihe he Code no a eaie conol in he cae of conadico poiion. Inead, he "la in ime" docine i ilied o ha he mo ecen iem i applicable. p. 2-20 Coec
Mak fo hi bmiion: 1/1.
Hio of Repone: # Action Response Time Ra score Grade 1 Gade Fale 07:48:14 on 2/02/12 1 1 2 Close&Grade False 07:48:14 on 2/02/12 1 1

4
Mak: 1/1 Chooe

one ane. a. Te b. Fale Coec. The Inenal Reene Code a fi codified in 1954.

The fi codified eion a he Inenal Reene Code of 1939. The ne ecodificaion a in 1954. p. 2-2 Coec
Mak fo hi bmiion: 1/1.
Hio of Repone: # Action
2/2/12 ACC-3400: Chapter 2 Exam open.uvu.edu/mod/qui/review.php?attempt=392395&showall=true 3/13

Response Time Ra score Grade 1 Gade Fale 16:53:02 on 1/02/12 1 1 2 Close&Grade False 16:53:02 on 1/02/12 1 1

5
Mak: 1/1

Chooe one ane. a. Te b. Fale Coec. Pobabl he be combinaion of eleconic a eoce i o condc da-o-da ok on online acce em agmened ih CD eache hee i i jdged ciical o do o. J he oppoie i he cae. p. 2-24 Coec
Mak fo hi bmiion: 1/1.
Hio of Repone: # Action Response Time Ra score Grade 1 Gade Fale 16:54:13 on 1/02/12 1 1 2 Close&Grade False 16:54:13 on 1/02/12 1 1

6
Mak: 1/1

Chooe one ane. a. Te b. Fale Coec. Regla deciion of he Ta Co geneall inole noel ie peiol eoled b he U.S. Ta Co. Noe, hoee, omeime he ie in a Regla deciion of he Ta Co ae no noel. p. 2-17 Coec
Mak fo hi bmiion: 1/1.
Hio of Repone: # Action Response Time Ra score Grade 1 Gade Fale 17:01:05 on 1/02/12 1 1 2 Close&Grade False 17:01:05 on 1/02/12 1 1

7 The compe-baed CPA eaminaion ha fo ecion ih


efal e, mliple-choice, and cae die (called imlaion).
2/2/12 ACC-3400: Chapter 2 Exam open.uvu.edu/mod/qui/review.php?attempt=392395&showall=true 4/13

Marks: 1/1

Answer: True False Correct. No true-false questions appear on the CPA exam. REF: p. 2-38 Correct
Marks for this submission: 1/1.
History of Responses: # Action Response Time Ra score Grade 1 Grade False 17:08:53 on 1/02/12 1 1 2 Close&Grade False 17:08:53 on 1/02/12 1 1

8
Marks: 1/1

Choose one answer. a. True b. False Correct. A Proposed Regulation under 274 of the Code would be cited as follows: Prop. Reg. 274. Regulation section numbers contain a prefix number (usually 1) followed by a period (such as, 1.). Therefore, the correct cite is Prop. Reg. 1.274. pp. 2-7 and 2-8 Correct
Marks for this submission: 1/1.
History of Responses: # Action Response Time Ra score Grade 2 Grade False 19:30:39 on 2/02/12 1 1 3 Close&Grade False 19:30:39 on 2/02/12 1 1

9
Marks: 1/1

Answer: True False A taxpayer must pay any tax deficiency assessed by the IRS and sue for a refund to bring suit in the U.S. District Court. Correct.
Onl in the Ta Court can jurisdiction be obtained ithout first paing the
2/2/12 ACC-3400: Chapter 2 Exam open.uvu.edu/mod/qui/review.php?attempt=392395&showall=true 5/13

assessed ta deficienc. REF: Concept Summar 2.1

Correct
Marks for this submission: 1/1.
History of Responses: # Action Response Time Ra score Grade 1 Grade True 17:57:54 on 1/02/12 1 1 2 Close&Grade True 17:57:54 on 1/02/12 1 1

10
Marks: 1/1 Answer:

True False Temporary Regulations are published in The Fedeal Regie. Correct. Proposed, Temporary and final Regulations are published in the Federal Register, in the Internal Revenue Bulletin, and by major tax services. REF: p. 2-8 Correct
Marks for this submission: 1/1.
History of Responses: # Action Response Time Ra score Grade 1 Grade True 18:38:39 on 1/02/12 1 1 2 Close&Grade True 18:38:39 on 1/02/12 1 1

11
Marks: 1/1

Choose one answer. a. Legal periodicals. b. Treatises. c. Technical Advice Memoranda. Correct. d. Legal opinions. e. None of above. Which of the following is substantial authority for purposes of the accuracy related penalty? Substantial authority for purposes of the accuracy-related penalty include the following secondary materials: letter rulings, general counsel and technical advice memoranda, and the Bluebook. p. 233
2/2/12 ACC-3400: Chapter 2 Exam open.uvu.edu/mod/qui/review.php?attempt=392395&showall=true 6/13

Coec
Mak fo hi bmiion: 1/1.
Hio of Repone: # Action Response Time Ra score Grade 1 Gade Technical Adice Memoanda. 18:42:45 on 2/02/12 11

2 Close&Grade Technical Adice Memoranda. 18:42:45 on 2/02/12 11

12
Mak: 1/1 Chooe

one ane. a. 40 T.C. 1018. b. 159 F. 2d 848 (CA-2, 1947). c. 354 F. Spp. 1003 (D. C. Ga, 1972). d. 914 F. 2d 396 (CA-3, 1990). Coec. Which ciaion efe o a Thid Cici Co of Appeal deciion? REF: p. 2-19 Coec
Mak fo hi bmiion: 1/1.
Hio of Repone: # Action Response Time Ra score Grade 2 Gade 914 F. 2d 396 (CA-3, 1990). 18:49:54 on 2/02/12 11 3 Close&Grade 914 F. 2d 396 (CA-3, 1990). 18:49:54 on 2/02/12 11

13
Mak: 1/1 Chooe

one ane. a. Penal ae. b. S Copoaion. c. Ecie ae. d. Pane and Panehip. Coec. e. None of hee opic. Sbchape K of he Inenal Reene Code coe ha opic? Sbchape K coe Pane and Panehip. p. 2-6 Coec
Mak fo hi bmiion: 1/1.
2/2/12 ACC-3400: Chapter 2 Exam open.uvu.edu/mod/qui/review.php?attempt=392395&showall=true 7/13

History of Responses: # Aci Ree Tie Ra ce Gade

1 Grade Partners and Partnerships. 20:22:10 on 1/02/12 11 2 Ce&Gade Pae ad Paehi. 20:22:10 1/02/12 11

14
Marks: 1/1 Choose

one answer. a. Give equal weight to the Code and regulations. Correct. b. Give more weight to the Code rather than to a regulation. c. Give more weight to the regulation rather than to the Code. d. Give less weight to the Code rather than to a regulation. e. None of the other answers are correct. In assessing the importance of a regulation, an IRS agent must: REF: . 2-8, 2-9, and 2-28 Correct
Marks for this submission: 1/1.
History of Responses: # Aci Ree Tie Ra ce Gade 1 Grade Give equal weight to the Code and regulations. 20:25:33 on 1/02/12 11 2 Ce&Gade Gie ea eigh he Cde ad egai. 20:25:33 1/02/12 11

15
Marks: 1/1

Choose one answer. a. U.S. Tax Court. b. U.S. District Court. Correct. c. U.S. Court of Federal Claims.

d. Small Cases Division of the Tax Court. Which trial courts jurisdiction depends on the geographical location of the taxpayer?
2/2/12 ACC-3400: Chapter 2 Exam open.uvu.edu/mod/qui/review.php?attempt=392395&showall=true 8/13

. U.S. S C REF: . 2-12 C S 2.1 C


M : 1/1.
H R: # Action Response Time Ra score Grade 1 G U.S. D C. 20:26:35 1/02/12 1 1 2 Close&Grade U.S. District Court. 20:26:35 on 1/02/12 1 1

16
M: 1/1 C

. . T 16 . C. . T R M . . T IRS . . A . . N . W no U.S. T C? T 19 . C S 2-1 C


M : 1/1.
H R: # Action Response Time Ra score Grade 1 G T 16 . 20:40:29 1/02/12 1 1 2 Close&Grade There are 16 judges. 20:40:29 on 1/02/12 1 1

17
M: 1/1 C

. . P . . E . . E . S A I R C ?
2/2/12 ACC-3400: Chapter 2 Exam open.uvu.edu/mod/qui/review.php?attempt=392395&showall=true 9/13

d. Emplomen ae. e. Income Tae Coec. Sbile A coe income ae. p. 2-6 Coec
Mak fo hi bmiion: 1/1.
Hio of Repone: # Action Response Time Ra score Grade 1 Gade Income Tae 20:42:17 on 1/02/12 1 1 2 Close&Grade Income Taes 20:42:17 on 1/02/12 1 1

18
Mak: 1/1 Chooe

one ane. a. Taaion Commiee. b. Wa and Mean Commiee. c. Finance Commiee. Coec. d. Bdge Commiee. e. None of he ohe ane ae coec. Ta bill ae handled b hich commiee in he U.S. Senae? p. 2-4 Coec
Mak fo hi bmiion: 1/1.
Hio of Repone: # Action Response Time Ra score Grade 1 Gade Finance Commiee. 20:54:01 on 1/02/12 1 1 2 Close&Grade Finance Committee. 20:54:01 on 1/02/12 1 1

19
Mak: 1/1 Chooe

one ane. a. Income ae. Coec. b. Eae and gif ae. c. Ecie ae. d. Emplomen ae. Sbile A of he Inenal Reene Code coe hich ae?
2/2/12 ACC-3400: Chapter 2 Exam open.uvu.edu/mod/qui/review.php?attempt=392395&showall=true 10/13

e. All of the other answers. p. 2-5 Correct


Marks for this submission: 1/1.
History of Responses: # Acion Repone Time Ra coe Gade

1 Grade Income taxes. 20:55:41 on 1/02/12 1 1 2 Cloe&Gade Income ae. 20:55:41 on 1/02/12 1 1

20
Marks: 1/1

Choose one answer. a. Actions on Decisions. b. Revenue Procedures. c. Technical Advice Memoranda. d. General Counsel Memoranda. e. All of the other answers are administrative sources. Correct. Which of the following is no an administrative source of the tax law? REF: p. 2-7 to 2-11 Exhibit 2.1 Correct
Marks for this submission: 1/1.
History of Responses: # Acion Repone Time Ra coe Gade 1 Grade All of the other answers are administrative sources. 21:02:12 on 1/02/12 11 2 Cloe&Gade All of he ohe ane ae adminiaie oce. 21:02:12 on 1/02/12 11

21
Marks: 1/1

Choose one answer. a. The Supreme Court did not disagree with the First Circuit Court of Appeals. b. The First Circuit Court of Appeals disagreed with the Tax Court. Correct. What statement is no true about this citation: C. W. Seadman, 50 T.C. 369 (1968), aff'd 424 F.2d 1 (CA-1, 1970), cert. den. 400 U.S. 869 (1970)?
2/2/12 ACC-3400: Chapter 2 Exam open.uvu.edu/mod/qui/review.php?attempt=392395&showall=true 11/13

c. The apae did no hae o pa he

deficienc befoe going o co. d. The Ta Co deciion a on page 369. e. All of he aboe ae e. pp. 2-21 and 2-22 Coec
Mak fo hi bmiion: 1/1.
Hio of Repone: # Action Response Time Ra score Grade 1 Gade The Fi Cici Co of Appeal diageed ih he Ta Co. 19:07:56 on 2/02/12 11 2 Close&Grade The First Circuit Court of Appeals disagreed ith the Ta Court. 19:07:56 on 2/02/12 11

22
Mak: 1/1

Chooe one ane. a. 1913. b. 1933. c. 1954. Coec. d. 1957. e. None of he aboe. The Inenal Reene Code a codified in hich of he folloing ea? REF: p. 2-2 Coec
Mak fo hi bmiion: 1/1.
Hio of Repone: # Action Response Time Ra score Grade 1 Gade 1954. 21:11:47 on 1/02/12 1 1 2 Close&Grade 1954. 21:11:47 on 1/02/12 1 1

23 Wha aemen i no e ih epec o Tempoa


Reglaion?
2/2/12 ACC-3400: Chapter 2 Exam open.uvu.edu/mod/qui/review.php?attempt=392395&showall=true 12/13

Marks: 1/1

Choose one

answer. a. May be cited as precedent. b. Issued as Proposed Regulations. c. Automatically expire within three years after the date of issuance. d. Found in the Federal Register. e. All of the above statements are true. Correct. pp. 2-7 and 2-8 Correct
Marks for this submission: 1/1.
History of Responses: # Acion Repone Time Ra coe Gade 1 Grade All of the above statements are true. 21:14:47 on 1/02/12 11 2 Cloe&Gade All of he aboe aemen ae e. 21:14:47 on 1/02/12 11

24
Marks: 1/1

Choose one answer. a. Temporary. b. Legislative. Correct. c. Interpretative. d. Procedural. e. None of the other answers are correct. Which of the following types of Regulations has the highe tax validity? REF: p. 2-7 p. 2-29 Correct
Marks for this submission: 1/1.
History of Responses: # Acion Repone Time Ra coe Gade 1 Grade Legislative. 21:15:38 on 1/02/12 1 1 2 Cloe&Gade Legilaie. 21:15:38 on 1/02/12 1 1
2/2/12 ACC-3400: Chapter 2 Exam open.uvu.edu/mod/qui/review.php?attempt=392395&showall=true 13/13

25
Marks: 1/1

Choose

one answer. a. Research Institute of America. b. Commerce Clearing House. Correct. c. Prentice-Hall. d. LexisNexis. e. None of the above. Which publisher offers the Standard Federal Tax Reporter? REF: p. 2-22 Correct
Marks for this submission: 1/1.
History of Responses: # Action Response Time Ra score Grade 1 Grade Commerce Clearing House. 21:25:29 on 1/02/12 11 2 Close&Grade Commerce Clearing House. 19:32:40 on 2/02/12 11

Finish review You are logged in as Heidi Dalton (Logout)

Validate HTML Section 508 Check WCAG__

Chapter 3:
Marks: 1/1

Tad claims his 70-year-old father as a dependent. Tad may also claim an additional standard deduction for his father's age. Choose one answer. a. True b. False Correct.

The additional standard deduction is only available for the taxpayer or the spouse, not the dependents. The father could claim the additional standard deduction for being 65 or over if he files his own return. Example 10 Correct Marks for this submission: 1/1.

Lope is a citizen of Spain and a resident of the U.S. If he files a U.S. income tax return, Lope can claim the standard deduction. Choose one answer. a. True b. False Correct.

Either U.S. citizenship or residency will suffice. p. 3-9 Correct Marks for th Recent changes to eliminate the marriage penalty place married persons filing seperately in the same tax position as single persons. Choose one answer. a. True b. False Correct.

Many tax benefits (e.g. child tax credit, earned income credit) ate still unavailable to married persons filing seperately. p. 3-27

Correct The Tax Tables will yield the exact same tax liability as the Tax Rate Schedules. Choose one answer. a. True b. False Correct.

The tax tables assign the liability from the middle of a range of taxable income to a range of taxable income. The Tax Rate Schedule may be a few dollars more or less than the tax table amount.

Ref p. 3-29 Correct Marie furnishes more than 50% of the support of her son and daughter-in-law who live with her. Even if the son and daughter-in-law file a joint return, Marie can claim them as dependents. Choose one answer. a. True b. False Correct.

Only if certain conditions are satisfied (e.g., they did not have to file but did so to obtain a refund), the son and daughter-in-law can qualify as Marie's dependents. p. 3-18 and Example 28 Correct Marks for this submission: 1/1.

In 2010, Sally is 72 and single. If she has itemized deductions of $6,000, she should claim the standard deduction alternative. Answer: True Correct. False

The standard deduction yields $7,250 ($5,800 + $1,450). Example 7 Correct Marks for this submission: 1/1.

Marks: 1/1

Stealth taxes are directed at higher income taxpayers. Choose one answer.

a. True b. False

Correct.

Such stealth taxes as the phaseout of exemptions do not begin until taxpayers reach significant income levels. Tax in the News on p. 3-20 Correct

Gwen, a widow, dies on January 7, 2011. Even though she lived for only a week, her final income tax return can claim the full amount of the standard deduction for 2011. Choose one answer. a. True b. False Correct.

No proration of the standard deduction is necessary in this case. p. 3-9 Correct Marks for this submission: 1/1. The amount of the standard deduction allowed varies with the filing status of the taxpayer involved. Choose one answer. a. True b. False Correct.

pp. 3-6 to 3-8 and Tables 3-1 and 3-2 Correct Marks for this submission: 1/1.

As used in the income tax formula, gross income would not include the receipt of a loan the taxpayer obtained from a bank. Choose one answer. a. True b. False Correct.

Borrowing money does not result in gross income. REF: Example 1 Correct

Marks: 1/1

In 2011, Weston had the following transactions: Salary Damages for personal injury (car accident) Damages (punitive-same accident) Life insurance proceeds (Weston was the beneficiary of his uncle's policy) Repayment of a loan made to sister several years ago Lottery prize Weston's AGI is: Choose one answer. a. $70,000. b. $120,000. c. $140,000. d. $170,000. e. None of the other answers are correct. Correct. $ 60,000 20,000 50,000 100,000 20,000 10,000

$120,000 (60,000 Salary + $50,000 Punitive Damages + $10,000 Lottery prize). The compensation for personal injury and life insurance proceeds are excluded from income. The loan repayment is a return of his capital.

p. 3-5, 3-34 and exhibit 3-3 Correct

For tax year 2011, an exception to the kiddie tax rules includes: Choose one answer. a. A child who is a full-time student. b. A child who is 18 years old. c. A child who is married and files Correct. a joint return. d. A child whose unearned income is more than half of his or her support. e. None of the other answers are correct. Student status (choice a.) is relevant only to include, in the application of the tax, those at least 19 but under age 24. Choice c. relates to pre-2008 rules. Choice d. would be an exception if the reference was to earned income (not unearned income). Correct Marks for this submission: 1/1.

Marks: 1/1

Which, if any, of the following statements relating to the standard deduction is correct? Choose one answer. a. If a taxpayer dies during the year, his (or her) standard deduction must be prorated. b. If spouses file separate returns, both spouses must claim the standard deduction (rather than itemize their deductions from AGI).

c. If a taxpayer is claimed as a Correct. dependent of another, his (or her) additional standard deduction is allowed in full (i.e., no adjustment is necessary). d. If a taxpayer is claimed as a dependent of another, no basic standard deduction is allowed. In the case of death, no apportionment is required and the full standard deduction is allowed (choice a.). If married taxpayers file separate returns and one spouse itemizes, the other spouse must also itemize. However, there is no requirement that they each claim the standard deductionalthough they may choose to do so (choice b.). A basic standard deduction is allowed for dependents although its determination is subject to special rules (choice d.). REF: Example 10 Correct

Which of the following, if any, is considered in applying the support test for dependency exemption purposes? Choose one answer. a. A scholarship received by the person to be claimed as a dependent. b. The income of a dependent that could have been used for support. c. The citizenship of the person to be claimed as a dependent. d. Capital expenditures made on behalf of the person to be claimed as a dependent. e. None of the above. Correct.

A scholarship (choice a.) is disregarded. The income of a dependent (choice b.) would be relevant only if used for support. Such income is, of course, fully considered in applying the gross income test. The citizenship of the dependent (choice c.) has nothing to do with the support test. p. 3-11 and Examples 13 to 15 Correct Marks for this submission: 1/1.

Gerald has the following capital transactions: LTCG Long-term collectible gain STCG STCL After the netting process, the following results: Choose one answer. a. Long-term collectible gain of $1,000. b. LTCG of $3,000, Long-term collectible gain of $1,000, and a STCL of $3,000. c. LTCG of $3,000, Long-term collectible gain of $1,000, and a STCL carryover to 2003 of $3,000. d. LTCG of $1,000. e. None of the other answers are correct. First, the STCG and STCL are combined, resulting in a STCL of $3,000. Of this STCL, $1,000 is applied against the collectible gain of $1,000, and the $2,000 balance is applied against the LTCG of $3,000. The result is a LTCG of $1,000. Example 47 Correct Tim, age 16, is claimed as a dependent by his grandmother. During 2011, Tim had interest income from City of Omaha bonds of $1,000 and earnings from a part-time job of $700. Tim's taxable income is: Choose one answer. a. $0. b. $1,700 - $700 - $850 = $150. c. $1,700 - $1000 = $750. d. $1,700 - $850 = $850. e. None of the other answers are correct. Correct. Correct. $3,000 1,000 2,000 5,000

His taxable income less his exemption is less than zero. 700-(700+300) Municipal bond interest is not taxable. p. 3-9, Exhibit 3-1 Correct Hubert and Edie filed a joint return. They provide more than 50% of the support of Sophie, Gabe, and Isaiah. Sophie (age 19) is Hubert's niece and earns $9,000 from a part-time job. Gabe (age 25) is their son and is a full-time law student. He received from the university a $5,600 scholarship for room and board. Isaiah is Edie's father and is a citizen and resident of Israel. How many personal and dependency exemptions can Hubert and Edie claim on their Federal income tax return? Choose one answer. a. Two. b. Three. c. Four. d. Five. e. None of the above. Correct.

Hubert and Edie can claim two personal exemptions. They cannot claim Sophie and Gabe due to the gross income test. Sophie is not a child under age 19 and Gabe, although a full-time student, is not under age 24. In Gabes case, the taxable portion (i.e., room and board) is not disregarded in applying the gross income test. Edies father does not qualify as their dependent due to the citizenship or residency test. pp. 3-13 and 3-14 Correct

Marks: 1/1

Perry is in the 33% tax bracket and had the following capital asset transactions: Long-term gain from the sale of a coin collection (held 10 years) Long-term gain from the sale of a land investment (held 4 years) Short-term gain from the sale of a stock investment (held for $30,000 10,000 4,000

8 months) Perry's tax consequences from these gains are as follows: Choose one answer. a. (15% X $10,000) + (28% X $30,000) + (33% X $4,000) b. (15% X $30,000) + (33% X $4,000) c. (5% X $10,000) + (28% X $30,000) + (33% X $4,000) d. (15% X $40,000) + (33% X $4,000) e. None of the other answers are correct. Collectibles are taxed at a maximum of 28%, while long-term capital gains are subject to a top rate of 15%. Short-term capital gains are treated the same as ordinary income. p. 3-35 Correct Marks for this submissi Correct.

In which, if any, of the following situations may the individual not be claimed as a dependent of the taxpayer? Choose one answer. a. A former spouse who does not live with the taxpayer (divorce took place two years ago). b. A stepmother who does not live with the taxpayer. c. An unmarried daughter who lives with the taxpayer. d. A half brother who does not live with the taxpayer. e. An unrelated party who lives with the taxpayer. Correct.

A former spouse does not meet the relationship test. All of the other parties either satisfy the relationship or member of the household test. p. 3-13 Correct Which of the following unmarried taxpayers may file as a head of household in 2011? Ron provides all the support for his mother, Mary, who lives by herself in an apartment in Fort Lauderdale. Ron pays the rent and other expenses for the apartment and properly claims his mother as a dependent. Tammy provides over one-half the support for her 20-year old grandson, Dan. Dan earned $2,700 in 2011 working at a fast food restaurant and is saving his money to attend college in 2012. Dan lives in Tammy's home. Joe's wife left him late in December of 2010. No legal action was taken and Joe has not heard from her in 2011. Joe supported his 8-year-old son, who lived with him throughout 2011. Choose one answer. a. Ron only. b. Tammy only. c. Joe only. d. Ron and Joe only. e. Ron, Tammy, and Joe. Correct.

Ron may file as a head of household. His mother is not required to live in his household in order for him to qualify as a head of household. Tammy can claim Dan as a dependent because Dan is subject to the gross income requirement under the qualifying relative rules; but, made less than the exemption amount. Joe can file as a head of household under the abandoned spouse rules. pp. 3-30 and 3-31 Correct Marks for this submission: 1/1.

Which, if any, of the following is a deduction for AGI, not a deduction from AGI? Choose one answer. a. Real estate taxes paid.

b. Mortgage interest paid. c. State income taxes withheld. d. Contribution to Individual Retirement Account. e. All of the above. Correct.

All of the choices listed are deductions from AGI except Individual Retirement Account contribution (option d). pp. 3-5 and 3-6 and Exhibit 3.3 Correct

During 2011, Joan sold the following assets: business equipment for a $5,000 loss, stock investment for a $7,000 loss, and her principal residence for a $16,000 loss. Presuming adequate income, how much of these losses may Joan claim on her 2011 return? Choose one answer. a. $3,000. b. $8,000. c. $12,000. d. $28,000. e. None of the above. Correct.

The loss on the business equipment is an ordinary loss, while the loss on the stock investment is a capital loss. The ordinary loss is deducted in full. For the net capital loss of $7,000, only $3,000 can be deducted in 2011. The loss on the principal residence is personal and, therefore, cannot be deducted. pp. 3-34 to 3-36, and Example 33 Correct

Marks: 1/1

Troy and Edie are married and under 65 years of age. During 2011, they furnish more than half of the support of their 18-year old daughter, Jobeth, who lives with them.

Jobeth earns $15,000 from a part-time job, most of which she sets aside for future college expenses. Troy and Edie also provide more than half of the support of Troy's cousin who does not live with them. Edie's father, who died on January 3, 2011, at age 80, has for many years qualified as their dependent. How many personal and dependency exemptions should Troy and Edie claim? Choose one answer. a. Two. b. Three. c. Four. d. Five. e. None of the other answers are correct. Four (Troy, Edie, Jobeth, and the father). Jobeth can be claimed because as a qualifying child she is not subject to the gross income test. Troys cousin does not meet the relationship test because he is not a member of their household. It is assumed that Edies father, as was true in the past, qualified as a dependent up to the point of death. pp. 3-10, 3-11, and 3-13 Correct Correct.

Gladys, age 70, is claimed as a dependent on her sons tax return. During 2011, she had interest income of $4,900 and $800 of wages from a part-time job. Gladyss taxable income is: Choose one answer. a. $-0-. b. $3,150. c. $4,600. d. $3,900. e. None of the other answers are correct. Corrrect.

$5,700 gross income greater of $950 or ($800 earned income + $300) - $1,450 (additional standard deduction for age 65 and older) = $3,150. Example 10 Correct

Marks: 1/1

In 2011, Scott had the following transactions: $30,000 Wages earned 10,000 Lottery prize 50,000 Life insurance proceeds 4,000 Contribution to traditional IRA (Individual Retirement Account) 2,000 Medical insurance premium payments Scott's AGI for 2011 is: Choose one answer. a. $34,000. b. $36,000. c. $86,000. d. $90,000. e. None of the other answers are correct Correct.

$30,000 (wages) + $10,000 (prize) - $4,000 (traditional IRA contribution) = $36,000. The life insurance proceeds are not taxable, and the medical insurance payment is a deduction from AGI. Exhibits 3-1, 3-2, 3-3, and Examples 3 and 4 Correct

Chapter 4:

Mabel is age 65 and lives on her Social Security benefits and gifts from her son, Fred. Fred is a full-time teacher. He has written a book and receives royalties. This year Fred directed the publisher to make the royalty check payable to Mabel because she needs the money for support. Mabel must include the amount of the royalty check in her gross income. Choose one answer. a. True b. False Correct.

The royalties must be included in the gross income of Fred, the taxpayer who earned it. pp. 4-14 and 4-15 Correct

Taxable income and financial accounting income should always be the same because the goals of the Federal income tax system and financial accounting are the same. Choose one answer. a. True b. False Correct.

The principal goal of the Federal income tax system is to raise revenues in an equitable manner, whereas the goal of financial accounting is to present information to investors and potential investors to assist them in their decisions. pp. 4-4 and 4-5 Correct Marks for this submission: 1/1. ABC Corporation mails out its annual Christmas bonuses to employees on December 23rd. Ed, a cash basis taxpayer, is an employee who is on a ski trip until January 3rd of the new year, but is aware that his annual Christmas bonus arrives on the 28th of December. Ed can delay reporting the income from the bonus until the new year. Choose one answer. a. True

b. False

Correct.

The income was set aside and made available to Ed in December. p. 4-10 Correct

Marks: 1/1

An advantage to operating a business as a partnership is that the partner has no gross income from the partnership unless the partner makes a withdrawal during the year. Choose one answer. a. True b. False Correct.

The partnership profits flows through to the partners regardless of whether they are distributed to the partners. Therefore, each partner must include his or her share of the partnership profits in gross income. p. 4-16 Correct

In the case of a gift loan of less than $100,000, the imputed interest rules apply if the donee has net investment income of over $1,000. Choose one answer. a. True b. False Correct.

The imputed interest rules apply to gift loans. However, if the amount of the loan is for $100,000 or less, the imputed interest cannot exceed the borrower's net investment income for the tax year. If net investment is $1,000 or less, it is considered to be $0. pp. 4-25 and 4-26 Correct Marks for this submission: 1/1.

Paula transfers stock to her former spouse, Fred. The transfer is pursuant to a divorce agreement. Paula's cost of the stock was $50,000 and its fair market value on the date of the transfer is $75,000. Fred later sells the stock for $78,000. Fred's recognized gain from the sale of the stock is $3,000. Choose one answer. a. True b. False Correct.

Fred's basis is $50,000, the same as Paula's basis. So Fred's recognized gain is $28,000 ($78,000 - $50,000). p. 4-19 and Example 31 Correct Jeff owned stock that decreased in value by $15,000 during the year, but he did not sell the stock. He earned $40,000 salary, but received only $29,000 because $11,000 in taxes were withheld. Jeff saved $5,000 of his salary and used the remainder for personal living expenses. Jeff's economic income exceeded his gross income for tax purposes for the year. Choose one answer. a. True b. False Correct.

Jeffs economic income would be reduced by the unrealized losses on his stock. However, these unrealized losses are not recognized for tax purposes until the securities are sold. pp. 4-3 to 4-5 Correct

In the case of a below-market loan for which there is no exception to the imputed interest rules, the borrower must recognize imputed interest income.

Choose one answer. a. True b. False Correct.

The lender must recognize imputed interest income. p. 4-23 and Concept Summary 4-2 Correct Marks for this submission: 1/1

Delilah purchased an annuity for $60,000 in 2011. Under the contract, Delilah will receive $1,000 each month for the rest of her life. According to the actuarial estimates, Delilah will live to receive 200 payments and will receive a 3% return on her original investment. Choose one answer. a. If Delilah lives to collect more than 200 payments, she must amend her prior years' returns to increase her taxable portion of each payment received in the past. b. If Delilah collects $12,000 in 2011, $8,400 is treated as a recovery of capital and thus is not taxable. c. If Delilah lives to collect more than 200 payments, all amounts received after the 200th payment are excluded from her gross income. d. If Delilah dies after collecting a total of 100 payments, she has an economic loss that is deductible on her final tax return. e. None of the other answers are correct. Correct.

Options a. and b. are simply contrary to the scheme provided in the Code for the taxation of annuities. The amount excluded from income as a recovery of capital is $3,600. If Delilah dies after collecting only 100 payments, before she has recovered all of her capital, a loss can be claimed on her final return. Therefore, d. is correct. pp. 4-28 to 4-31 Correct Under the terms of a divorce agreement, Lanny was to pay his wife Joyce $2,500 per month in alimony and $500 per month in child support. For a twelve-month period, Lanny can deduct from gross income (and Joyce must include in gross income): Choose one answer. a. $0. b. $6,000. c. $30,000. d. $36,000. e. None of the above. Correct.

The $500 per month for child support is not deductible by Lanny. p. 4-23, Example 30, and Concept Summary 4-1 Correct Marks for this submission: 1/1.

Local Cable TV Company, an accrual basis taxpayer, allows its customers to pay by the year in advance ($540 per year), or two years in advance ($1,000). In June 2011, the company collected the following amounts applicable to future services: July 2011-June 2013 services (two-year contracts) July 2011-June 2012 services (one-year contracts) Total $ 34,000 35,000 $69,000

As a result of the above, Local Cable should report as gross income: Choose one answer.

a. $69,000 in 2011. b. $34,500 in 2011. c. $26,000 in 2011. d. $17,500 in 2012. e. None of the other answers are correct. One-half of the payments on the one-year contracts was earned (1/2 X $35,000 = $17,500) and one-fourth (1/4 X $34,000 = $8,500) of the payments on the two-year contracts was earned in 2011 and is included in 2011 gross income. Answer d. is incorrect because under Rev. Proc. 2004-34, the remaining earned income from the one-year contracts (1/2 X $35,000 = $17,500) and the two-year contracts (3/4 X $34,000 = $25,500) must be included in 2012 gross income. p. 4-13 and Example 19 Correct Correct.

Marks: 1/1

On June 30, 2011, Billy, a cash basis taxpayer, gave Dan the gift of a bond, $10,000 face amount, that pays $1,000 interest each December 31. When Dan collected the interest on December 31, 2011: Choose one answer. a. Billy must include all of the interest in his gross income. b. Dan must include all of the interest in his gross income. c. Billy must report $500 interest income and Dan must report $500 interest income. d. Billy must recognize $500 of income at the time of the gift. e. Dan must recognize $10,000 of income at the time of the gift. Correct.

p. 4-16 and Example 21

Correct

Seth, a calendar year taxpayer, purchased an annuity for $20,000 in 2008. The annuity was to pay him $2,000 on the first day of each year, beginning in 2008, for the remainder of his life. Seth's life expectancy at the time he purchased the annuity was 15 years. In 2010, Seth developed a deadly disease, and doctors estimated that he would live for no more than 24 months. Choose one answer. a. If Seth dies in 2011, a loss can be claimed on his final return for his unrecovered cost of the annuity. b. If Seth dies in 2011, his returns for the two previous years can be amended to allocate the entire cost of the annuity to the years in which he received payments and reported gross income. c. If Seth is still alive at the end of 2009, he is not required to recognize any gross income because of his terminal illness. d. If Seth is still alive in 2026, his recovery of capital for that year is $1,333. e. None of the other answers are correct. The annuity exclusion formula is investment/expected return = $60,000/($1,500 X 120) = $60,000/$180,000 = .333. Therefore, when Ted collects $1,500, he must recognize $1,500 X (1 .333) = $1,000 gross income. Answer b. is incorrect. Instead of amending prior returns, Ted will be allowed to deduct a loss on his final return for the cost of the annuity less the amount previously excluded as a return of capital. pp. 4-27 to 4-30 Correct Marks for this submission: 1/1. Correct.

Marks: 1/1

Darryl, a cash basis taxpayer, gave 1,000 shares of Copper Company common stock to his daughter on September 29, 2011. Copper Company is a publicly held company that has declared a $1.00 per share dividend on September 30th every year for the last 20 years. Just as Darryl had expected, Copper Company declared a $1.00 per share dividend on September 30th, payable on October 15th, to stockholders of record as of October 10th. The daughter received the $1,000 dividend on October 18, 2011. Choose one answer. a. Darryl must recognize the $1,000 dividend as his income because he knew the dividend would be paid. b. Darryl must recognize $750 of the dividend because he owned the stock for three-fourths of the year. c. Darryl must recognize the income of $1,000 because he constructively received the $1,000. d. The daughter must recognize the Correct. income because she owned the stock when the dividend was declared and she received the $1,000. e. None of the other answers are correct.

The gift of the stock is made prior to the declaration date. p. 4-15 Correct

Marks: 1/1

Our tax laws encourage taxpayers to: Choose one answer. a. Sell assets that have appreciated, but keep assets that have declined in value. b. Sell assets that have appreciated as well as those assets that have

declined in value. c. Sell assets that have declined in value, but keep those assets that have appreciated. d. Keep those assets that have declined in value as well as those that have appreciated. e. None of the above. Correct.

Because gains and losses are not recognized until realization has occurred, the taxpayer should sell depreciated assets (i.e., basis exceeds fair market value) and deduct any loss. The taxpayer should hold the appreciated assets and, thus, defer gain. p. 4-35 Correct

If a group term insurance plan discriminates in favor of certain key employees, how much must the key employee include in income? Choose one answer. a. The greater of the actual premium paid or the amount under the uniform premiums table. b. The actual premium paid by the employer c. The amount calculated from the uniform premiums table. d. The lesser of the actual premium paid or the amount under the uniform premiums table. e. None of the other answers are correct. Correct.

If a group term insurance plan discriminates in favor of certain key employees, the key employees must include inincome the greater of the amount calculated from the Uniform Premiums table or the employers cost. pp. 4-32 and 4-33 Correct

On June 30, 2011, Jill, a cash basis taxpayer, gave Tina a bond with a $25,000 face amount that pays $2,500 interest each December 31. When Tina collected the interest on December 31, 2011: Choose one answer. a. Jill must include all of the interest in her gross income. b. Tina must include all of the interest in her gross income. c. Jill must report $1,250 interest income, and Tina must report $1,250 interest income. d. Jill must recognize $1,250 of interest income at the time the gift.

e. Tina must recognize $25,000 of income at the time of the gift.

Jill and Tina each must report half of the interest income in 2011. pp. 4-8, 4-11, 4-15, and Example 21 Correct The tax concept and economic concept of income are not in agreement on which of the following: Choose one answer.

a. The fair rental value of an owner-occupied home should be included in income. b. The increase in value of assets held for the entire year should be included in income for the year.

c. The decrease in value of assets held for the entire year should reduce income for the year. d. All of the above. e. None of the above.

The realization requirement applies to the taxable income, but does not apply to economic income. pp. 4-3 and 4-4

Correct

Under the terms of a divorce agreement, Caleb is to pay his wife Juliette $600 per month. The payments are to be reduced to $450 per month when their 12 year-old child reaches age 18. During the current year, Caleb paid $7,200 under the agreement. Assuming all of the other conditions for alimony are satisfied, Caleb can deduct from gross income (and Juliette must include in gross income) as alimony: Choose one answer. a. $-0-. b. $1,800. c. $5,400. d. $7,200. e. None of the other answers are correct Correct.

$450 X 12 = $5,400. The remaining $150 per month is child support because of the contingency relative to Caleb's child. p. 4-23 and Example 36 Correct

Kathy operates a gym. She sells memberships that entitle the member to use the facilities at any time. A one-year membership costs $300 ($300/12 = $25 per month); a two-year membership costs $576 ($576/24 = $24 per month). Cash payment is required at the beginning of the membership. On July 1, 2011, Kathy sold a one-year membership and a two-year membership. Choose one answer. a. If Kathy is an accrual basis taxpayer, her gross income from the contracts for 2011 is $876 ($576 + $300). b. If Kathy is an accrual basis taxpayer, her gross income from the contracts for 2011 is $726 ($150 + $576). c. If Kathy is an accrual basis Correct.

taxpayer, her gross income from the contracts for 2011 is $294 ($150 + $144). d. If Kathy is an accrual basis taxpayer, her gross income from the contracts for 2011 is $444 ($300 + $144). e. None of the other answers are correct.

The cash basis taxpayer is required to recognize the income in the year it is received. The accrual basis taxpayer can defer the unearned income at the end of the first tax year to the second tax year. So the income is reported as follows: One-year contract Year 1 Year 2 $150 150 $300 The limited deferral is provided under Notice 2002-79. pp. 4-13 and 4-14 Correct As a general rule: Choose one answer. a. Income from property is taxed to the person who owns the property. b. Income from property is taxed to the person who benefits from the income. c. Income from services is always taxed to the person who receives the income. d. The assignee of income from property must pay the tax on the income. Correct. Two-year contract $144 432 $576

e. None of the above.

Generally, the owner of the property must pay the tax on the income. Income from services is taxed to the person who provides the services. Thus, answers b., c., and d. are incorrect. pp. 4-14 to 4-16 Correct On June 30, 2011, an accrual basis taxpayer sold a contract to provide the same service each month for 36 months, July 2011 through June 2014. The contract price received was $3,600. The taxpayer may recognize $600 gross income in 2011 and $3,000 in 2012. Choose one answer. a.True Correct. The accrual basis taxpayer cannot defer the income from the advance payments beyond the tax year following the year of receipt. p. 4-13

b.False

The accrual basis taxpayer cannot defer the income from the advance payments beyond the tax year following the year of receipt. p. 4-13 Correct Marks for this submission: 1/1.

Marks: 1/1

A cash basis taxpayer is not required to recognize income from the partnership until it is distributed to him or her. Choose one answer. a. True b. False Correct.

A partner is required to recognize his or her share of partnership income even if it is not distributed to the partner. p. 4-17

Correct Marks for this submission: 1/1.

Orlando and Katelyn were divorced. Their only marital property was a personal residence with a value of $250,000 and cost of $100,000. Under the terms of the divorce agreement, Katelyn would receive the house and Katelyn would pay Orlando $25,000 each year for 5 years, or until Orlando's death, whichever should occur first. Orlando and Katelyn lived apart when the payments were made to Orlando. The divorce agreement did not contain the word "alimony." Katelyn paid the $125,000 to Orlando over the five-year period. Then, Katelyn sold the residence for $300,000. Katelyn's recognized gain is: Choose one answer. a. $0. b. $50,000 ($300,000 - $250,000). c. $125,000 ($300,000 - $125,000 - $50,000). d. $200,000 ($300,000 - $100,000). e. None of the other answers are correct. Correct.

Katelyn's basis in the property is $100,000. The periodic payments constitute a property settlement in the divorce and have no effect on the basis of the house. The gain is therefore $200,000. Example 33 and p. 4-20 Correct Chapter 5: Amber Machinery Company purchased a building from Ted for $250,000 cash and a mortgage of $750,000. One year after the transaction, the mortgage had been reduced to $725,000 by principal payments by Amber, but it was apparent that Amber would not be able to continue to make the monthly payments on the mortgage. Ted reduced the amount owed by Amber to $600,000. This reduced the monthly payments to a level that Amber could pay. Amber must recognized $125,000 of income from the reduction in the debt by Ted. Choose one answer. a. True b. False Correct.

The reduction in the debt is being made by the person who sold the property to Amber. Therefore, Amber can reduce its basis in the building by $125,000, rather than recognizing income. pp. 5-33 and 5-34 Correct Marks for this submission: 1/1.

Marks: 1/1

If the employer pays for the employee's long-term care insurance premiums, the employee can exclude the premiums from gross income, but some of the benefits collected may be taxable. Choose one answer. a. True b. False Correct.

The exclusion for benefits received is subject to a statutory ceiling. pp. 5-15 and 5-16 Correct Marks for this submission: 1/1.

Amber received an academic scholarship that was to pay her tuition, room and board, and books. Amber is not required to recognize income from the scholarship proceeds. Choose one answer. a. True b. False Correct.

The room and board part does not qualify for exclusion treatment under 117. The amount received for tuition and books is excludible. p. 5-9 Correct Marks for this submission: 1/1.

Carla is president of the New State University and is provided a house on campus so that she will be readily accessible in the case of an emergency and to entertain faculty, students, alumni, and prospective donors. The rental value of the residence would be $48,000 a year. Carla must include the $48,000 rental value of the home in her gross income. Choose one answer. a. True b. False Correct.

The 119 exclusion for lodging applies. Example 19 and pp. 5-17 and 5-18 Correct Sam received $25,000 of salary, interest, and dividends in 2011. He also received $10,000 as worker's compensation benefits. Sam must include either 50% or 85% of the worker's compensation benefits in gross income for 2011. Choose one answer. a. True b. False Correct.

Worker's compensation benefits are excludible from gross income. p. 5-12 Correct Marks for this submission: 1/1. Alec was injured on the job and was unable to work for three months. While he was away from work, he collected $5,000 per month from a wage continuation insurance policy purchased by his employer. Alec was not required to include the premiums on the insurance policy in his gross income. Alec also received $3,000 of workers' compensation benefits. Alec is not required to include any of the above amounts in his gross income. Choose one answer. a. True

b. False

Correct.

The $5,000 of worker's compensation is excluded from Alec's gross income. Since Alec's employer purchased the wage continuation insurance policy, the $9,000 ($3,000 X 3 months) he received under this policy must be included in Alec's gross income. p. 513 Correct Marks for this submission: 1/1. Fresh Bakery often has unsold donuts at the end of the day. The bakery allows employees to take the leftovers home. The employees are not required to recognize income because the bakery does not incur any additional cost. Choose one answer. a. True b. False Correct.

The no-additional-cost exclusion applies only to services. p. 5-21 Correct Marks for this submission: 1/1. Sarah's employer pays the hospitalization insurance premiums for a policy that covers all employees and their family members. The premiums for Sarah's medical coverage can be excluded from her gross income, but the premiums for her family members must be included in her gross income. Choose one answer. a. True b. False Correct.

All of the medical insurance premiums are excluded from Sarah's gross income. p. 5-14

Correct Marks for this submission: 1/1. The earnings from funds placed in a qualified tuition program are included in the gross income of the beneficiary in the year (or years) the funds are withdrawn to pay tuition. Choose one answer. a. True b. False Correct.

The earnings from a qualified tuition program are excluded from gross income of the parents and the student when the funds are withdrawn to pay the educational expenses. pp. 5-31 and 5-32 Correct Gold Company was experiencing financial difficulties, but was not bankrupt or insolvent. Pink, Inc., the holder of a mortgage on Gold's building, agreed to accept $40,000 in full payment of the $50,000 due. Pink had sold the property to Gold for $150,000 five years ago. The National Bank, which held a mortgage on other real estate owned by Gold, reduced the principal from $100,000 to $85,000. The bank had made the loan to Gold when it purchased the real estate from Silver, Inc. As a result of the above, Gold must: Choose one answer. a. Include $25,000 in gross income. b. Reduce the basis in its assets by $25,000. c. Include $10,000 in gross income and reduce its basis in its assets by $15,000. d. Include $15,000 in gross income and Correct. reduce its basis in the building by $10,000. e. None of the above.

The $10,000 reduction in the mortgage is an adjustment to the cost of the building. The $15,000 reduction by the bank is includible in gross income. pp. 5-33 and 5-34

Correct

Johnny decided to care for his Uncle Nolan in his old age. Johnny was unaware that his uncle had securities with a fair market value of $100,000 at the time he started caring for his uncle. Uncle Nolan made no promise to Johnny regarding payment for his care. However, the cost of comparable care in a nursing home would have been $60,000. Nolan executed a will that gave the securities to Johnny. The fair market value of the securities at the time of Nolan's death was $115,000. Johnny was Nolan's favorite relative and Johnny did not need the money. Johnny's gross income from the receipt of the stock is: Choose one answer. a. $0. b. $60,000. c. $100,000. d. $115,000. e. None of the other answers are correct. Correct.

The nephew did not enter into a contract with his Uncle Nolan. The inheritance received by the nephew is not income to him. The nephew takes a basis of $115,000 in the securities received from his uncles estate. pp. 5-4 and 5-5 Correct Hazel, a solvent individual, owed $5,000 to her employer. The employer forgave the employee for her indebtedness. Hazel owned property with a cost of $50,000 subject to a mortgage of $60,000 and with a fair market value of $60,000. Hazel allowed the mortgage holder to foreclose on the property in satisfaction of the debt. Hazel's gross income from the above is: Choose one answer. a. $0. b. $5,000.

c. $10,000. d. $15,000. e. None of the above. Correct.

Absent any additional information, the employer would not be deemed to have made a gift to the employee. Even if the employer had intended that a gift be made, 102(c) prohibits exclusion treatment. The transfer of property to the creditor is treated as a sale or exchange. Hazel must recognize a $10,000 gain ($60,000 $50,000). Thus, her gross income is $15,000 ($5,000 + $10,000). pp. 5-33 and 5-34 Correct Todd's personal residence was destroyed by fire. He collected insurance proceeds that equaled the cost of the house. Todd had recently paid $900 for fire insurance premiums, but as a result of the fire, his policy was cancelled. He received $500 from the cancellation of his fire insurance policy. Choose one answer. a. Todd must include $500 in gross income from the cancellation of the policy. b. Todd can recognize a $400 ordinary loss on the cancellation of the policy. c. Todd is not required to recognize any gross income under the claim of right doctrine. d. Todd is not required to recognize any Correct. gross income under the tax benefit rule. e. None of the above.

The $500 rebate is not included in Todds gross income because when he paid the premium of $900, a deduction was not permitted (i.e., personal use asset). pp. 5-32 and 5-33 Correct

The Amber Fruit Cake Company gives all of its employees a 5 pound fruit cake each Christmas. The value of the fruit cake: Choose one answer. a. Must be included in the employee's gross income. b. Must be included in the employee's gross income unless the employee does not like fruit cake and gives it to someone else. c. >May be excluded as a "no-additionalcost" fringe benefit. d. May be excluded as a de minimis fringe Correct. benefit. e. None of the above.

The House Ways and Means Committee specifically cited holiday gifts with a low market value as de minimis fringes. p. 5-23 Correct Marks for th Jena is a full-time student at State University and is claimed by her parents as a dependent. Her only source of income is a $6,000 scholarship ($400 for books, $3,000 tuition, $200 school supplies, and $2,400 room and board). Jena's gross income for the year is: Choose one answer. a. $0. b. $400. c. $2,400. d. $3,000. e. None of the above. Correct.

The portion included in gross income is $2,400 ($2,400 for room and board). Only the books ($400), school supplies($200) and tuition ($3,000) qualify for exclusion. p. 5-9 Correct

Isabel's income from her investments for the current year was as follows: Gain on the sale of Rosebud County school bonds Interest on Rosebud County school bonds Interest received during the year on U. S. Government bonds Total Isabel's gross income from the above is: Choose one answer. a. $3,000. b. $9,500. c. $15,000. d. $18,000. e. None of the other answers are correct. Correct. $ 3,000 8,500 6,500

$18,000

While the $8,500 interest on the Rosebud County school bonds is tax-exempt, the $3,000 gain on the sale of these bonds is included in Isabel's gross income. The interest received on the U.S. Government bonds is included in Isabel's gross income. Thus, Isabel's gross income is $9,500 ($3,000 + $6,500). pp. 5-28 and 5-29 Correct The taxpayer is a Ph.D. student in accounting at State University. The student is paid $1,000 per month for teaching two classes. Choose one answer. a. The $1,000 is excludible if all Ph.D. students are required to teach. b. The $1,000 is excludible because it is a gift.

c. The $1,000 is considered a scholarship and, therefore, is excluded. d. Only $500 is included in income. e. The $1,000 is all taxable. Correct.

The $1,000 per month represents compensation for services rendered and must be included in the student's gross income. pp. 5-10 and 5-11 Correct The employees of the Daily Newspaper are given a free newspaper each day. The annual subscription fee for a newspaper is $270. Choose one answer. a. The employees can exclude the value of the newspaper from their gross income as a no-additional cost fringe benefit. b. The employees can exclude the value of the newspaper from their gross income because the employees were not given an option of receiving cash. c. An employee is required to include the value of the newspaper in gross income only if the employee would otherwise have bought a newspaper. d. The employees are not required to include the value of the newspaper in their gross income because the employer would have otherwise thrown away the newspapers. e. The employees are required to include Correct. the value of the newspaper in their gross income.

Option a. is incorrect because the newspaper is property and the no-additional-cost exclusion applies only to services. Based on the facts provided, options b., c., and d. do

not fit within any of the 132 exclusion provisions either. Thus, the employees must each include the value of the newspapers in their gross income. pp. 5-20 to 5-25 Correct Marks for this submi Albert has a terminal illness and will require almost constant nursing care for the remaining two years of his estimated life, according to his doctor. Albert has $25,000 in unpaid medical expenses and a life insurance policy with a face amount of $100,000. Albert has paid $10,000 of premiums on the policy. The insurance company has offered to pay him $75,000 to cancel the policy, although its cash surrender value is only $60,000. Assume Albert takes the $75,000, cancels the policy, and pays the $25,000 in medical expenses. Choose one answer. a. Albert must recognize $65,000 ($75,000 - $10,000) of gross income. b. Albert must recognize $40,000 ($65,000 - $25,000) of gross income. c. Albert is not required to recognize gross income because of his terminal illness. d. Albert must recognize $10,000 of gross income, but he has $25,000 of deductible medical expenses. e. None of the above. Correct.

Albert is chronically ill and, therefore, can qualify for the accelerated death benefit exclusion for his life insurance policy. pp. 5-8 and 5-9 Correct Marks for this submission: 1/1. Green Company purchased a $1 million life insurance policy on the company's chief executive officer, Howard. After the company had paid $300,000 in premiums, Howard died and the company collected the $1 million face amount of the policy. The company also purchased group-term life insurance on all its employees. Howard's widow, Agnes, received the $150,000 proceeds from the group-term life insurance policy. Choose one answer.

a. Green Company and Agnes can exclude the life insurance proceeds of $1,000,000 and $150,000 respectively from gross income. b. Agnes can exclude the life insurance proceeds of $150,000, but Green Company must include $700,000 ($1,000,000 - $300,000) in gross income. c. Green Company can exclude the life insurance proceeds of $1,000,000 from gross income, but Agnes must include the $150,000 in gross income. d. Green Company must include $700,000 ($1,000,000 - $300,000) in gross income and Agnes must include $150,000 in gross income. e. None of the above.

Correct.

All of the proceeds qualify for the life insurance exclusion because the payments were received as a result of the death of the insured. pp. 5-7 and 5-8 Correct Marks for this submission: 1/1. Ted was diagnosed with a terminal illness. His physician estimated that Ted would live no more than 18 months. Ted cashed in his life insurance policy to pay some medical bills, after he received the doctor's diagnosis. Ted has paid $12,000 in premiums and he collected $25,000, the cash surrender value of the policy. George enjoys excellent health, but he cashed in his life insurance policy to purchase a new home. He had paid premiums of $12,000 and collected $25,000 from the insurance company. Choose one answer. a. Neither Ted nor George is required to recognize gross income. b. Both Ted and George must recognize $13,000 ($25,000 $12,000) gross income. c. Ted must recognize $13,000

($25,000 - $12,000) of gross income, but George does not recognize any gross income. d. George must recognize $13,000 Correct. ($25,000 - $12,000) of gross income, but Ted does not recognize any gross income. e. None of the above.

The redemption of the policy by Ted qualifies as an accelerated death benefit. Thus, the realized gain is excluded from his gross income. p. 5-8 and 5-9 Correct In the case of income from discharge of indebtedness: I. If the debt is secured by property sold to the debtor by the creditor, the debtor reduces the basis in the property instead of recognizing income. II. If the debtor is a business, the discharge usually is considered a gift. III. If a corporation's debt is forgiven by a shareholder in the corporation, the corporation increases paid-in capital and is not required to recognize income. Choose one answer. a. I, II and III are true. b. I and II are true, III is false. c. I and III are true, II is false. d. II and III are true, I is false. e. I, II, and III are false. Correct.

II is false. Business persons do not ordinarily forgive indebtedness out of "unattached generosity." pp. 5-34 Correct Marks for this submission: 1/1.

Early in the year, Mike was in an automobile accident during the course of his employment. As a result of the injuries he sustained, he received the following payments during the year: Regular salary under the company's sick pay plan Reimbursement of medical expenses from the company's group medical insurance plan Worker's compensation benefit $3,000 6,000

4,000

What is the amount that Mike must include in gross income for the current year? Choose one answer. a. $0. b. $3,000. c. $7,000. d. $9,000. e. None of the above. Correct.

The $3,000 Mike received under the companys sick pay plan must be included in his gross income. pp. 5-11 to 5-14 Correct Cardinal Corporation paid $150,000 for a $1 million life insurance policy on a key employee. When the employee died, the corporation collected $1 million on the life insurance policy. The corporation is notrequired to recognize any gross income from collecting the life insurance proceeds. Choose one answer. a. True b. False Correct.

The life insurance proceeds of $1 million are excludible under 101(a) regardless of the employer-employee relationship. Example 2 and p. 5-6

Correct George, an unmarried taxpayer, received the following amounts: Interest on savings accounts Original issue discount on a certificate of deposit purchased on July 1, 2010, and matured on June 30, 2011 Dividends on USG common stock Dividends on life insurance policies Interest on City of Radford school bonds $2,400 800 200 300 600

What amount should George report as gross income, after exclusions if any, from dividends and interest for 2011? Choose one answer. a. $4,300. b. $3,900. c. $3,400. d. $3,300. e. $3,000. Correct.

The dividend on the life insurance policy of $300 is a recovery of premiums, and thus is not taxable. The interest on the Radford school bonds of $600 is tax-exempt. Since the life of the certificate is only 1 year, the $800 is included in gross income in 2011. Thus, the gross income is $2,400 + $800 + $200 = $3,400. pp. 5-29, 5-30, and Chapter 4 Correct Marks for this submission: 1/1.

Chapter 6:

Chapter 6 Exam

Review of attempt 1
Started on Completed on Time taken Marks Grade Tuesday, 28 February 2012, 08:09 AM Tuesday, 13 March 2012, 12:15 PM 14 days 3 hours 21/25 42 out of a maximum of 50 (84%)

Question1
Marks: 1/1

A taxpayer pays his dependent son's real estate taxes. The taxpayer may deduct the taxes, but the son cannot. Choose one answer. a. True b. False The expenses are neither for the taxpayers benefit nor arise from the taxpayers obligation. Thus, neither person can deduct them. p. 6-23 Correct Marks for this submission: 1/1.
History of Responses: # 1 Grade 2 Close&Grade Action Response False False Time 08:09:19 on 28/02/12 08:09:19 on 28/02/12 1 1 Raw score 1 1 Grade

Question2
Marks: 1/1

A taxpayer in the 25% tax bracket may receive a different tax benefit for a $100 expenditure that is classified as a deduction from AGI than he or she will receive for a $100 expenditure that is classified as a deduction for AGI. Choose one answer.

a. True b. False The value of the tax benefit for the deduction from AGI may be less than that for the deduction for AGI. The value of the deduction for AGI for a taxpayer in the 25% bracket for a $100 expenditure is $25 ($100 X 25%). If the taxpayer takes the standard deduction rather than itemizing deductions, then the $100 expenditure that is classified as a deduction from AGI has no tax benefit. Likewise, if the expenditure that is classified as a deduction from AGI is subject to a floor limitation (e.g., 7.5% floor on medical expenses), then the value of the $100 expenditure will be less than $25. pp. 62, 6-3, and Example 1 Correct Marks for this submission: 1/1.
History of Responses: # 1 Grade 2 Close&Grade Action Response True True Time 08:10:50 on 28/02/12 08:10:50 on 28/02/12 1 1 Raw score 1 1 Grade

Question3
Marks: 1/1

If a vacation home is primarily rental, a deduction for all of the Expenses of the rental is allowed. Choose one answer. a. True b. False The personal use portion must be prorated. p. 6-20 and Concept Summary 6-2 Correct Marks for this submission: 1/1.
History of Responses: # 1 Grade 2 Close&Grade Action Response False False Time 08:21:31 on 28/02/12 08:21:31 on 28/02/12 1 1 Raw score 1 1 Grade

Question4
Marks: 0/1

Landscaping expenditures on a new rental property are deductible. Choose one answer.

a. True b. False They must be capitalized and depreciated. p. 6-25 Correct Marks for this submission: 1/1. With previous penalties this gives 0/1.
History of Responses: # 1 Grade 2 Grade 3 Close&Grade Action Response True False False Time 10:04:28 on 13/03/12 10:04:35 on 13/03/12 10:04:35 on 13/03/12 0 1 1 Raw score 0 0 0 Grade

Question5
Marks: 1/1

Ralph, a shareholder-employee of Warbler, Inc., receives a $250,000 salary. The IRS classifies $80,000 of this amount as unreasonable compensation. The effect of this reclassification is to increase Ralph's gross income by $80,000. Choose one answer. a. True b. False While Ralphs salary income decreases by $80,000, his dividend income increases by $80,000. Thus, his gross income does not change. Example 6 Correct Marks for this submission: 1/1.
History of Responses: # 1 Grade 2 Close&Grade Action Response False False Time 08:32:03 on 28/02/12 08:32:03 on 28/02/12 1 1 Raw score 1 1 Grade

Question6
Marks: 1/1

If an item such as property taxes exceeds the income from a hobby, the excess amount of this item over the hobby income can be deducted. Choose one answer. a. True b. False

The excess in this case qualifies as an itemized deduction. p. 6-17 Correct Marks for this submission: 1/1.
History of Responses: # 1 Grade 2 Close&Grade Action Response True True Time 08:34:10 on 28/02/12 08:34:10 on 28/02/12 1 1 Raw score 1 1 Grade

Question7
Marks: 1/1

A hobby activity can result in all of the hobby income being included in AGI and no deductions being allowed. Choose one answer. a. True b. False The 2%-of-AGI limitation can wipe out any deductible amount. If the taxpayer does not itemize, no deduction will be permitted. p. 6-17 and Example 22 Correct Marks for this submission: 1/1.
History of Responses: # 1 Grade 2 Close&Grade Action Response True True Time 08:36:22 on 28/02/12 08:36:22 on 28/02/12 1 1 Raw score 1 1 Grade

Question8
Marks: 1/1

Ordinary and necessary business expenses, other than cost of goods sold, of an illegal drug trafficking business do not reduce taxable income. Choose one answer. a. True b. False The amount of cost of goods sold reduces taxable income. Cost of goods sold is part of the calculation of gross income rather than being treated as an expense. Thus, the 280E prohibition on deductions for drug dealers does not affect cost of goods sold. pp. 6-13 and 6-14

Correct Marks for this submission: 1/1.


History of Responses: # 1 Grade 2 Close&Grade Action Response True True Time 08:38:51 on 28/02/12 08:38:51 on 28/02/12 1 1 Raw score 1 1 Grade

Question9
Marks: 1/1

Beulah's personal residence has an adjusted basis of $190,000 and a fair market value of $175,000. Beulah converts the property to rental use on December 1, 2011. The vacation home rules that limit the amount of the deduction to the rental income will apply and the adjusted basis for depreciation is $175,000. Choose one answer. a. True b. False The adjusted basis for depreciation is $175,000, the lower of Beulahs adjusted basis of $190,000 or the fair market value of $175,000 on the date of the conversion. However, additional data is necessary to determine whether the vacation home gross income ceiling on deductions rule applies. If the qualified rental period exception applies, the gross income limit does not apply. If the qualified rental period exception does not apply, then the gross income limit does apply. pp. 6-22 and 6-23 Correct Marks for this submission: 1/1.
History of Responses: # 1 Grade 2 Close&Grade Action Response False False Time 09:17:02 on 28/02/12 09:17:02 on 28/02/12 1 1 Raw score 1 1 Grade

Question10
Marks: 1/1

Bonnie sells her personal use SUV for $22,000 (adjusted basis of $38,000). Her realized loss of $16,000 ($22,000 - $38,000) can be recognized for income tax purposes. Choose one answer. a. True

b. False A loss on the sale of a personal use asset cannot be deducted. p. 6-7 Correct Marks for this submission: 1/1.
History of Responses: # 1 Grade 2 Close&Grade Action Response False False Time 07:58:08 on 4/03/12 07:58:08 on 4/03/12 1 1 Raw score 1 1 Grade

Question11
Marks: 0/1

For a president of a publicly held corporation, which of the following are not subject to the $1 million limit on executive compensation? Choose one answer. a. Salary. b. Contribution to pension plan. c. Group term life insurance of $50,000. d. Only b. and c. are not subject to the limit. e. a., b., and c., are not subject to the limit. The salary is subject to the $1 million limit. pp. 6-14 and 6-15 Correct Marks for this submission: 1/1. With previous penalties this gives 0/1.
History of Responses: # Action Response Group term life insurance of $50,000. Only b. and c. are not subject to the limit. Time 12:12:46 on 13/03/12 12:13:51 on 13/03/12 12:15:01 on 13/03/12 Raw score 0 1 1 Grade 0 0 0

2 Grade 3 Grade

4 Close&Grade Only b. and c. are not subject to the limit.

Question12
Marks: 1/1

Judy, a calendar year cash basis taxpayer, owns and operates several TV rental outlets in N.C., and wants to expand into the TV rental business in South Carolina and Georgia. During the current year, she spends $30,000 to investigate TV rental businesses in South Carolina and $15,000 to investigate TV rental businesses in Georgia. She acquires the South Carolina operations, but not the outlets in Georgia. As to these expenses, Judy should:

Choose one answer. a. Amortize $30,000 over 60 months and capitalize $15,000. b. Expense $45,000 for the current year.

c. Expense $15,000 for the current year and capitalize $30,00 d. Capitalize $45,000. e. None of the other answers are correct. Judy currently owns and operates several TV rental outlets. Thus, all $45,000 of the investigation expenses can be currently deducted. It does not matter whether she acquires the businesses or not since she is already in this line of business. Example 18 and p. 6-15 Correct Marks for this submission: 1/1.
History of Responses: # Action Response Expense $45,000 for the current year. Expense $45,000 for the current year. Time 08:43:42 on 4/03/12 08:43:42 on 4/03/12 Raw score 1 1 Grade 1 1

1 Grade 2 Close&Grade

Question13
Marks: 1/1

Caroline, a calendar year cash basis taxpayer, owns and operates several TV rental outlets in Florida, and wants to expand to other states. During the current year, she spends $24,000 to investigate TV rental stores in South Carolina and $9,000 to investigate TV rental stores in Georgia. She acquires the South Carolina operations, but not the outlets in Georgia. As to these expenses, Caroline should: Choose one answer. a. Capitalize $24,000 and not deduct $9,000. b. Expense $33,000 for 2009. c. Expense $9,000 for 2009 and capitalize $24,000 d. Capitalize $33,000. e. None of the other answers are correct. Since Caroline owns and operates TV rental outlets, all of the investigation expenses can be deducted. p. 6-15 and Example 18 Correct

Marks for this submission: 1/1.


History of Responses: # Action Response Expense $33,000 for 2009. Expense $33,000 for 2009. Time 08:45:29 on 4/03/12 08:45:29 on 4/03/12 1 1 Raw score Grade 1 1

1 Grade 2 Close&Grade

Question14
Marks: 1/1

During the first year of operations, Al's fast food restaurant had cash sales of $200,000. Other relevant information is as follows: Purchases Salaries Other expenses Ending inventory Choose one answer. $70,000 90,000 10,000 20,000

a. If Al's business uses the accrual method, the net profit is $50,000. b. If Al's business uses the accrual method, the net profit is $30,000. c. Al's business is not eligible to use the cash method. d. Only a. and c. e. Only b. and c. The net profit of Al's business is calculated as follows: Sales Less: Expenses Salaries Other expenses Purchases net of ending inventory ($70,000 - $20,000) 50,000 (150,000) Net profit $ 50,000 $90,000 10,000 $200,000

Since the business has inventory, the accrual method must be used to calculate cost of goods sold. pp. 6-9 and 6-10 Correct Marks for this submission: 1/1.
History of Responses: # 1 Grade 2 Close&Grade Action Response Only a. and c. Only a. and c. Time 08:49:03 on 4/03/12 08:49:03 on 4/03/12 1 1 Raw score 1 1 Grade

Question15
Marks: 1/1

Saul is single, under age 65, and has gross income of $50,000. His bona fide deductible expenses are as follows: Alimony Charitable contributions Contribution to a traditional IRA Expenses paid on rental property Interest and taxes on personal residence State income tax What is Saul's AGI? Choose one answer. a. $19,800. b. $30,000. c. $35,000. d. $38,000. e. $42,000. Sauls AGI is calculated as follows: Gross Income Deductions for AGI: Alimony IRA Expenses on rental property AGI pp. 6-3 to 6-5 $50,000 $12,000 3,000 5,000 $12,000 2,000 3,000 5,000 7,000 1,200

(20,000) $30,000

Correct Marks for this submission: 1/1.


History of Responses: # 1 Grade 2 Close&Grade Action Response $30,000. $30,000. Time 08:57:05 on 4/03/12 08:57:05 on 4/03/12 1 1 Raw score 1 1 Grade

Question16
Marks: 1/1

Tiger, Inc. is an accrual basis taxpayer. For books, Tiger uses the aging approach to calculate the reserve for bad debts. During the current year, the following occur associated with bad debts. Credit sales Collections on credit sales Amount added to the reserve Beginning balance in the reserve $400,000 360,000 35,000 -0-

Identifiable bad debts during the current year 18,000 The amount of the tax deduction for bad debt expense for Tiger for the current year is: Choose one answer. a. $18,000. b. $35,000. c. $40,000. d. $53,000. e. None of the other answers are correct. Only the specific charge-off method can be used for taxes. Reserves for estimated expenses are not allowed for tax purposes because the economic performance test cannot be satisfied. p. 6-11 Correct Marks for this submission: 1/1.
History of Responses:

# 2 Grade

Action

Response $18,000. $18,000.

Time 10:08:52 on 13/03/12 10:08:52 on 13/03/12 1 1

Raw score 1 1

Grade

3 Close&Grade

Question17
Marks: 1/1

Which of the following are deductions for AGI? Choose one answer. a. Alimony payments. b. Property taxes on a personal residence. c. Charitable contributions. d. Fines and penalties incurred in a trade or business. e. None of the above. pp. 6-3 to 6-5 Correct Marks for this submission: 1/1.
History of Responses: # 1 Grade 2 Close&Grade Action Response Alimony payments. Alimony payments. Time 09:04:32 on 4/03/12 09:04:32 on 4/03/12 1 1 Raw score 1 1 Grade

Question18
Marks: 1/1

Student loan interest should be treated as: Choose one answer. a. Not deductible. b. Deductible for AGI.

c. A deduction from AGI not subject to the 2 percent of AGI floo d. A deduction from AGI subject to the 2 percent of AGI floor. e. None of the other answers are correct. Student loan interest is classified as a deduction for AGI. p. 6-4 Correct

Marks for this submission: 1/1.


History of Responses: # 1 Grade 2 Close&Grade Action Response Deductible for AGI. Deductible for AGI. Time 09:48:20 on 4/03/12 09:48:20 on 4/03/12 1 1 Raw score 1 1 Grade

Question19
Marks: 0/1

Which of the following is a deduction for AGI? Choose one answer. a. Casualty losses. b. Charitable contributions. c. Real estate tax on personal residence. d. Qualified tuition expense. e. None of the other answers are correct. Qualified tuition expense is a deduction for AGI. The other items are deductions from AGI. pp. 6-3 to 6-5 and Figure 6-1 Correct Marks for this submission: 1/1. With previous penalties this gives 0/1.
History of Responses: # Action Response None of the other answers are correct. Qualified tuition expense. Qualified tuition expense. Time 10:13:46 on 13/03/12 10:13:54 on 13/03/12 10:13:54 on 13/03/12 Raw score 0 1 1 Grade 0 0 0

1 Grade 2 Grade 3 Close&Grade

Question20
Marks: 1/1

Gerald owns an illegal casino. Which of the following expenses incurred in connection with this activity are deductible currently? Choose one answer. a. Rent. b. Illegal kickbacks to officials to stay in business. c. Fines.

d. Purchase of a building. e. None of the above. All usual expenses of operating an illegal business are deductible; the illegal expenses are not. The building must be capitalized and depreciated. pp. 6-13 and 6-25 Correct Marks for this submission: 1/1.
History of Responses: # 1 Grade 2 Close&Grade Action Response Rent. Rent. Time 10:07:08 on 4/03/12 10:07:08 on 4/03/12 1 1 Raw score 1 1 Grade

Question21
Marks: 1/1

Sierra pursued a hobby of knitting socks in her spare time. During the year she sold the socks for $ 4,500. She incurred expenses as follows: Supplies Interest on loan to get business started Advertising $2,200 350 475

Assuming that the activity is deemed a hobby and that she cannot itemize this year, how should she report these items on her tax return? Choose one answer. a. Include $4,500 in income and deduct $3,025 for

b. Ignore both income and expenses since hobby lo b. Ignore both income and expenses since hobby losses are disallowed.

c. Include $4,500 in income and deduct nothing for AGI since ho expenses must be itemized.

d. Include $4,500 in income and deduct interest of $350 for AGI e. None of the other answers are correct. Hobby income is included in AGI. However, hobby related expenses are deducted from AGI as itemized deductions. Example 22 Correct Marks for this submission: 1/1.

History of Responses: # Action Response Include $4,500 in income and deduct nothing for AGI since hobby expenses must be itemized. Time 10:50:48 on 4/03/12 10:50:48 on 4/03/12 Raw score Grade 1 1 1 1

1 Grade

2 Close&Grade Include $4,500 in income and deduct nothing for AGI since hobby expenses must be itemized.

Question22
Marks: 1/1

If a vacation home is determined to be a personal/rental use residence, which of the following statements is false? Choose one answer. a. All income is included in AGI. b. All rental related expenses are deductible from AGI. c. Expenses must be allocated between rental and personal use. d. Some expenses are deductible from AGI. e. None of the above. Any deductible expenses related to the rental use are deductions for AGI. p. 6-21 Correct Marks for this submission: 1/1.
History of Responses: # Action Response All rental related expenses are deductible from AGI. Time 10:27:00 on 4/03/12 10:27:00 on 4/03/12 Raw score Grade 1 1 1 1

1 Grade

2 Close&Grade All rental related expenses are deductible from AGI.

Question23
Marks: 1/1

Lindsay incurred the following expenses for her dependent son during the current year: Payment of principal on son's mortgage loan Interest on son's mortgage loan Payment of son's medical expenses Payment of son's property taxes $7,000 3,000 2,500 1,000

How much may Lindsay consider in computing her itemized deductions? Choose one answer. a. $1,000. b. $2,500. c. $3,500. d. $5,500. e. $12,500. Only the medical expenses (subject to the 7.5% of AGI floor) are deductible. The other items are not incurred for the taxpayer's benefit or as a result of the taxpayer's obligation. p. 6-25 Correct Marks for this submission: 1/1.
History of Responses: # 1 Grade 2 Close&Grade Action Response $2,500. $2,500. Time 08:27:58 on 13/03/12 08:27:58 on 13/03/12 1 1 Raw score 1 1 Grade

Question24
Marks: 1/1

Which of the following is not deductible? Choose one answer. a. Moving expenses. b. Tax return preparation fees. c. Expenses incurred for the production of income. d. Hobby expenses in excess of hobby income. e. None of the above. Moving expenses ( 217); tax return preparation fee ( 212); and expenses incurred for the production of income ( 212) are deductible. The hobby expenses in excess of the hobby income cannot be deducted. p. 6-16 and Example 22 Correct Marks for this submission: 1/1.
History of Responses:

Action

Response Hobby expenses in excess of hobby income.

Time 08:26:17 on 13/03/12

Raw score Grade 1 1 1

1 Grade

2 Close&Grade Hobby expenses in excess of hobby income.

08:26:17 on 13/03/12 1

Question25
Marks: 0/1

A residence rented for 160 days and used for personal purposes for more than 17 days is: Choose one answer. a. Primarily rental use. b. Primarily personal use. c. Personal/rental use. d. Primarily production of income use. e. None of the other answers are correct. A residence rented for more than 15 days and used personally for more than the greater of 14 days or 10% of rental days is considered to be personal/rental use property. pp 622 and 6-23. Correct Marks for this submission: 1/1. With previous penalties this gives 0/1.

Chapter 8 Marks: 1/1 If an automobile is placed in service in 2009, the limitation for cost recovery in 2011 will be based on the cost recovery limits for the year 2011. Choose one answer. a. True b. False Correct.

The limits will be based on the limits for automobiles placed in service in 2009, the year placed in service. REF: p. 8-16 Correct Marks for this submission: 1/1. 2 Marks: 1/1 Under the alternative depreciation system (ADS), only the half-year convention is applicable for personalty. Choose one answer. a. True Correct. b. False The mid-quarter convention also is applicable for personalty. p. 8-20 Correct Marks for this submission: 1/1.

3 Marks: 1/1 The statutory dollar cost recovery limits under 280F on passenger automobiles are imposed after any reduction for personal use is considered. Choose one answer. a. True Correct. b. False These limits are imposed before any reduction for personal use. p. 8-15-16 Correct Marks for this submission: 1/1 5 Marks: 1/1
Personal use property that is subject to wear and tear is eligible for cost recovery.

Choose one answer. a. True Correct. b. False Personal use property is not eligible for cost recovery. p. 8-3 Correct

7 Marks: 1/1 Under the MACRS straight-line election for personalty, the mid-quarter convention is not applicable. Choose one answer. a. True Correct. b. False The mid-quarter convention is applicable under the MACRS straight-line election for personalty. p. 8-11 Correct

8 Marks: 1/1
The key date for calculating cost recovery is the date the asset is purchased.

Choose one answer. a. True Correct. b. False The key date for calculating cost recovery is the date the asset is placed in service. p. 83 Correct

9 Marks: 1/1 If a new car that is used predominantly in business is placed in service in 2011, the statutory dollar cost recovery limit under 280F will depend on whether the taxpayer takes MARCS or straight-line depreciation. Choose one answer. a. True Correct. b. False The statutory cost recovery limits apply whether MACRS or straight-line apply. pp. 8-16 Correct 10

Marks: 1/1 The 179 deduction can exceed $500,000 in 2011 if the taxpayer had a 179 amount which exceeded the taxable income limitation in the prior year. Choose one answer. a. True b. False Correct.

The 179 amount eligible for expensing in a carryforward year is limited to the lesser of (1) the statutory dollar amount ($500,000 in 2011) reduced by the cost of 179 property placed in service in excess of $2 million in the carryforward year or (2) the business income limitation in the carryforward year. REF: p. 8-13 Correct

13 Marks: 1/1 James purchased a new business asset (three-year property) on July 23, 2011, at a cost of $50,000. He did not elect to expense any of the asset under 179, nor did he elect straight-line cost recovery. Determine the cost recovery deduction for 2011. Choose one answer. a. $8,333. b. $33.333 c. $16,665 d. $41,665. e. None of the other answers are correct. Additional first-year depreciation is $25,000 ($50,000 50%). Regular MACRS is $8,333 ($25,000 .3333). So total cost recovery is $33,333 ($25,000 + $8,333). Correct Marks for this submission: 1/1. Correct.

14 Marks: 1/1 Which of the following statements is true regarding depletion? Choose one answer. a. Cost depletion must be used for tax purposes. b. Percentage depletion must be used for tax purposes c. A taxpayer will select the greater of cost or percentage depletion for tax purposes. d. A taxpayer will select the lower of cost or percentage depletion for tax purposes. e. None of the above. The taxpayer will select the greater of cost or percentage depletion for tax purposes. pp. 8-25 Correct Correct.

15 Marks: 1/1
Pat purchased a used five-year class asset on March 15, 2011, for $60,000. He did not elect 179 expensing. Determine the cost recovery deduction for 2011 for earnings and profits purposes.

Choose one answer. a. $2,000. b. $3,000. c. $6,000. d. $12,000. e. None of the other answers are correct. .10 X $60,000 = $6,000. p. 8-20 and Table 8-5 Correct Marks for this submission: 1/1. Correct.

17 Marks: 1/1

For tax purposes, a deduction is allowed for the consumption of the cost of timber through: Choose one answer. a. Cost recovery. b. Amortization. c. Depletion. d. All of the other answers are acceptable. e. None of the other answers are correct. Timber is subject to depletion. p. 8-3 Correct Correct.

18 Marks: 1/1 The only asset Bill purchased during 2011 was a new seven-year class asset. The asset, which was listed property, was acquired on June 17 at a cost of $50,000. The asset was used 40% for business, 30% for the production of income, and the rest of the time for personal use. Bill always elects to expense the maximum amount under 179 whenever it is applicable. The net income from the business before the 179 deduction is $100,000. Determine Bills maximum deduction with respect to the property for 2011. Choose one answer. a. $1,428. b. $2,499. c. $26,749. d. $33,375. e. None of the other answers are correct. The listed property does not pass the predominantly business usage test. Therefore, neither 179 expensing nor additional first-year depreciation can be taken. In addition, only straight-line cost recovery can be used. Maximum deduction ($50,000 X .0714 X 70%) $2,499 REF: p. 8-13 to 8-15 | Table 8-3 Correct Correct.

19 Marks: 1/1 On June 1 of the current year, Roman converted a building to rental property. At the time of the conversion, the building was worth $140,000. Five years ago Roman purchased the building for $120,000. The building is still encumbered by a $50,000 mortgage. What is the basis of the building for cost recovery? Choose one answer. a. $70,000. b. $90,000. c. $120,000. d. $140,000. e. None of the above. The basis is $120,000, the lower of the adjusted basis ($120,000) or fair market value ($140,000) at the date of conversion. p. 8-4 and Example 3 Correct Marks for this submission: 1/1. Correct.

Marks: 1/1
In 2010, Gail had a 179 deduction carryover of $15,000. In 2008, she elected 179 for an asset acquired at a cost of $115,000. Gails 179 business income limitation for 2010 is $127,000. Determine Gails 179 deduction for 2011.

Choose one answer. a. $15,000. b. $115,000. c. $127,000. d. $130,000. e. None of the other answers are correct.
$130,000 ($15,000 + $115,000), limited to $127,000.

Correct.

REF: p. 8-13 | p. 8-14 Correct Marks for this submission: 1/1.

22 Marks: 1/1

On January 15, 2011, Penelope purchased the rights to a mineral interest for $12,000,000. At that time, it was estimated that the recoverable units would be 2,500,000. During the year, 300,000 units were mined and 175,000 units were sold for $3,000,000. Penelope incurred expenses during 2011 of $1,000,000. The percentage depletion rate is 22 percent. Determine Penelope's depletion deduction for 2011. Choose one answer. a. $1,000,000. b. $840,000. c. $660,000. d. $450,000. e. None of the other answers are correct. Cost Basis $12,000,000/2,500,000 = $4.80 per unit; Cost Depletion 175,000 units sold X $4.80 = $840,000 Percentage Depletion 22% X $3,000,000 = $660,000 Percentage limit ($3,000,000 - $1,000,000) X 50% = $1,000,000 Thus, the depletion deduction is $840,000. pp. 8-22 to 8-25 Correct Correct.

24 Marks: 1/1
On June 1, 2011, James places in service a new automobile that cost $40,000. The car is used 60% for business and 40% for personal use. (Assume this percentage is maintained for the life of the car.) Determine the cost recovery deduction for 2011.

Choose one answer. a. $4,800. b. $1,836. c. $8,000. d. $11,060. e. None of the other answers are correct. $40,000 X .20 = $8,000 (limited to $11,060*). Correct.

$8,000 X 60% = $4,800. *These depreciation limits are indexed annually. REF: Example 24 | Table 8-1 Correct

25 Marks: 1/1 The only asset Harry purchased during 2011 was a new seven-year class asset. The asset, which was listed property, was acquired on June 17 at a cost of $100,000. The asset was used 40% for business, 30% for the production of income, and the rest of the time for personal use. Harry always elects to expense the maximum amount under 179 whenever it is applicable. The net income from the business before the 179 deduction is $100,000. Determine Harrys maximum deduction with respect to the property for 2011. Choose one answer. a. $1,999. b. $4,998. c. $48,574. d. $75,176. e. None of the above. The listed property does not pass the predominantly business usage test. Therefore, 179 expensing cannot be taken. In addition, only straight-line cost recovery can be used. Maximum deduction ($100,000 X .0714 X 70%) =$4,998 pp. 8-4 to 8-15 and Table 8-3 Correct Marks for this submission: 1/1. Correct.

4 Marks: 1/1
Under the alternative depreciation system (ADS), the salvage value of the asset is ignored.

Choose one answer.

a. True b. False

Correct.

p. 8-20 Correct Marks for this submission: 1/1. 6 Marks: 1/1 Taxpayers may elect to use the straight-line method under MACRS for personal property. Choose one answer. a. True b. False p. 8-11 Correct Marks for this submission: 1/1. Patents are amortized over what period of time? Choose one answer. a. 60 months or more. b. 15 years. c. 20 years. d. 30 years. e. None of the above. Patents are 197 intangibles and thus are amortized over a 15-year period. Correct
Chapter 10

Correct.

Correct.

Question 1 Marks: 1/1 For purposes of computing the deduction for qualified residence interest, a qualified residence includes the taxpayer's principal residence and one other residence of the taxpayer or spouse. Choose one answer. a. True Correct.

b. False p. 10-16 Correct Marks for this submission: 1/1. Question 2 Marks: 1/1 Judy paid $40 for Girl Scout cookies and $40 for Boy Scout popcorn. Judy may claim an $80 charitable contribution deduction. Choose one answer. a. True b. False Correct.

The deduction is the excess over the value of the benefit received (i.e., the cookies and popcorn). p. 10-20 Correct Marks for this submission: 1/1. Question 3 Marks: 1/1 Interest paid or accrued during the tax year on aggregate acquisition indebtedness of $2 million or less ($1 million or less for married persons filing separate returns) is deductible as qualified residence interest. Choose one answer. a. True b. False Correct.

The $1 million limit for qualified residence interest applies to acquisition indebtedness ($500,000 or less for married persons filing separately). p. 10-16 Correct Marks for this submission: 1/1. Question 4 Marks: 1/1 Taxes assessed for local benefits, such as a new sidewalk, are deductible as real property taxes. Choose one answer. a. True b. False Correct.
Such taxes must be capitalized (added to the adjusted basis of the taxpayers property). p. 1010

Correct

Marks for this submission: 1/1. Question 5 Marks: 1/1 During the year, Victor spent $300 on bingo games sponsored by his church. If all profits went to the church, Victor has a charitable contribution deduction of $300. Choose one answer. a. True b. False Correct.

Costs of raffles, bingo, or lottery tickets do not qualify for the charitable deduction regardless of who receives the profits. p. 10-21 Correct Marks for this submission: 1/1. Question 6 Marks: 1/1 Medical expenses must relate to a particular ailment in order to be deductible. The cost of a periodic physical exam is not deductible because it relates to the taxpayer's general state of health. Choose one answer. a. True b. False Correct.

Medical expenses do not have to relate to a particular ailment in order to be deductible. p. 10-4 Correct Marks for this submission: 1/1. Question 7 Marks: 0/1 Gordy contributed stock worth $22,000 to a qualified charity. He acquired the stock fourteen months ago for $7,000. He may deduct $22,000 as a charitable contribution deduction (subject to percentage limitations). Choose one answer. a. True b. False Incorrect.

Gordys deduction is $22,000, the FMV of the property. Long-term capital gain property contributions are measured by FMV. p. 10-24 and Example 33 Incorrect Marks for this submission: 0/1. Question 8 Marks: 1/1

Joe, a cash basis taxpayer, took out a 12-month business loan on December 1, 2009. He prepaid all $3,600 of the interest on the loan on December 1, 2009. Joe may deduct $300 of the prepaid interest in 2009. Choose one answer. a. True b. False Cash basis taxpayers are required to allocate prepaid interest to the tax years to which the interest relates. Therefore, Joe can deduct $300 in 2009 and $3,300 in 2010. p. 10-18 Correct Marks for this submission: 1/1. Question 9 Marks: 1/1 In 2010, Rhonda received an insurance reimbursement for medical expenses incurred in 2009. She is not required to include the reimbursement in gross income in 2010 if she claimed the standard deduction in 2009. Choose one answer. a. True b. False The reimbursement is included in gross income only to the extent the taxpayer derived a tax benefit from deducting the expense in the previous year. Because Rhonda did not itemize in 2009, she received no tax benefit. Example 12 Correct Marks for this submission: 1/1. Question 10 Marks: 1/1 In order to dissuade his pastor from resigning and taking a position with a larger church, Michael, an ardent leader of the congregation, gives the pastor a new car. The cost of the car is deductible by Michael as a charitable contribution. Choose one answer. a. True b. False Correct. Correct. Correct.

The pastor is not a qualified charitable organization. p. 10-21 Correct Marks for this submission: 1/1. Question 11 Marks: 1/1

During 2009, Dustin made the following contributions to the University of New Mexico (a qualified charitable organization): Cash $32,000 Stock in Bluejay, Inc. (a publicly traded 73,000 corporation) Dustin acquired the stock in Bluejay, Inc. as an investment fifteen months ago at a cost of $28,000. Dustins AGI for 2009 is $126,000. What is Dustins charitable contribution deduction for 2009? Choose one answer. a. $37,800. b. $42,000. c. $63,000. d. $105,000. e. None of the above. No reduction for the appreciation on the Bluejay, Inc. stock is necessary because, if sold, it would yield a long-term capital gain. Thus, Dustins potential charitable deduction for 2009 is $105,000 [$32,000 (cash) + $73,000 (fair market value of Bluejay, Inc. stock)], but his allowable contribution deduction for the year is limited to $63,000 (50% of $126,000 AGI). Although the 30% of AGI limitation applies to long-term capital gain property, which would result in a current deduction for the Bluejay, Inc. stock of $37,800 (30% of $126,000 AGI), the overall 50% of AGI limitation applies to limit the total deduction to $63,000 for 2009. When the contributions for the tax year involve both 50% property (the cash of $32,000 in this case) and 30% property (the Bluejay, Inc. stock), the allowable deduction comes first from the 50% property. Therefore, Dustins allowable deduction of $63,000 for 2009 consists of: Cash $32,000 Bluejay, Inc. stock [overall limitation of $63,000 (50% of 31,000 $126,000 AGI) $32,000 (cash)] $63,000 The unused portion of the Bluejay, Inc. stock contribution of $42,000 [$73,000 (fair market value) $31,000 (portion used in 2009)] may be carried over for five years. The carryover continues to be classified as 30% property in the carryover years. If Dustin plans his charitable deductions wisely, sooner or later he will be able to deduct the full $73,000 fair market value of the stock. pp. 10-25 to 10-27 Correct Marks for this submission: 1/1. Question 12 Marks: 1/1 Correct.

Ron and Tom are equal owners in Robin Corporation. On July 1, 2009, each loans the corporation $20,000 at annual interest of 10%. Ron and Tom are brothers. Both shareholders are on the cash method of accounting, while Robin Corporation is on the accrual method. All parties use the calendar year for tax purposes. On June 30, 2010, Robin repays the loans of $40,000 together with the specified interest of $4,000. How much of the interest can Robin Corporation deduct in 2009? Choose one answer. a. $0. b. $1,000. c. $2,000. d. $4,000. e. None of the above. Robin Corporation can deduct interest expense of $4,000 in 2010 and $0 in 2009. Under 267, Ron and Tom are regarded as related to the corporation. Consequently, the deductibility must await actual payment (in 2010). p. 10-18 Correct Marks for this submission: 1/1. Question 13 Marks: 1/1 Which of the following items would be an itemized deduction on Schedule A of Form 1040 subject to the 2% of AGI floor? Choose one answer. a. Professional dues paid by an accountant (employed by Ford Motor Co.) to the National Association of Accountants. b. Fees paid to tax return preparer. c. Job hunting costs. d. Appraisal fee paid to a valuation expert to determine the fair market value of art work donated to a qualified museum. e. All of the other deductions are subject to the 2% of AGI floor. Correct. Correct.

Items a., b., c., and d. are miscellaneous deductions and are subject to the 2% of AGI limitation. p. 10-28 Correct Marks for this submission: 1/1. Question 14 Marks: 1/1 Which of the following is not deductible as a charitable contribution? Choose one answer.

a. Donations to a veterans organization. b. Donations to a cemetery company. c. Donations to a state or possession of the United States. d. Donations of blood to a blood blank. Correct.

e. All of the above are deductible. All of the donations are valid charitable deductions except the donation of blood to a blood bank. p. 10-21 Correct Marks for this submission: 1/1. Question 15 Marks: 1/1 Edna had an accident while competing in a rodeo. She sustained facial injuries that required cosmetic surgery. While having the surgery done to restore her appearance, she had additional surgery done to reshape her chin, which was not injured in the accident. The surgery to restore her appearance cost $9,000 and the surgery to reshape her chin cost $6,000. How much of Edna's surgical fees will qualify as a deductible medical expense (before application of the 7.5% limitation)? Choose one answer. a. $0. b. $6,000. c. $9,000. d. $15,000. e. None of the above. Cosmetic surgery is necessary (and therefore deductible) when it ameliorates (1) a deformity arising from a congenital abnormality, (2) a personal injury, or (3) a disfiguring disease. The $9,000 cost incurred in connection with the restorative surgery (required as a result of the accident) is deductible because the surgery was necessary. Amounts paid for the unnecessary cosmetic surgery ($6,000 for reshaping the chin) are not deductible as a medical expense. Examples 2 and 3 Correct Marks for this submission: 1/1. Question 16 Marks: 1/1 Shanna sold her personal residence to Christopher on March 1 for $200,000. Before the sale, Shanna paid the real estate taxes of $3,000 for the calendar year. For income tax purposes, the real estate tax deduction is apportioned as follows: $750 to Shanna and $2,250 to Christopher. Christopher's basis in the residence is: Choose one answer. a. $197,750. Correct. Correct.

b. $199,250. c. $200,750. d. $202,250. e. None of the other answers are correct. Christopher's basis in the residence is $197,750 [$200,000 (purchase price) - $2,250 (property taxes allocated to Christopher but paid by Shanna)]. pp. 10-12, 10-13, and Example 19 Correct Marks for this submission: 1/1. Question 17 Marks: 1/1 In 2009, Gerard pays $4,000 to become a charter member of Private Universitys Athletic Council. The membership ensures that Gerard will receive choice seating at all of Privates home football games. Also in 2009, Gerard pays $1,000 (the regular retail price) for season tickets for himself and his wife. For these items, how much qualifies as a charitable contribution? Choose one answer. a. $0. b. $1,000. c. $3,200. d. $5,000. e. None of the above. Under the exception to the tangible-benefit-received rule, Gerard can deduct $3,200 (80% of $4,000) of the charter membership to Private Universitys Athletic Council. The amount Gerard pays ($1,000) to purchase tickets at the regular price is not deductible as a charitable contribution. p. 10-20 and Example 30 Correct Marks for this submission: 1/1. Question 18 Marks: 1/1 Which of the following qualifies for the medical expense deduction? Choose one answer. a. Funeral expenses. b. Toiletries. c. Nonprescription drugs. d. Long-term care insurance premiums. e. None of the other answers are correct. Correct. Correct.

None of the listed expenses qualify for the medical expense deduction, except long-term care insurance premiums. Exhibit 10-1 Correct Marks for this submission: 1/1. History of Responses: # Action Response Long-term care insurance premiums. Long-term care insurance premiums. Time 07:33:49 on 3/04/12 07:33:49 on 3/04/12 Raw score Grade 1 1 1 1

1 Grade 2 Close&Grade

Question 19 Marks: 1/1 Joseph and Sandra, married taxpayers, took out a mortgage on their home for $350,000 in 1989. In May of 2005, when the home had a fair market value of $450,000 and they owed $250,000 on the mortgage, they took out a home equity loan for $220,000. They used the funds to purchase a single engine airplane to be used for recreational travel purposes. What is the maximum amount of debt on which they can deduct home equity interest? Choose one answer. a. $50,000. b. $100,000. c. $220,000. d. $230,000. e. None of the above. Interest is deductible only on the portion of the $220,000 home equity loan that does not exceed the lesser of: Correct.

The fair market value of the residence, reduced by the acquisition indebtedness ($450,000 FMV $250,000 acquisition indebtedness = $200,000).

$100,000 ($50,000 for married persons filing separate returns).

Of the $220,000 home equity loan, interest on $100,000 is deductible as home equity interest. p. 10-16 and Example 26 Correct Marks for this submission: 1/1. History of Responses: # Action Response Time Raw score Grade

1 2

Grade Close&Grade

$100,000. $100,000.

07:37:24 on 3/04/12 07:37:24 on 3/04/12

1 1

1 1

Question 20 Marks: 1/1 Byron owned stock in Blossom Corporation that he donated to a museum (a qualified charitable organization) on June 8, 2009. What is the amount of Byron's deduction assuming that he had purchased the stock for $10,500 on August 7, 2008, and the stock had a value of $13,800 when he made the donation? Choose one answer. a. $3,300. b. $10,500. c. $12,150. d. $13,800. e. None of the above. If ordinary income property is contributed to a qualified charitable organization, the deduction is equal to the fair market value of the property less the amount of ordinary income that would have been reported if the property were sold. In most instances, the deduction is limited to the adjusted basis of the property to the donor. Since he had not held the property long enough to meet the long-term capital gain requirement (August 7, 2008 - June 8, 2009), Byron would have recognized a short-term capital gain of $3,300 if he had sold the property. Since short-term capital gain property is treated as ordinary income property for charitable contribution purposes, Byrons charitable contribution deduction is limited to the propertys adjusted basis of $10,500 ($13,800 $3,300). Example 32 Correct Marks for this submission: 1/1. History of Responses: # 1 Grade Action Response $10,500. Time 07:38:00 on 3/04/12 1 Raw score 1 Grade Correct.

2 Close&Grade $10,500. 07:38:00 on 3/04/12 1 1 Question 21 Marks: 1/1 Rosita is employed as a systems analyst. For calendar year 2009, she had AGI of $120,000 and paid the following medical expenses: Medical insurance premiums Doctor and dentist bills for Jose and Carmen (Rosita's parents) Doctor and dentist bills for Rosita Prescribed medicines for Rosita Nonprescribed insulin for Rosita $3,900 8,250 6,750 300 825

Jose and Carmen would qualify as Rosita's dependents except that they file a joint return. Rosita's medical insurance policy does not cover them. Rosita filed a claim for $3,150 of her own expenses with her insurance company in December 2009 and received the reimbursement in January 2010. What is Rosita's maximum allowable medical expense deduction for 2009? Choose one answer. a. $2,775. b. $11,025. c. $17,325. d. $17,775. e. None of the above. Rosita's medical expense deduction is $11,025, determined as follows: Medical insurance premiums Doctor and dentist bills for Jose and Carmen Doctor and dentist bills for Rosita Prescribed medicines for Rosita Nonprescribed insulin for Rosita Total medical expenses Less: 7.5% of $120,000 (AGI) Deductible portion of medical expenses $ 3,900 8,250 6,750 300 825 $20,025 (9,000) $11,025 Correct.

Although Jose and Carmen cannot be claimed as Rosita's dependents, they could have been had they not filed a joint return. Therefore, they qualify for the medical expense deduction. Insulin is an exception to the rule that nonprescribed drugs do not qualify as medical expenses. The insurance recovery was not received until 2010. Therefore, it has no effect on the medical expense deduction for 2009. pp. 10-3 to 10-9 and Exhibit 10-1 Correct Marks for this submission: 1/1. History of Responses: # Action Response Time Raw score Grade

1 2

Grade Close&Grade

$11,025. $11,025.

07:42:04 on 3/04/12 07:42:04 on 3/04/12

1 1

1 1

Question 22 Marks: 1/1 In 2009, Hannah makes the following charitable donations: Basis $ 8,000 16,000 4,000 Fair Market Value $ 7,200 40,000 20,000

Inventory held for resale in Hannah's business (a sole proprietorship) Stock in HBM held as an investment (acquired four years ago) Baseball card collection held as an investment (acquired six years ago)

The HBM stock and the inventory were given to Hannah's church, and the baseball card collection was given to the United Way. Both donees promptly sold the property for the stated fair market value. Disregarding percentage limitations, Hannah's charitable contribution deduction for 2009 is: Choose one answer. a. $28,000. b. $51,200. c. $52,000. d. $67,200. e. None of the above. Inventory is ordinary income property, but the fair market value ($7,200) must be used if lower than the basis ($8,000). Stock is intangible property and is not subject to the tangible personalty rules. Since a sale of the HBM stock would have yielded a long-term capital gain, the full fair market value qualifies for the deduction ($40,000). The baseball card collection comes under the tangible personalty exception, and the adjusted basis ($4,000) must be used. Thus, $7,200 + $40,000 + $4,000 = $51,200. pp. 10-24 and 10-25 Correct Marks for this submission: 1/1. History of Responses: # 1 Grade Action Response $51,200. Time 07:43:24 on 3/04/12 1 Raw score 1 1 Grade Correct.

2 Close&Grade $51,200. 07:43:24 on 3/04/12 1 Question 23 Marks: 1/1 During 2009, Courtney paid the following interest charges:

Home mortgage

$5,000 1,500

On loan to purchase a new car On student loan 800

If Courtney itemizes her deductions for 2009, the amount deductible for interest expense as an itemized deduction is:

Choose one answer. a. $1,500. b. $5,000. c. $6,500. d. $7,300. e. None of the above. The $1,500 interest on the loan to purchase the new car is nondeductible consumer interest. The $800 interest on the student loan is deductible for AGI and not from AGI as an itemized deduction. Only the home mortgage interest of $5,000 is deductible as an itemized deduction. pp 10-13 to 10-16 Correct Marks for this submission: 1/1. History of Responses: # 1 Grade Action Response $5,000. Time 08:09:59 on 3/04/12 1 Raw score 1 Grade Correct.

2 Close&Grade $5,000. 08:09:59 on 3/04/12 1 1 Question 24 Marks: 1/1 Your friend Scotty informs you that he received a "tax-free" reimbursement in 2010 of some medical expenses he paid in 2009. Which of the following statements best explains why Scotty is not required to report the reimbursement in gross income? Choose one answer. a. Scotty itemized deductions in 2009. b. Scotty did not itemize deductions in 2009. c. Scotty itemized deductions in 2010. d. Scotty did not itemize deductions in 2010. e. Scotty itemized deductions in 2010 but not in 2009. Correct.

If Scotty did not itemize in 2009, he can exclude the reimbursement from gross income in 2010. If Scotty itemized deductions in 2009, he must report the reimbursement as gross income in 2010 to the extent he received a tax benefit from deducting medical expenses in 2009. Whether he itemized in 2010 will have no impact on the treatment of the reimbursement. Example 12 Correct Marks for this submission: 1/1. History of Responses: # Action Response Scotty did not itemize deductions in 2009. Scotty did not itemize deductions in 2009. Time 07:50:51 on 3/04/12 07:50:51 on 3/04/12 Raw score Grade 1 1 1 1

1 Grade 2 Close&Grade

Question 25 Marks: 1/1 Phil is advised by his family physician that his dependent son, Tony, needs surgery for a benign tumor in his leg. Phil and his son travel to Rochester, Minnesota, for in-patient treatment at the Mayo Clinic which specializes in this type of surgery. Phil incurred the following costs: Round-trip airfare ($375 each) Phil's hotel in Rochester for four nights ($105 per night) Phil's meals while in Rochester Tony's medical treatment Tony's prescription medicine $ 750 420 150 1,500 300

Compute Phil's medical expenses for the trip (subject to the 7.5% floor). Choose one answer. a. $2,550. b. $2,750. c. $2,970. d. $3,120. e. None of the above. Phil's medical expense deduction for transportation is $750, and his medical expense deduction for lodging is $200 ($50 per night per person maximum). Phil is not allowed a deduction for the cost of his meals while in Rochester. His son's medical treatment and prescriptions are deductible. So his total deduction (before the 7.5% floor) is $2,750 ($750 + $200 + $1,500 + $300). pp. 10-7 and 10-8 Correct.

Correct

Chapter 11 Exam Review of attempt 1


Started on Tuesday, 3 April 2012, 08:38 AM Completed on Wednesday, 4 April 2012, 08:53 AM Time taken 1 day 23/25 Marks 46 out of a maximum of 50 (92%) Grade Question 1 Marks: 1/1 Gail exchanges passive Activity A, which has suspended losses of $15,000, for passive Activity B in a nontaxable exchange. The new owner of passive Activity A can offset the $15,000 suspended losses against passive income in the future. Choose one answer. a. True b. False Correct.

In a nontaxable exchange of a passive investment, the suspended losses are kept by the taxpayer. Gail, not the new owner of passive Activity A, will be allowed to offset the $15,000 suspended loss against passive income in the future. Examples 48 and 49 Correct Marks for this submission: 1/1. Question 2 Marks: 1/1 Coyote Corporation has active income of $45,000 and a passive loss of $23,000 in the current year. Coyote cannot deduct the $23,000 loss if it is a personal service corporation. Choose one answer. a. True Correct. b. False Personal service corporations may not offset passive losses against active income. Example 14 Correct Marks for this submission: 1/1. Question 3 Marks: 1/1

From January through November, Vern participated for 420 hours as a salesman in a partnership in which he owns a 50% interest. The partnership has four full-time employees. During December, Vern spends 110 hours cleaning the store and painting the walls in order to meet the material participation standards. Vern qualifies as a material participant. Choose one answer. a. True Correct. b. False Work performed in an individuals capacity as an investor or in performance of duties not normally performed by owners does not count toward material participation if such work is performed primarily to meet the material participation requirements. Example 29 Correct Marks for this submission: 1/1. Question 4 Marks: 1/1 If an owner participates for more than 500 hours in a DVD rental activity, any loss from that activity will be treated as an active loss that can be offset against active income. Choose one answer. a. True b. False A DVD rental activity is subject to the material participation rules because it is not treated as a rental activity. As the owner participated more than 500 hours, this is not a passive activity. REF: p. 11-18; Example 31 Correct Marks for this submission: 1/1. Question 5 Marks: 1/1 Chris receives a gift of a passive activity from his father whose basis was $60,000. Suspended losses related to the activity are $18,000. Chris will be allowed to offset the $18,000 suspended losses against future passive income. Choose one answer. a. True Correct. b. False A gift of a passive activity results in the suspended losses being added to the adjusted basis of the property. Example 46 Correct Marks for this submission: 1/1. Question 6 Marks: 1/1 In the current year, Louise invests $50,000 for a 10% interest in a passive activity. Her share of the loss this year is $10,000. If this is her only passive activity, the $10,000 loss from the activity this year is suspended for use in a future year. Correct.

Choose one answer. a. True b. False Assuming Louise had no passive income during the year, she may not deduct any of the loss from the activity in the current year. Example 6 Correct Marks for this submission: 1/1. Question 7 Marks: 1/1 Dick participates in an activity for 90 hours during the year. He has no employees and there are no other participants. Dick is a material participant. Choose one answer. a. True Correct. b. False Example 23 and related discussion Correct Marks for this submission: 1/1. Question 8 Marks: 1/1 Services performed by an employee are treated as being related to a real estate trade or business if the employee performing the services has more than a 5% ownership interest in the employer. Choose one answer. a. True Correct. b. False Services performed by an employee are treated as being related to a real estate trade or business only if the employee performing the services has more than a 5% ownership interest in the employer. p. 11-22 Correct Marks for this submission: 1/1. Question 9 Marks: 1/1 Lucy owns and actively participates in the operations of an apartment complex that produces a $50,000 loss during the year. Her modified AGI is $125,000 from an active business. Disregarding any at-risk amount limitation, she may deduct $25,000 of the loss, and the remaining $25,000 is a suspended passive loss. Answer: True Correct. False Correct.

The $25,000 deduction is reduced by 50% of modified AGI in excess of $100,000. Therefore, only $12,500 of Lucy's rental loss can be deducted this year [$25,000 50%($125,000 - $100,000)]. The remaining $37,500 ($50,000 - $12,500) is a suspended passive loss. REF: p. 11-22 | p. 11-23 Correct Marks for this submission: 1/1. Question 10 Marks: 1/1 Wayne owns a small apartment building that produces a $45,000 loss during the year. His AGI before considering the rental loss is $85,000. Because Wayne is an active participant with respect to the rental activity, he may deduct the $45,000 loss. Answer: True Correct. False

The ceiling on the deductibility of such losses is $25,000, not $45,000. REF: p. 11-22 | p. 11-23 Correct Marks for this submission: 1/1. Question 11 Marks: 1/1 In 2006, Joan invests $125,000 for a 25% partnership interest in an activity in which she is a material participant. The partnership reports losses of $300,000 in 2006 and $300,000 in 2007. Joans share of the partnerships losses is $75,000 in 2006 and $75,000 in 2007. How much of the losses from the partnership can Joan deduct? Choose one answer. a. $75,000 in 2006 and $75,000 in 2007. b. $75,000 in 2006 and $50,000 in 2007. c. $0 in 2006 and $0 in 2007. d. $75,000 in 2005 and $0 in 2006. e. None of the above. Joans losses are not subject to the passive loss limitations because she is a material participant in the activity. However, the at-risk rules limit her losses to $125,000, the amount she invested in the partnership interest. pp. 11-2 to 11-5 Correct Marks for this submission: 1/1. Question 12 Marks: 1/1 Correct.

Everest Corporation, a personal service corporation, has $300,000 of passive losses, $240,000 of active business income, and $60,000 of portfolio income. How much of the passive loss may Everest Corporation deduct? Choose one answer. a. $0. b. $60,000. c. $240,000. d. $300,000. e. None of the above. Everest Corporation, a personal service corporation, may not offset the passive loss against the active or portfolio income. Example 14 Correct Marks for this submission: 1/1. Question 13 Marks: 0/1 Cole has investments in two passive activities. Activity A, acquired three years ago, produces income in the current year of $275,000. Activity B, acquired last year, produces a loss of $375,000 in the current year. At the beginning of this year, Coles at-risk amounts in Activities A and B are $150,000 and $300,000, respectively. What is the amount of Coles suspended passive loss with respect to these activities at the end of the current year? Choose one answer. a. $0. b. $25,000. c. $50,000. d. $75,000. e. None of the above. The $275,000 of passive income from Activity A is offset by $275,000 of the passive loss from Activity B, leaving a net passive loss of $100,000. Of this loss $75,000 is suspended under the at-risk rules. None of the remaining $25,000 net passive loss is deductible in the current year. The remaining $25,000 net passive loss is suspended under the passive loss rules. p. 11-18 Incorrect Marks for this submission: 0/1. Question 14 Marks: 1/1 Maria, who owns a 50% interest in a restaurant, has been a material participant in the restaurant activity for the last 20 years. She retired from the restaurant at the end of last year and will not participate in the restaurant activity in the future. However, she continues to be a material participant in a retail store in which she is a 50% partner. The restaurant operations produce a loss for the current year, and Maria's share of the loss is $80,000. Her share of the income from the retail store is $150,000. She does not own interests in any other activities. Which of the following statements is correct? Incorrect. Correct.

Choose one answer. a. Maria cannot deduct the $80,000 loss from the restaurant because she is not a material participant. b. Maria can offset the $80,000 loss against the $150,000 of income from the retail Correct. store. c. Maria will not be able to deduct any losses from the restaurant until she has been retired for at least three years. d. None of the above. Statement b. is correct. Because Maria materially participated in the restaurant activity for at least five of the last ten taxable years before the current year (Test 5), she is still considered an active participant in the activity. Therefore, her loss is an active loss that can be offset against her active income from the retail store. Example 29 Correct Marks for this submission: 1/1. Question 15 Marks: 1/1 Irving invests in vacant land for the purpose of realizing a profit on its appreciation. He leases the land during the period he holds it. The unadjusted basis of the property is $25,000 and its fair market value is $35,000. The lease payments are $400 per year. Which of the following statements is correct? Choose one answer. a. The leasing activity will be treated as a rental activity and will be treated as a passive activity regardless of how many hours Irving participates. b. The leasing activity will be treated as a rental activity and will not be treated as a passive activity if Irving qualifies as a real estate professional. c. The leasing activity will not be treated as a rental activity. Correct. d. None of the above. This case illustrates an activity that is covered by one of the rental activity exceptions and, therefore, is not automatically treated as passive. Example 34 Correct Marks for this submission: 1/1. Question 16 Marks: 1/1 Identify from the list below the type of disposition of a passive activity where the taxpayer keeps the suspended losses of the disposed activity and utilizes them on a subsequent taxable disposition. Choose one answer. a. Disposition of a passive activity by gift. b. Nontaxable exchange of a passive activity. c. Disposition of a passive activity at death. d. Installment sale of a passive activity. Correct.

e. None of the above. Examples 50 and 51 Correct Marks for this submission: 1/1. Question 17 Marks: 1/1 Art owns significant interests in a hardware store and a bookstore at a mall in Washington, D.C. He also owns a hardware store and a bookstore at a mall in San Francisco. Which of the following is not a way in which the interests may be grouped? Choose one answer. a. One activity. b. A hardware activity and a bookstore activity. c. A Washington, D.C. activity and a San Francisco activity. d. Four separate activities. Correct. e. Any of the above may be the basis for grouping. Example 19 Correct Marks for this submission: 1/1. Question 18 Marks: 1/1 Jon owns an apartment building in which he is a material participant and a computer consulting business. Of the 2,000 hours he spends on these activities during the year, 55% of the time is spent operating the apartment building and 45% of the time is spent in the computer consulting business. Which of the following statements is correct? Choose one answer. a. The computer consulting business is a passive activity but the apartment building is not. b. The apartment building is a passive activity but the computer consulting business is not. c. Both the apartment building and the computer consulting business are passive activities. d. Neither the apartment building nor the computer consulting business is a Incorrect. passive activity. e. None of the above. More than half of Jons personal services are spent in a real property trade or business in which he materially participates, and the time spent exceeds 750 hours. Thus, the apartment building is not a passive activity. Because Jon participates for more than 500 hours in the computer consulting business, it also is not a passive activity. pp. 11-15 and 11-21 Correct Marks for this submission: 1/1.

Question 19 Marks: 1/1 Shalom owns four separate activities. She elects not to group them together as a single activity under the appropriate economic unit standard. Shalom participates for 121 hours in Activity A, 125 hours in Activity B, 110 hours in Activity C, and 145 hours in Activity D. She has one employee, who works 200 hours in Activity A. Which of the following statements is correct? Choose one answer. a. Losses from the activities cannot be used to offset Shaloms active income. b. Activities A, B, C, and D are not all significant participation activities. c. Shalom is a material participant with respect to only Activity A. d. Shalom is a material participant with respect to Activities A, B, C, and D. Correct.

e. None of the above. Activities A, B, C and D are all significant participation activities. A significant participation activity is one in which the individuals participation exceeds 100 hours during the year. The material participation standards are met with respect to Activities A, B, C and D because total participation in significant participation activities (121 + 125 + 110 + 145=501) exceeds 500 hours. Shalom is not considered to be a material participant with respect to activity A alone even though her participation exceeds 100 hours since someone elses participation in the activity exceeds hers (Test 3). Examples 25, 26, and related discussion Correct Marks for this submission: 1/1. Question 20 Marks: 1/1 Josh has investments in two passive activities. Activity A, acquired three years ago, produces income in the current year of $60,000. Activity B, acquired last year, produces a loss of $100,000 in the current year. At the beginning of this year, Josh's at-risk amounts in Activities A and B are $10,000 and $100,000, respectively. What is the amount of Josh's suspended passive loss with respect to these activities at the end of the current year? Choose one answer. a. $0. b. $36,000. c. $40,000. d. $100,000. e. None of the above. The $60,000 of passive income from Activity A is offset by $60,000 of the passive loss from Activity B, leaving a net passive loss of $40,000. None of the $40,000 net passive loss is deductible in the current year. The $40,000 net passive loss is suspended. pp. 11-8 and 11-20 Correct Marks for this submission: 1/1. Correct.

Question 21 Marks: 1/1 Jenny spends 32 hours a week, 50 weeks a year, operating a DVD rental store that she owns. She also owns a music store in another city that is operated by a full-time employee. Jenny spends 140 hours per year working at the music store. She elects not to group them together as a single activity under the appropriate economic unit standard. Which of the following statements is correct? Choose one answer. a. Neither store is a passive activity. b. Both stores are passive activities. c. Only the DVD rental store is a passive activity. d. Only the music store is a passive activity. e. None of the above.
A DVD rental store is not treated as a rental activity because the average period of customer use is 7 days or less. Because Jenny participates for more than 500 hours during the year, the DVD rental store is treated as an active business. Jenny participates for more than 100 hours in the music store which makes it a significant participation activity. Her total participation in the two significant participation activities is more than 500 hours, so neither business is treated as passive.

Correct.

REF: p. 11-13 | p. 11-14 | p. 11-16 | Example 25 | Example 31 Correct Marks for this submission: 1/1. Question 22 Marks: 1/1 Matt has three passive activities and has at-risk amounts in excess of $100,000 for each. During the year, the activities produced the following income (losses). Activity A Activity B Activity C Net passive loss ($75,000) (25,000) 25,000 ($75,000)

Matt's suspended losses are as follows: Choose one answer. a. $75,000 is allocated to C; $0 to A and B. b. $37,500 is allocated to A; $37,500 to B. c. $56,250 is allocated to A; $18,750 to B. d. $25,000 is allocated to A, B, and C. e. None of the above. Correct.

$75,000/$100,000 X $75,000 = $56,250 allocated to Activity A $25,000/$100,000 X $75,000 = $18,750 allocated to Activity B Example 9 Correct Marks for this submission: 1/1. Question 23 Marks: 0/1 Alicia, who is single, has $80,000 of salary, $60,000 of income from a limited partnership and a $100,000 passive loss from a real estate rental activity in which she actively participates. Her modified adjusted gross income is $175,000. Of the $100,000 passive rental loss, how much is deductible in the current year? Choose one answer. a. $0. b. $60,000. c. $85,000. d. $100,000. e. None of the above. $60,000. A loss of $60,000 is deducted against the passive income from the limited partnership interest and $0 of the remaining $40,000 rental real estate loss is deducted against Alicias salary even though she actively participates in the activity. The special $25,000 offset for rental real estate is reduced to $0 [$25,000 ($175,000 - $100,000).50]. Therefore, of the remaining $40,000 loss, $0 is deducted in the current year. pp. 11-20 and 11-21 Incorrect Marks for this submission: 0/1. Question 24 Marks: 1/1 Art's at-risk amount in a passive activity was $60,000 at the beginning of 2006. His loss from the activity in 2006 is $80,000, and he had no passive activity income during the year. Art had $20,000 of passive income from the activity in 2007. Which of the following statements is correct? Choose one answer. a. At the end of 2007, Art has a total loss of $20,000 suspended under the passive loss rules. b. At the end of 2007, Art has a total loss of $15,000 suspended under the passive loss rules. c. At the end of 2007, Art has a total loss of $45,000 suspended under the passive loss rules. d. At the end of 2007, Art has a total loss of $60,000 suspended under the passive Correct. loss rules. e. None of the other answers are correct. Incorrect.

In 2006, Art had a $80,000 loss, $20,000 of which was suspended under the at-risk rules; $60,000 was suspended under the passive loss rules. The $20,000 of passive income in 2007 increases Arts at-risk amount and allows for reclassification as a passive loss $20,000 of the 2006 loss suspended under the at-risk rules. Therefore, Art has a total loss of $60,000 suspended under the passive loss rules ($60,000 in 2006 + $20,000 reclassified in 2007 $20,000 of passive losses offset against passive income). Examples 40 and 41 Correct Marks for this submission: 1/1. Question 25 Marks: 1/1 Ned, a college professor, owns a separate business (not real estate) in which he participates in the current year. He has one employee who works part-time in the business. Which of the following statements is correct? Choose one answer. a. If Ned participates for 120 hours and the employee participates for 120 hours during the year, Ned does not qualify as a material participant. b. If Ned participates for 95 hours and the employee participates for 5 hours during the year, Ned probably does not qualify as material participant. c. If Ned participates for 500 hours and the employee participates for 520 hours during the year, Ned qualifies as material participant. d. If Ned participates for 600 hours and the employee participates for 2,000 hours Correct. during the year, Ned qualifies as a material participant. e. None of the above. Option a. is incorrect; Ned participates for more than 100 hours and this is not less than the participation of any other individual (Test 3). Option b. is incorrect; Neds participation constitutes substantially all of the participation, even though Neds participation is less than 100 hours (Test 2). Option c. is incorrect; Ned would have to participate for more than 500 hours for statement c. to be correct (Test 1). Option d. is correct; an individual who participates for more than 500 hours is a material participant regardless of how much others participate (Test 1). pp. 11-14 and 11-15 Correct Marks for this submission: 1/1.

Chapter 7 Exam Review of attempt 1

Started on Tuesday, 13 March 2012, 09:52 PM Completed on Monday, 23 April 2012, 07:25 PM Time taken 40 days 21 hours 21/25 Marks 42 out of a maximum of 50 (84%) Grade Question 1 Marks: 0/1 Al has a gain of $30,000 on the sale of 1244 stock (small business stock) and a loss of $35,000 on the sale of 1244 stock. As a result, Al has a $30,000 net capital gain and a $35,000 ordinary loss. Choose one answer. a. True Incorrect. b. False The $30,000 gain on the sale of the 1244 stock is classified as a capital gain. The $35,000 loss on the sale of the 1244 stock is classified as an ordinary loss. p. 7-7 Incorrect Marks for this submission: 0/1. Question 2 Marks: 1/1 The excess of nonbusiness capital losses over nonbusiness capital gains must be added to taxable income to compute the net operating loss of an individual. Choose one answer. a. True Correct. b. False p. 7-21 Correct Marks for this submission: 1/1. Question 3 Marks: 1/1 A loss attributable to rental property is classified as a deduction for adjusted gross income. Choose one answer. a. True Correct. b. False p. 7-14 Correct Marks for this submission: 1/1. Question 4 Marks: 1/1 Accrual basis taxpayers can use the reserve method for computing deductions for bad debts. Choose one answer. a. True

Correct. b. False Taxpayers can use only the specific charge-off method in accounting for bad debts. The reserve method is not permitted. p. 7-3 Correct Marks for this submission: 1/1. Question 5 Marks: 1/1 A nonbusiness bad debt is treated as an ordinary loss. Choose one answer. a. True Correct. b. False A nonbusiness bad debt is treated as a short-term capital loss. p. 7-5 Correct Marks for this submission: 1/1. Question 6 Marks: 1/1 If personal casualty losses (after deducting the $100 floor) exceed personal casualty gains, the itemized deduction is always equal to the losses, to the extent they exceed 10% of adjusted gross income. Choose one answer. a. True Correct. b. False The losses are subject to the 10% of adjusted gross income floor only to the extent they exceed the gains. p. 7-15 Correct Marks for this submission: 1/1. Question 7 Marks: 1/1 An NOL incurred in 2011 can be carried back for 2 years. Choose one answer. a. True Correct. b. False p. 7-19 Correct Marks for this submission: 1/1. Question 8 Marks: 1/1 In determining the amount of remaining NOL to be carried forward, the taxable income of the year to which the loss is being applied must be determined without regard to the excess of capital losses over capital gains. Choose one answer. a. True Correct.

b. False p. 7-23 Correct Marks for this submission: 1/1. Question 9 Marks: 1/1 "Other casualty" means casualties similar to those associated with fires, storms, or shipwrecks. Choose one answer. a. True Correct. b. False p. 7-8 Correct Marks for this submission: 1/1. Question 10 Marks: 1/1 If rental property is completely destroyed, the amount of the loss is the lesser of the fair market value of the property or the adjusted basis of the property at the time of the destruction. Choose one answer. a. True Correct. b. False The amount of the loss is equal to the adjusted basis of the property. p. 7-11 Correct Marks for this submission: 1/1. Question 11 Marks: 0/1 Janice had a car accident in which Janice was cited for willful negligence. An appraisal established that the fair market value of Janice's car declined by $3,000 as a result of the accident. The car was used by Janice for personal use. To have the car repaired, Janice paid $5,000. Her insurance company reimbursed $500. If Janice's AGI for the year was $20,000, determine her deductible casualty loss on the car. Choose one answer. a. $400. b. $500. c. $1,200. d. $2,400. e. None of the above. Janices loss was caused by her willful negligence. Thus, a casualty loss deduction is not permitted. p. 7-9 Incorrect Marks for this submission: 0/1. Question 12 Incorrect.

Marks: 1/1 Amos loaned James (a friend) $10,000 in 2010 with the agreement that the loan would be repaid in two years. In 2011, James filed for bankruptcy, and Amos was notified that he could expect $0.40 on the dollar. In 2012, final settlement was made, and Amos received $1,500. In 2011, Amos had AGI of $70,000, including $6,000 of short-term capital gain. In 2012, Amos had AGI of $80,000, including $4,000 of long-term capital gain. Assuming the loan is a nonbusiness bad debt, what is the amount available for Amos's bad debt deduction in 2012? Choose one answer. a. $1,500. b. $2,500. c. $6,000. d. $8,000 . Correct. e. $ 8,500. The non-business bad debt will be treated as a short-term capital loss in the year of settlement. The amount of the bad debt in 2012 is $8,500 ($10,000 - $1,500). Of the $8,500, $4,000 can be used to offset $4,000 of long-term capital gain and $3,000 can be used to offset ordinary income in 2012 because of the limit on capital losses. The remaining $1,500 is available to carry forward to the next year. p. 7-4 and 7-5 Correct Marks for this submission: 1/1. Question 13 Marks: 0/1 During the year, Wilsons personal residence was damaged by fire. Wilson was insured for 90% of his actual loss, and he received the insurance settlement. Wilson had adjusted gross income, before considering the casualty item, of $30,000. Pertinent data with respect to the residence follows: Cost basis Value before casualty Value after casualty $170,000 230,000 40,000

What is Wilsons allowable casualty loss deduction? Choose one answer. a. $0. b. $12,900. c. $13,000. d. $19,000 Incorrect.

e. None of the other answers are correct. The proceeds received are $171,000 [($230,000 - $40,000) X 90%]. The proceeds received are $1,000 ($171,000 - $170,000) greater than Wilson's adjusted basis for his personal residence. Hence, Wilson has no casualty loss. pp. 7-11 to 7-14 Incorrect Marks for this submission: 0/1. Question 14 Marks: 1/1 Steve and Holly have the following items for 2011: Dividend income Interest income Itemized deductions (none of the amount resulted from a casualty loss) Business capital gains Business capital losses $5,000 7,000 (9,000) 1,000 (3,000)

In calculating their net operating loss, and with respect to the above amounts only, what amount must be added back to taxable income (loss)? Choose one answer. a. $0. b. $1,300. c. $11,000. d. $3,000. e. $1,200. Correct.

$3,000 - [$1,000 + ($12,000 - $11,600*)] = $1,200. *Standard deduction would be used. REF: p. 7-21 | p. 7-22 Correct Marks for this submission: 1/1. Question 15 Marks: 1/1 Five years ago, Tom loaned his son John $20,000 to start a business. A note was executed with an interest rate of 8%, which is the Federal rate. The note required monthly payments of the interest with the $20,000 due at the end of ten years. John always made the interest payments until last year. During the current year, John notified his father that he was bankrupt and would not be able to repay the $20,000 or the accrued interest of $1,800. Tom is a cash basis taxpayer whose only income is salary and interest income. The proper treatment for the nonpayment of the note is:

Choose one answer. a. No deduction. b. $3,000 deduction. c. $20,000 deduction. d. 21,800 deduction. e. None of the other answers are correct. This is a bona fide loan to his son; therefore, Tom is entitled to a bad debt of $20,000. The deduction for the current year is limited to $3,000, since the bad debt is classified as a short-term capital loss. No deduction is allowed for the $1,800 of accrued interest receivable because Tom is a cash basis taxpayer. pp. 7-3 to 7-6 Correct Marks for this submission: 1/1. Question 16 Marks: 1/1 John, who is single, had the following items for the current year: Salary of $90,000. Gain of $30,000 on the sale of 1244 stock acquired two years earlier. Loss of $75,000 on the sale of 1244 stock acquired three years earlier. Worthless stock of $7,000. The stock was acquired on February 1 of the prior year and became worthless on January 15 of the current year. Correct.

Determine John's AGI for the current year. Choose one answer. a. $37,000. b. $38,000. c. $42,000. d. $47,000. e. None of the other answers are correct. Salary 1244 ordinary loss Long-term capital gain Long-term capital loss Excess 1244 loss ($75,000-$50,000) $25,000 Worthless security 7,000 (32,000) $90,000 (50,000) $30,000 Correct.

Net long-term capital losss Adjusted gross income pp. 7-6 and 7-7 Correct Marks for this submission: 1/1. Question 17 Marks: 1/1

(2,000) $38,000

Joshua and Antonia file a joint return. In 2011, they had the following items: Salary of $90,000. Loss of $108,000 on the sale of 1244 stock acquired two years ago. Interest income of $4,000. Determine Joshua and Antonias AGI for 2011. Choose one answer. a. $0. b. ($6,000). c. ($9,000). d. ($14,000). e. None of the other answers are correct. Salary $ 90,000 + Interest income 4,000 - Small Business Stock Ordinary loss* (100,000) = AGI ($ 6,000). *$8,000 ($108,000 - $100,000) long-term capital loss will be carried forward. There is no ordinary income and hence, $3,000 of the capital loss that normally could be deducted against ordinary income cannot be used in the current year. pp. 7-5 to 7-8 Correct Marks for this submission: 1/1. Question 18 Marks: 1/1 Kiara, a married taxpayer filing a joint return, had the following items for 2011: Salary of $65,000. Correct.

Loss of $45,000 on the sale of stock acquired two years ago from Mac, an investor. Gain of $40,000 on the sale of 1244 stock acquired three years ago. Stock acquired on January 15, 2010, for $5,000 became worthless on July 1, 2011. Determine Kiaras AGI for 2011. Choose one answer. a. $65,000. b. $62,000. c. $55,000. d. $50,000. e. None of the other answers are correct. Salary $65,000 Long-term capital gain $40,000 Less long-term capital loss (not 1244 stock) (45,000) Net long-term capital loss ($ 5,000) Less short-term capital loss (worthless securities) ( 5,000) Net capital loss ($10,000) Limited. to (3,000) Adjusted gross income $62,000 pp. 7-6 and 7-7 Correct Marks for this submission: 1/1. Question 19 Marks: 1/1 In the case of theft of stock the allowable loss is reduced by: Choose one answer. a. $100 floor and 10% of AGI. b. $100 floor. c. 10% of AGI. d. Misc. Itemized deduction not subject to 2% of AGI. Correct. Correct.

e. None of the other answers are correct. Securities are property used in an activity entered into for profit but not attributable to rents and royalties. Therefore, the loss is classified as a miscellaneous itemized deduction not subject to the 2% of AGI floor. pp. 7-10 to 7-16 Correct Marks for this submission: 1/1. Question 20 Marks: 1/1 Tuwanna is the sole proprietor of National Loan Company. On May 1, 2010, National loaned Jerome $200,000. In 2011, Jerome filed for bankruptcy. At that time, it was

revealed that Jerome's creditors could expect to receive 75 cents on the dollar. In March 2012, final settlement was made, and National received $80,000. National's policy is to deduct losses as soon as permitted. How much loss can National deduct and in which year? Choose one answer. a. 2011 - $200,000. b. 2011 - zero; 2012 - $120,000. c. 2011 - $50,000; 2012 - $70,000. d. 2011 - $70,000; 2012 - $50,000. e. None of the other answers are correct. This debt was incurred in connection with a trade or business. Therefore, National can claim a bad debt deduction in the following years: 2010 - zero. 2011 $50,000[$200,000(loan)-$150,000 (expected settlement)]. 2012 $70,000[$150,000(balance)-$80,000 Proceeds)]. pp. 7-3 to 7-5 Correct Marks for this submission: 1/1. Question 21 Marks: 1/1 Which of the following is true regarding net operating losses? Choose one answer. a. They are carried back for 3 years. b. They are carried back for 5 years. c. They are carried back for 2 years and forward for 20 years. d. They are carried back for 1 years. e. None of the other answers are correct. Net operating losses are carried back for 2 years and forward for 20 years. p. 7-20 Correct Marks for this submission: 1/1. Question 22 Marks: 0/1 Sandy, a married taxpayer filing a joint return, had the following items for 2011: Salary of $40,000. Loss of $50,000 on the sale of stock acquired two years ago from Max, an investor. Gain of $55,000 on the sale of 1244 stock acquired three years ago. Stock acquired on January 15, 2011, for $15,000 became worthless on July 1, 2011. Determine Sandy's AGI for 2011. Correct. Correct.

Choose one answer. a. $40,000. b. $30,000. c. $37,000. d. $25,000. Incorrect.

e. None of the other answers are correct. Salary $40,000 + Long-term capital gain $55,000 Less long-term capital loss (not 1244 stock) (50,000) + Net long-term capital gain 5,000 Less short-term capital loss (worthless securities) ( 15,000) Net capital loss limited to $3,000 ($10,000) Limited To(3,000) Adjusted gross income $37,000 pp. 7-6 and 7-7 Incorrect Marks for this submission: 0/1. History of Responses: # 1 2 Grade Close&Grade Action Response $25,000. $25,000. Time 19:22:29 on 23/04/12 19:22:29 on 23/04/12 0 0 Raw score 0 0 Grade

Question 23 Marks: 1/1 On February 20, 2010, Bill purchased stock in Pink Corporation (the stock is not small business stock) for $1,000. On May 1, 2011, the stock became worthless. During 2011, Bill also had an $8,000 loss on 1244 small business stock purchased two years ago, a $9,000 loss on a nonbusiness bad debt, and a $5,000 long-term capital gain. How should Bill treat these items on his 2011 tax return? Choose one answer. a. $4,000 long-term capital loss and $9,000 short-term capital loss. b. $4,000 long-term capital loss and $3,000 short-term capital loss. c. $8,000 ordinary loss and $3,000 short-term capital loss. d. $8,000 ordinary loss and $5,000 short-term capital loss. e. $8,000 long-term capital loss and $6,000 short-term capital loss. Ordinary loss (small business stock) ($8,000) Long-term capital gain $5,000 Less long-term capital loss (worthless securities) 1,000) Net longterm capital gain $4,000 Less short-term capital loss (nonbusiness bad debt) (9,000) Net shortterm capital loss ($5,000) Short-term capital loss limited to ($3,000) pp. 7-5 and 7-7 Correct.

Correct Marks for this submission: 1/1. Question 24 Marks: 1/1 Nicholas loaned Lyle (a friend) $30,000 in 2010 with the agreement that the loan would be repaid in two years. In 2011, Lyle filed for bankruptcy and Nicholas learned that he could expect to receive $0.50 on the dollar. In 2012, final settlement was made and Nicholas received $16,000. Assuming the loan is a nonbusiness bad debt, how should Nicholas account for the bad debt? Choose one answer. a. $14,000 ordinary loss in 2012. b. $15,000 ordinary loss in 2011 and $9,000 ordinary loss in 2012. c. $14,000 short-term capital loss in 2012. d. $15,000 short-term capital loss in 2011 and $9,000 short-term capital loss in 2012. e. None of the other answers are correct. Nonbusiness bad debts are characterized as short-term capital losses of $14,000. The loss is recognized in the year of final settlement. pp. 7-3 to 7-5 Correct Marks for this submission: 1/1. Question 25 Marks: 1/1 Elizabeth has the following items for the current year: Nonbusiness capital gains Nonbusiness capital losses Interest income Itemized deductions (including a $20,000 casualty loss) $ 5,000 (3,000) 3,000 (24,000) Correct.

In calculating Elizabeth's net operating loss, and with respect to the above amounts only, what amount must be added back to taxable income (loss)? Choose one answer. a. $0. b. $1,000. c. $3,000. d. $20,000. e. None of the other answers are correct. ($24,000 $20,000) [$3,000 + ($5,000 $3,000)] = ($1,000). Therefore, nothing is added back. pp. 7-21 and 7-22 Correct.

Correct Marks for this submission: 1/1.

Chapter 17

1 Marks: 1/1 Nonrecaptured 1231 losses from the seven prior tax years may cause current year net 1231 gain to be treated as ordinary income. Choose one answer. a. True Correct. b. False The lookback period is five years. p. 17-9 Correct Marks for this submission: 1/1. Question 2 Marks: 1/1 Recapture potential carries over from a decedent to an heir. Choose one answer. a. True Correct. b. False Recapture potential does not carry over from a decedent to an heir. p. 17-20 Correct Marks for this submission: 1/1. Question 3 Marks: 1/1 Cattle must be held more than 18 months to qualify as 1231 property. Choose one answer. a. True Correct. b. False Cattle must be held 24 months or more to qualify as 1231 property. p. 17-6 Correct Marks for this submission: 1/1. Question 4 Marks: 1/1 Depreciation recapture under 1245 and 1250 is reported on Form 4797. Choose one answer. a. True b. False Correct.

pp. 17-24, 17-25, and 17-28 Correct Marks for this submission: 1/1. Question 5 Marks: 1/1 Section 1250 depreciation recapture will apply when accelerated depreciation was used on property used outside the United States and the property is sold at a gain. Choose one answer. a. True Correct. b. False p. 17-18 Correct Marks for this submission: 1/1. Question 6 Marks: 1/1 Nonpersonal use property casualty gains and losses and personal use property casualty gains and losses are netted with one another to determine whether there is a net gain subject to the 1231 rules. Choose one answer. a. True Correct. b. False Only nonpersonal use property casualty gains and losses are netted against each other to determine whether there is a net gain subject to the 1231 rules. pp. 17-6 and 17-7 Correct Marks for this submission: 1/1. Question 7 Marks: 1/1 When 1239 (relating to the sale of depreciable property between related taxpayers) applies, it results in the entire gain being treated as 1231 gain. Choose one answer. a. True Correct. b. False Section 1239 results in classification as ordinary income. p. 7-23 and Example 23 Correct Marks for this submission: 1/1. Question 8 Marks: 1/1 Part II of Form 4797 is used to report gains from sale of depreciable business equipment sold at a gain and held more than one year. Choose one answer. a. True b. False Correct.

Part III of Form 4797 is used to report gains and losses of depreciable business equipment sold at a gain and held more than one year. p. 17-25 Correct Marks for this submission: 1/1. Question 9 Marks: 1/1 Due to 1231, net gain from the disposition of depreciable real property is sometimes treated as long-term capital gain. Choose one answer. a. True Correct. b. False p. 17-3 Correct Marks for this submission: 1/1. Question 10 Marks: 1/1 A personal use property casualty loss is deductible only to the extent it exceeds 10% of AGI. Choose one answer. a. True Correct. b. False p. 17-10 Correct Marks for this submission: 1/1. Question 11 Marks: 1/1 Steven sells equipment for $16,000 which had been purchased for $50,000 and on which $30,000 of depreciation had been taken. He had used the equipment in his business for several years. Steven has: Choose one answer. a. A $4,000 1245 loss. b. A $4,000 1231 loss. c. A $4,000 1250 loss. d. A $16,000 1231 gain. e. None of the other answers are correct. Amount realized Adjusted basis ($50,000 - $30,000) $ 16,000 (20,000) Correct.

Recognized loss

($ 4,000)

The 1245 recapture provision does not apply because the property was disposed of at a loss. The entire loss is classified as a 1231 loss. pp. 17-12 and 17-13 Correct Marks for this submission: 1/1. Question 12 Marks: 1/1 A business building owned by an individual is destroyed by fire. The building was depreciated using straight-line depreciation. The building was insured and the insurance proceeds exceeded the building's adjusted basis. The building was not replaced. The insurance proceeds did not exceed the original cost of the building. The gain from disposition of the building is initially: Choose one answer. a. Casualty gain. b. Casualty loss. c. Section 1231 gain. d. Section 1250 gain. e. None of the above. There is no 1250 depreciation recapture because straight-line depreciation was used. Since the disposition of the asset was by casualty and depreciation recapture does not apply, the initial characterization of the gain is casualty gain. pp. 17-8 and 17-16 Correct Marks for this submission: 1/1. Question 13 Marks: 1/1 Orange Company had machinery destroyed by a fire on December 23, 2007. The machinery had been acquired on April 1, 2005 for $49,000 and its adjusted basis was $14,200. The machinery was completely destroyed and Orange received $30,000 of insurance proceeds for the machine and did not replace it. This was Orange's only casualty or theft event for the year. As a result of this event, Orange has: Choose one answer. a. $4,200 ordinary loss. b. $15,800 1245 recapture gain . c. $14,200 1245 recapture gain. d. $30,000 1231 gain. e. None of the above. Since the machine was held more than 12 months and was depreciated, it was a 1231 asset. However, since it was disposed of at a $15,800 gain ($30,000 insurance proceeds $14,200 adjusted basis), all of the gain is initially 1245 depreciation recapture gain and not casualty gain. pp. 17-13 and 17-14 Correct. Correct.

Correct Marks for this submission: 1/1. Question 14 Marks: 1/1 A machine used in business is stolen. The original cost of the machine four years ago was $56,000, its adjusted basis was zero at the time of the theft, and $44,000 was recovered from the business' insurance policy. There are no other casualties or thefts and no other business depreciable asset dispositions during the year. Which of the statements below is correct? Choose one answer. a. Before it was stolen, the machine was an ordinary asset. b. Before it was stolen, the machine was a 1231 asset. c. Before it was stolen, the machine was a capital asset. d. The gain from disposition of the machine is a 1231 gain. e. None of the above. Since the machine was used in business, was depreciable, and was held more than 12 months, it was a 1231 asset when it was stolen. The gain equals the $44,000 insurance recovery since the machine had a zero basis. All of the gain is ordinary gain due to 1245 depreciation recapture. pp. 17-5 and 17-13 Correct Marks for this submission: 1/1. Question 15 Marks: 1/1 Jamie elects to treat the cutting of timber as a sale or exchange under 1231. Jamie purchased the land for $100,000 and the timber for $125,000 several years ago. On the first day of 2010, the timber was appraised at $220,000 and in September 2010 it was cut and sold for $275,000. What is Jamie's ordinary income from this transaction? Choose one answer. a. $0. b. $55,000. c. $95,000. d. $150,000. e. None of the other answers are correct. The recognized 1231 gain is determined at the time the timber is cut and is equal to the difference between the timber's fair market value as of the first day of the taxable year and the adjusted basis for depletion. If a taxpayer sells the timber for more or less than the fair market value as of the first day of the taxable year in which it is cut, the difference is ordinary income ($55,000 in this case) or loss. p. 17-5 Correct. Correct.

Correct Marks for this submission: 1/1. Question 16 Marks: 1/1 Which of the following statements is correct? Choose one answer. a. When depreciable property is gifted to another individual taxpayer, the depreciation recapture potential is extinguished. b. When depreciable property is inherited by a taxpayer, the depreciation recapture Correct. potential is extinguished. c. When corporate depreciable property is distributed as a dividend, the depreciation recapture potential is generally not recognized. d. When depreciable property is contributed to charity, the depreciation recapture potential has no effect on the amount of the charitable contribution deduction. e. All of the above are correct. Depreciation recapture potential is extinguished when property is received as an inheritance, carries over when the property is received by gift, is recognized by the corporation when property is distributed by a corporation and a gain would have been recognized if the property had been sold, and reduces the charitable contribution deduction amount. p. 17-22 Correct Marks for this submission: 1/1. Question 17 Marks: 1/1 The following assets in Jack's business were sold in 2007: Asset Office Equipment Automobile ABC Stock (capital asset) Holding Period 6 years 8 months 2 years Gain/(Loss) $1,100 ($ 800) $1,400

The office equipment had a zero adjusted basis and was purchased for $8,000. The automobile was purchased for $2,000 and sold for $1,200. The ABC stock was purchased for $1,800 and sold for $3,200. In 2007 (the year of sale), Jack should report what amount of net capital gain and net ordinary income? Choose one answer. a. $1,700 LTCG. b. $600 LTCG and $300 ordinary gain. c. $1,400 LTCG and $300 ordinary gain. d. $2,500 LTCG and $800 ordinary loss. e. None of the above. The sale of the office equipment results in a $1,100 1245 gain. The sale of the auto results in an ordinary loss of $800 because the auto was not held for the long-term holding period. The Correct.

1245 gain of $1,100 offsets the $800 ordinary loss for a net ordinary gain of $300. The sale of the stock results in a $1,400 LTCG. pp. 17-8 to 17-12 Correct Marks for this submission: 1/1. Question 18 Marks: 1/1 A retail building used in the business of a sole proprietor is sold on February 10, 2007 for $453,000. Accelerated depreciation of $107,000 had been taken on the building and it had been acquired in 1985 for $113,000. Straight-line depreciation for the same holding period would have been $102,000. What is the maximum unrecaptured 1250 gain from this disposition after considering depreciation recapture? Choose one answer. a. $107,000. b. $102,000. c. $113,000. d. $447,000. Correct. e. $0. The building is nonresidential real estate acquired in 1985 on which accelerated depreciation was taken. Therefore, it is subject to the 1245 depreciation recapture rules and all of the $107,000 of depreciation taken is recaptured by 1245 as ordinary income. There is no depreciation taken left to be treated as unrecaptured 1250 gain. The $447,000 gain [$453,000 selling price ($113,000 cost $107,000 depreciation)] is classified as $107,000 ordinary gain and $340,000 ($447,000 $107,000) 1231 gain. pp. 17-12 to 17-14 Correct Marks for this submission: 1/1. Question 19 Marks: 1/1 Yellow Company sold machinery for $5,000 on December 23, 2007. The machinery had been acquired on April 1, 2005 for $49,000 and its adjusted basis was $14,200. The 1231 gain, 1245 recapture gain, and 1231 loss from this transaction are: Choose one answer. a. $0 1231 gain, $0 1245 recapture gain, $10,000 1231 loss. b. $0 1231 gain, $0 1245 recapture gain, $14,200 1231 loss. c. $0 1231 gain, $0 1245 recapture gain, $9,200 1231 loss. d. $0 1231 gain, $4,200 1245 recapture gain, $10,000 1231 loss. e. None of the above. Since the machine was held more than 12 months and was depreciated, it was a 1231 asset. However, since it was sold at a $9,200 loss ($5,000 selling price $14,200 adjusted basis), all of the loss is 1231 loss. pp. 17-13 and 17-14 Correct Marks for this submission: 1/1. Correct.

Question 20 Marks: 1/1 Kari owns depreciable residential rental real estate which has accumulated depreciation (all from straight-line) of $45,000. If Kari sold the property, she would have a $33,000 gain. The initial characterization of the gain would be: Choose one answer. a. Section 1245 gain. b. Section 1231 gain. c. Section 1250 gain. d. Section 1239 gain. e. None of the above. The gain is 1231 gain. Since straight-line depreciation was used, there is no 1250 recapture. Also, since Kari is an individual, there is no "ordinary gain adjustment" under 291. Section 1239 would not apply because there is no reason to conclude that the property would be sold to a related taxpayer. pp. 17-17, 17-18, and 17-23 Correct Marks for this submission: 1/1. Question 21 Marks: 1/1 White Company is an accrual basis taxpayer that sold $45,000 of accounts receivable for $33,000. The $12,000 loss on the sale is an ordinary loss because: Choose one answer. a. The asset is a capital asset. b. The asset is a 1231 asset. c. The asset is an ordinary asset. d. The asset was disposed of for a loss. e. None of the above are correct. The asset is not a 1231 asset because it is an account receivable and it is not a capital asset because accounts receivable are specifically excluded from the definition of a capital asset. Consequently, the account receivable falls into the ordinary asset default category. p. 17-5 and Chapter 16, pp. 16-3 and 16-4 Correct Marks for this submission: 1/1. Question 22 Marks: 1/1 Near the end of 2007, Spencer calculates his gains and losses from 2007 dispositions of 1231 assets. He determines that he has a net 1231 gain of $30,000. Spencer has 1231 lookback losses of $17,000. Which of the following transactions (if Spencer completed them in 2007) would increase Spencer's 1231 gain treated as long-term capital gain? Choose one answer. Correct. Correct.

a. Sale of a business machine for $40,000 which was purchased for $56,000 and has an adjusted basis of $16,000. b. Sale of a business machine for $40,000 which was purchased for $56,000 and has an adjusted basis of $46,000. c. Sale of business land (held 16 months) for $40,000 which was purchased for $56,000 and has an adjusted basis of $56,000. d. Sale of business land (held 16 months) for $40,000 which was purchased for Correct. $36,000 and has an adjusted basis of $36,000. e. None of the above. Spencer already has a 1231 gain of $30,000; $17,000 of this gain will be treated as ordinary gain because of the 1231 lookback rules and $13,000 will be treated as long-term capital gain. Only a transaction which generates 1231 gain will increase the $13,000 1231 gain already existing. Answer a. generates a $24,000 1245 gain; answer b. generates a $6,000 1231 loss; and answer c. generates a $16,000 1231 loss. Answer d. generates a $4,000 1231 gain. pp. 17-9 and 17-12 Correct Marks for this submission: 1/1. Question 23 Marks: 0/1 A retail building used in the trucking business of a sole proprietor is sold on February 10, 2010 for $2,000,000. Accelerated depreciation of $700,000 had been taken on the building and it had been acquired in 1987 for $1,200,000. Straight-line depreciation for the same holding period would have been $600,000. What is the maximum unrecaptured 1250 gain from this disposition after considering depreciation recapture? Choose one answer. a. $0. b. $100,000. c. $700,000. d. $1,100,000. e. None of the other answers are correct. The building is nonresidential real estate acquired in 1987 on which accelerated depreciation was taken. Therefore, it is subject to the 1245 depreciation recapture rules and all of the $700,000 of depreciation taken is recaptured by 1245 as ordinary income. There is no depreciation taken left to be treated as unrecaptured 1250 gain. The $1,500,000 gain [$2,000,000 selling price - ($1,200,000 cost - $700,000 depreciation)] is classified as $700,000 ordinary gain and $800,000 ($1,500,000 $700,000) 1231 gain. pp. 17-12 to 17-18 Incorrect Incorrect.

Marks for this submission: 0/1. Question 24 Marks: 1/1 Eighteen-year residential real property owned by an individual has accumulated accelerated depreciation of $150,000 at January 1, 2007. If depreciation had been computed under the straight-line method, accumulated depreciation would be $140,000. The property is sold on January 1, 2007 with a recognized gain of $175,000. What is the amount of depreciation recapture? Choose one answer. a. $10,000. b. $140,000. c. $150,000. d. $175,000. e. None of the above. The recapture is calculated under 1250. Therefore, only the additional depreciation of $10,000 is recaptured ($150,000 $140,000). pp. 17-15 and 17-16 Correct Marks for this submission: 1/1. Question 25 Marks: 1/1 Blue Company sold machinery for $55,000 on December 23, 2007. The machinery had been acquired on April 1, 2005 for $49,000 and its adjusted basis was $14,200. The 1231 gain, 1245 recapture gain, and 1231 loss from this transaction are: Choose one answer. a. $6,000 1231 gain, $34,800 1245 recapture gain, $0 1231 loss. b. $0 1231 gain, $40,400 1245 recapture gain, $0 1231 loss. c. $6,000 1231 gain, $40,400 1245 recapture gain, $0 1231 loss. d. $0 1231 gain, $40,400 1245 recapture gain, $14,200 1231 loss. e. None of the above. Since the machine was held more than 12 months and was depreciated, it was a 1231 asset. Since it was sold at a gain and the selling price exceeded the original cost, all of the depreciation taken of $34,800 ($49,000 cost $14,200 adjusted basis) is gain recaptured by 1245 and the remaining $6,000 gain ($55,000 selling price $49,000 original cost) is 1231 gain. pp. 17-13 and 17-14 Correct Marks for this submission: 1/1. Correct. Correct.

1 Marks: 1/1 Like S corporations, partnerships serve as conduits for their owners. Choose one answer. a. True Correct. b. False Both partnerships and S corporations serve as conduits for their owners. p. 20-30 Correct Marks for this submission: 1/1. Question 2 Marks: 1/1 Under certain conditions, corporations may be able to deduct charitable contributions in a year that is different than the year of payment. Choose one answer. a. True Correct. b. False If properly authorized by the Board of Directors before the end of the year, it can be deducted in that year even though not paid until the following year. This applies only to accrual basis corporations and payment must take place on or before the fifteenth day of the third month of the following tax year. p. 20-8 and Example 6 Correct Marks for this submission: 1/1. Question 3 Marks: 1/1 A corporation with no taxable income need not file a tax return. Choose one answer. a. True Correct. b. False The lack of taxable income does not excuse the filing requirement for a corporation. p. 20-14 Correct Marks for this submission: 1/1. Question 4 Marks: 1/1 Qualified personal service corporations are subject to a flat income tax rate of 35% and are not allowed to use the graduated rates available to regular corporations. Choose one answer. a. True Correct. b. False Personal service corporations are taxed at a flat 35% rate on all taxable income. p. 20-13. Correct Marks for this submission: 1/1.

Question 5 Marks: 1/1 An S corporation makes a $20,000 cash distribution to a shareholder whose basis in the stock is $19,000. The distribution will not result in a negative basis in the stock. Choose one answer. a. True Correct. b. False A capital gain of $1,000 results to the shareholder since the distribution exceeds the shareholders basis by this amount. A shareholders basis in a stock investment can never be a negative amount. p. 20-29 Correct Marks for this submission: 1/1. Question 6 Marks: 1/1 In determining which organizational expenditures can be amortized, it is the year incurred (and not paid) that controls only if the corporation uses the accrual method of accounting. Choose one answer. a. True Correct. b. False This rule also governs corporations that use the cash basis method of accounting. p. 20-12 and Footnote 18 Correct Marks for this submission: 1/1. Question 7 Marks: 1/1 Satisfying the definition of a small business corporation is necessary only when the S election is initially made. Choose one answer. a. True Correct. b. False Compliance with the definition of a small business corporation is a continuing requirement for maintaining S corporation status. p. 20-26 Correct Marks for this submission: 1/1. Question 8 Marks: 1/1 Although the corporate income tax is progressive, the lower brackets are phased out at higher taxable income levels. Choose one answer. a. True b. False p. 20-6 Correct.

Correct Marks for this submission: 1/1. Question 9 Marks: 0/1 A distribution to a shareholder will not be treated as dividend income if the corporation had a deficit in accumulated earnings and profits. Choose one answer. a. True Incorrect. p. 20-20 b. False Dividend income results to the extent of any current earnings and profits. Incorrect Marks for this submission: 0/1. Question 10 Marks: 1/1 The AMT is not applicable to regular corporations. Choose one answer. a. True Correct. b. False Only certain small business corporations are exempt from the AMT. p. 20-14 Correct Marks for this submission: 1/1. Question 11 Marks: 1/1 Joshua is the sole shareholder of Bluejay Corporation, a calendar year C corporation. As of January 1, 2007, Bluejay has a deficit of $10,000 in accumulated earnings and profits. During 2007, it has current earnings and profits of $60,000 and distributes a cash dividend of $80,000 to its shareholder. As of January 1, 2007, Joshua's basis in the Bluejay stock is $15,000. One result of these transactions is: Choose one answer. a. Dividend income to Joshua of $70,000. b. Dividend income to Joshua of $80,000. c. As of January 1, 2008, Joshua's basis in his stock is $0. d. As of January 1, 2008, Bluejay Corporation has a deficit in accumulated earnings and profits of $30,000. e. None of the other answers are correct. Joshua has dividend income of $60,000 (the amount of Bluejays current earnings and profits). The remaining $20,000 of the $80,000 distribution is a return of capital. As his basis in the stock is only $15,000, the extra $5,000 is a capital gain. Consequently, Joshuas basis in the Bluejay stock as of January 1, 2008, is $0 (choice c.). Since a dividend cannot increase a deficit, Bluejays accumulated earnings on January 1, 2008, is still a deficit of $10,000. p. 20-20 and Example 20 Correct Correct.

Marks for this submission: 1/1. Question 12 Marks: 1/1 Which of the following, if any, correctly characterize the check-the-box Regulations? Choose one answer. a. A one-owner business becomes a sole proprietorship if default (no election is made) occurs. b. A one-owner business cannot elect to be taxed as a corporation. c. If default (no election is made) occurs, a limited liability company is taxed as a corporation. d. The check-the-box Regulations apply to all entities that are already incorporated under state law.

Correct.

e. None of the above. A one-owner business can elect to be taxed as a corporation (choice b.). If default occurs, (no election is made), a limited liability company is taxed as a partnership (choice c.). The checkthe-box Regulations do not apply to entities that are incorporated under state law (choice d.). p. 20-4 Correct Marks for this submission: 1/1. Question 13 Marks: 1/1 Hailey redeems some of the stock she holds in Mallard Corporation for $300,000. The stock has a basis to her of $100,000 and has been held by her as an investment for many years. Mallard Corporation has more than $2 million in earnings and profits. In the year of the redemption, Hailey is in the 33% income tax bracket. The redemption generates a tax of: Choose one answer. a. $99,000 if it does not qualify for sale or exchange treatment. b. $45,000 if it does not qualify for sale or exchange treatment. c. $45,000 if it does qualify for sale or exchange treatment. d. $66,000 if it does qualify for sale or exchange treatment. e. None of the above. If the redemption qualifies for sale or exchange treatment, Hailey has a long-term capital gain of $200,000 [$300,000 (redemption amount) $100,000 (basis in the stock)]. Thus, the resulting tax is $30,000 ($200,000 X 15%). If the redemption does not so qualify, Hailey has dividend income of $300,000. Now the tax becomes $45,000 ($300,000 X 15%). p. 20-23 Correct Marks for this submission: 1/1. Question 14 Marks: 1/1 To improve its liquidity, the shareholders of Apple Corporation make a capital contribution of $100,000 in cash. To facilitate the building of a manufacturing plant, the Correct.

City of Atlanta donates land (worth $150,000) to Apple Corporation. As a result of these transactions, Apple has: Choose one answer. a. No income and a basis in the land of zero. b. Income of $250,000 and a basis in the land of zero. c. Income of $150,000 and a basis in the land of $150,000. d. No income and a basis in the land of $150,000. e. None of the other answers are correct. Neither a capital contribution by shareholders nor a property contribution by a nonshareholder results in income to the corporation. The basis of the property contributed is zero. p. 20-16 and Example 19 Correct Marks for this submission: 1/1. Question 15 Marks: 1/1 Apricot Company is a wholesale distributor of grocery products to food service businesses. During the year it donates groceries (cost of $10,000 and retail value of $14,000) to the local church soup kitchen. Disregarding percentage limitations, Apricot's charitable deduction is: Choose one answer. a. $14,000 if Apricot is a C corporation. b. $12,000 if Apricot is not a C corporation. c. $10,000 if Apricot is a C corporation. d. $12,000 if Apricot is a C corporation. e. None of the other answers are correct. If Apricot is a C corporation, the deduction is $10,000 + 50%($14,000 - $10,000) = $12,000 (choice d.). If Apricot is not a C corporation, the deduction is limited to $10,000. Example 9 Correct Marks for this submission: 1/1. Question 16 Marks: 1/1 Correct. Correct.

Brent contributes land (basis of $200,000 and fair market value of $225,000) to a partnership in return for a 50% interest. The land is subject to a mortgage of $100,000, which is assumed by the partnership. Brent's basis in the partnership is: Choose one answer. a. $100,000. b. $150,000. c. $200,000. d. $225,000. e. None of the other answers are correct. Brent's basis in the partnership is $200,000 (basis in the land) less $50,000 (50% of the mortgage assumed by the other partners). pp. 20-31, 20-32, and Example 46 Correct Marks for this submission: 1/1. Question 17 Marks: 0/1 Elizabeth and her brothers each owns a one-third interest in Ecru Partnership. In 2005, Elizabeth sells Ecru land (basis $280,000) for its fair market value of $260,000. In 2007, Ecru sells the land to a third party for $285,000. Which of the following correctly describes these transactions? Choose one answer. a. Elizabeth recognized loss of $20,000; Ecru recognized gain of $25,000. b. Elizabeth recognized loss of $0; Ecru recognized gain of $25,000. c. Elizabeth recognized loss of $0; Ecru recognized no gain or loss. d. Elizabeth recognized loss of $0; Ecru recognized gain of $5,000. e. None of the above. Elizabeth and Ecru Partnership are considered to be related parties. Elizabeth is treated as having a 100% capital interest (1/3 direct ownership + 2/3 indirect ownership from her brothers). Thus, none of Elizabeths realized loss of $20,000 can be recognized. Ecru Partnership can utilize Elizabeths disallowed loss of $20,000 to reduce its realized gain of $25,000 to a recognized gain of $5,000. pp. 20-33, 20-34, and Examples 47 and 48 Incorrect Marks for this submission: 0/1. Question 18 Marks: 1/1 As of January 1, 2010, Sarah, the sole shareholder of Shrike (a calendar year S corporation), had a basis in her stock of $80,000. During 2010, Shrike had a net operating loss of $120,000 and distributed a $50,000 dividend to Sarah. As of January 1, 2010, Shrike had accumulated E & P of $200,000. What is Sarah's basis in her Shrike stock as of January 1, 2011? Choose one answer. Incorrect. Correct.

a. $80,000. b. $30,000. c. ($90,000). d. $0. Correct.

e. None of the above. Sarahs basis is reduced to zero by the NOL pass-through. A negative basis is not allowed. p. 20-29 Correct Marks for this submission: 1/1. Question 19 Marks: 1/1 As of January 1, 2010, Michael, the sole shareholder of Kingfisher (a calendar year C corporation), had a basis in his stock of $80,000. During 2010, Kingfisher had an operating loss of $120,000 and distributed a $50,000 dividend to Michael. As of January 1, 2010, Kingfisher had accumulated E & P of $200,000. What is Michael's basis in his Kingfisher stock as of January 1, 2011? Choose one answer. a. $80,000. b. $30,000. c. ($90,000). d. $0. e. None of the above. A shareholders basis in the stock of a C corporation is not affected by profits (or losses) of the corporation. Likewise, as long as covered by E & P, dividend distributions have no effect on the shareholders basis. Examples 19 and 20 Correct Marks for this submission: 1/1. Question 20 Marks: 1/1 Ashley is a 60% shareholder in Gadwall Ltd., a calendar year S corporation. As of January 1, 2011, Ashley's basis in her stock investment was $200,000. During 2011, she made an additional capital contribution to Gadwall of $50,000 and also loaned the corporation $40,000. During 2011, Gadwall Ltd. had an operating loss of $500,000. One of the results of these transactions as to 2011 is that: Choose one answer. a. Ashley cannot recognize any of the corporation's loss due to the related party disallowance rules. b. Ashley can recognize a loss of $200,000. c. Ashley can recognize a loss of $240,000. Correct.

d. Ashley can recognize a loss of $290,000.

Correct.

e. None of the above. Ashleys share of the recognized loss, $300,000 (60% X $500,000), is limited to her basis in the stock and any loans she has made to the corporation. Thus, her available basis for a loss passthrough is $290,000 [$200,000 (stock basis) + $50,000 (capital contribution) + $40,000 (loan to corporation)]. The remaining unused loss of $10,000 ($300,000 $290,000) can be carried over to future years. p. 20-29 Correct Marks for this submission: 1/1. Question 21 Marks: 1/1 Which, if any, of the following transactions incurred by an S corporation is a separately stated item? Choose one answer. a. Salaries paid to shareholder-employees. b. Qualified dividend income. c. Amortization of organizational expenses. d. Gift to city health inspector to avoid sanitation penalty. e. None of the above. The salaries paid are assumed to be reasonable (choice a.). Otherwise they would not be deductible in arriving at operating income. The gift to the city health inspector (choice d.) is a nondeductible bribe that has no tax effect on either the corporation or its shareholders. p. 20-27 Correct Marks for this submission: 1/1. Question 22 Marks: 1/1 In 2010, Papaya Corporation (a calendar year taxpayer) has the following income and expenses: Gross income from operations Expenses from operations Dividends from Macadamia Corporation $400,000 425,000 175,000 Correct.

Assume Papaya Corporation owns 30% of the stock of Macadamia Corporation. The dividends received deduction for 2010 is: Choose one answer. a. $105,000.

b. $120,000. c. $140,000. d. $175,000. e. None of the other answers are correct. 80% X $175,000 = $140,000 would be the amount of the dividend received deduction. However, the taxable income limitation applies limiting the deduction to $120,000 ($150,000 (income before dividends received deduction) X 80%). pp. 20-11, 20-12, and Example 14 Correct Marks for this submission: 1/1. Question 23 Marks: 0/1 Molly is a 50% shareholder in a calendar year S corporation. As of January 1, 2010, Molly had a basis in her stock of $50,000 and had loaned the corporation $30,000. For 2010, the corporation had an operating loss of $250,000. Of this loss, Molly may deduct: Choose one answer. a. $30,000. b. $50,000. c. $80,000. d. $125,000. e. none of the other answers are correct. Although Molly's share of the loss is $125,000 (50% X $250,000), the deduction is limited to the summation of her basis in the stock ($50,000) and the amount of the loan ($30,000). Thus, $45,000 ($125,000 - $80,000) of her loss cannot be used in 2010. p. 20-30 and Example 42 Incorrect Marks for this submission: 0/1. Question 24 Marks: 0/1 At a time when Walnut Corporation has earnings and profits of $125,000, it redeems half of Oliver's shares for $200,000. Oliver's basis in all of his shares is $150,000. Oliver's shares are held as an investment and were acquired five years ago. If the redemption qualifies for sale or exchange treatment, he must: Incorrect. Correct.

Choose one answer. a. Recognize dividend income of $125,000 and reflect a return of capital distribution of $75,000. b. Recognize a long-term capital gain of $125,000. c. Recognize a long-term capital gain of $50,000. d. Recognize a dividend income of $125,000. e. None of the other answers are correct. Oliver's recognized long-term capital gain is calculated as follows: Amount realized Adjusted basis ($150,000 X 50%) Long-term capital gain p. 20-23 and Example 28 Incorrect Marks for this submission: 0/1. Question 25 Marks: 0/1 Bill and Linda form Owl Corporation with the following investment: Cash Property (basis of $290,000 and fair market value of $320,000) From Bill From Linda $300,000 $320,000 $200,000 (75,000) $125,000 Incorrect.

Bill and Linda each receive one-half of Owl Corporation's stock (worth $300,000). In addition, Linda receives cash of $20,000. As a result of these transfers, Owl's basis in the property it receives from Linda will be: Choose one answer. a. $290,000. b. $300,000. c. $310,000. d. $320,000. Incorrect.

e. None of the above. $290,000 (Lindas basis) + $20,000 (Lindas recognized gain) = $310,000. Although Linda has realized gain of $30,000 [$300,000 (value of stock received) + $20,000 (cash) $290,000

(basis)], her recognized gain cannot exceed the $20,000 boot received. pp. 20-17, 20-19, 20-20, and Examples 17 and 18 Incorrect Marks for this submission: 0/1. 1 Marks: 1/1 A cash balance plan is a hybrid form of pension plan that is similar in many aspects to a defined contribution plan. Choose one answer. a. True b. False Correct.

A cash balance plan is similar in many aspects to a defined benefit plan. p. 19-7 Correct Marks for this submission: 1/1. Question 2 Marks: 1/1 The payout to an employee in a cash balance plan is based upon a formula based on years of service. Choose one answer. a. True Correct. b. False pp. 19-9 and 19-10 Correct Marks for this submission: 1/1. Question 3 Marks: 1/1 A 15% excise tax is imposed on nondeductible contributions to a retirement plan. Choose one answer. a. True Correct. b. False The 10% excise tax is imposed on the employer rather than on the employee. Correct Marks for this submission: 1/1. Question 4 Marks: 1/1 p. 19-14

A failure to make a minimum required distribution to a participant in any taxable year results in a 10% nondeductible excise tax on any excess of the amount that should have been distributed over the amount that actually was distributed. Choose one answer. a. True Correct. b. False The correct percentage is 50%. p. 19-10 Correct Marks for this submission: 1/1. Question 5 Marks: 1/1 The maximum annual elective contribution for a participant who is age 47 in a 401(k) plan is $16,500 for 2011. Choose one answer. a. True b. False
REF: p. 19-16

Correct.

Correct Marks for this submission: 1/1. Question 6 Marks: 1/1 A company is denied a deduction for a golden parachute payment to an employee, but not for a golden parachute payment to an independent contractor. Choose one answer. a. True Correct. b. False Both employees and independent contractors are covered by the golden parachute provision. p. 19-31 Correct Marks for this submission: 1/1. Question 7 Marks: 1/1 Benefits of a profit sharing plan may not be distributed through lump-sum payouts. Choose one answer. a. True Correct. b. False Since a profit sharing plan does not necessarily emphasize retirement, lump-sum payouts occur frequently. p. 19-6 Correct Marks for this submission: 1/1.

Question 8 Marks: 1/1 The "spread" on an incentive stock option is subject to the alternative minimum tax. Choose one answer. a. True Correct. b. False The spread on an ISO is a positive adjustment for AMT purposes. pp. 19-35 and 19-36 Correct Marks for this submission: 1/1. Question 9 Marks: 1/1 Most NQDC plans must meet the discrimination and funding requirements of qualified deferred compensation plans. Choose one answer. a. True Correct. b. False This is a key factor in selecting a NQDC plan versus a qualified deferred compensation plan. p. 19-30 Correct Marks for this submission: 1/1. Question 10 Marks: 1/1 An individual is considered an active participant in an employer-sponsored retirement plan merely because an individual's spouse is an active participant for any part of a plan year in applying the IRA phase-out provision. Choose one answer. a. True Correct. b. False Times New Roman Correct Marks for this submission: 1/1. Question 11 Marks: 1/1 Harry receives a $10,000 distribution from a CESA. On this date, his total account balance is $16,000, with $4,000 representing earnings. If his qualified higher education expenses are $8,000, the amount included in gross income is: Choose one answer. a. None. b. $500. Correct.

c. $2,000. d. $2,500. e. None of the other answers are correct. A portion of the distribution is treated as a nontaxable return of capital, $7,500 [$10,000 ($12,000/$16,000)]. The balance of $2,500 ($10,000 - $7,500) is a distribution of earnings. Since Harry's qualified higher education expenses are less than the $10,000 distribution, a portion of the earnings are taxable. Excludible portion of earnings: ($8,000/$10,000) $2,500 earnings = $2,000. Included in gross income: $2,500 - $2,000 = $500. REF: p. 19-25 | Example 26 Correct Marks for this submission: 1/1. Question 12 Marks: 1/1 Which statement is true with respect to the $1 million compensation deduction limitation? Choose one answer. a. Compensation includes salary and bonus payment. b. The limitation is decreased by any nondeductible golden parachute payments. c. The limit applies only to publicly held corporations. d. The limitation does not apply to qualified performance based compensation. e. All of the above are true. pp. 19-31 and 19-32 Correct Marks for this submission: 1/1. Question 13 Marks: 1/1 Barbara has four years of service completed as of March 8, 2007, her employment anniversary date. If her 401(k) plan has a 401(m) arrangement and uses the graded vesting rule, determine Barbara's nonforfeitable percentage. Choose one answer. a. 0%. Correct.

b. 40%. c. 60%. d. 100%. e. None of the above. p. 19-9 and Table 19-2 Correct Marks for this submission: 1/1. Question 14 Marks: 1/1 What is the maximum amount Venessa can contribute to a Coverdell Education Savings Account (CESA) on behalf of a grandson in 2010? She is single with an AGI of $99,000. Choose one answer. a. $0. b. $500. c. $1,467. d. $2,000. e. None of the above. Venessa may contribute $1,467 to the Coverdell Education Savings Account, calculated as follows: $99,000 AGI - $95,000 threshold = $4,000 excess AGI $4,000/$15,000 phaseout range $2,000 = $533 phaseout $2,000 maximum contribution - $533 phaseout = $1,467 contribution ceiling PTS: 1 DIF: 1 REF: p. 19-25 Correct Marks for this submission: 1/1. Question 15 Marks: 0/1 Olivia, self-employed, has gross earned income (before a Keogh contribution) of $35,000 (but after the deduction for one-half of self employment-tax). What is Olivia's maximum contribution to her defined contribution pension plan? Choose one answer. a. $5,000. Incorrect. Correct. Correct.

b. $35,000. c. $40,000. d. $49,000. e. None of the other answers are correct. Olivia can contribute $35,000 which is 100% of $35,000. p. 19-21 and Example 17 Incorrect Marks for this submission: 0/1. Question 16 Marks: 1/1 Which of the following is considered an advantage of an IRA over a 401(k) plan? Choose one answer. a. Employer involvement is minimal. b. Favorable ten-year averaging may be available. c. Higher contribution limitation. d. Distributions taxed at capital gain rates. e. None of the above. Concept Summary 19-5 Correct Marks for this submission: 1/1. Question 17 Marks: 1/1 Joshua, age 39, and Beverly, age 40, who have been married for seven years, are both active participants in qualified retirement plans. Their total AGI for 2010 is $160,000. Each is employed and earns a salary of $80,000. What is their combined deductible contributions to traditional IRAs? Choose one answer. a. $0. b. $5,000. c. $7,500. d. $10,000. e. None of the other answers are correct. Joshua and Beverly may contribute a total of $10,000 to a traditional IRA, but because their AGI exceeds the phaseout ceiling of $99,000, nothing is deductible. pp. 19-22 and 19-23 Correct. Correct.

Correct Marks for this submission: 1/1. Question 18 Marks: 1/1 Daniel is a self-employed accountant with gross earned income of $90,000 per year (after the deduction for one-half of any self-employment tax). He has a profit sharing plan (i.e., defined contribution plan). What is the maximum amount Daniel can contribute to his retirement plan? Choose one answer. a. $18,000. b. $22,500. c. $49,000. d. $90,000. e. None of the other answers are correct. Daniel can contribute $18,000, which is 20% of $90,000, or 25%($90,000 - $18,000). p. 19-21 Correct Marks for this submission: 1/1. Question 19 Marks: 0/1 Mickie, a single parent, has modified AGI of $106,000. Calculate her maximum contribution to a Coverdell Education Savings Account for her 8-year old son in 2010. Choose one answer. a. $-0-. b. $540. c. $1,460. d. $2,000. e. None of the other answers are correct. The maximum contribution to a Coverdell Education Savings Account for a beneficiary of Mickie in 2010 is $2,000. However, the contribution is phased out for single taxpayers beginning at $95,000 of AGI and becoming completely phased out at $110,000 of AGI. With Mickie's AGI of $106,000, 73% ($106,000 - $95,000/$15,000) of the contribution is phased out. Thus, she is entitled to make a contribution of only $540 to the educational IRA in 2009. pp. 19-25 and 19-26 Incorrect. Correct.

Incorrect Marks for this submission: 0/1. Question 20 Marks: 1/1 Dana, age 48, is the sole remaining participant of a money purchase pension plan. The plan is terminated and a $240,000 taxable distribution is made to Dana. The early distribution penalty tax, if any, for 2011 is: Choose one answer. a. $0. b. $12,000. c. $24,000. d. $30,000. e. None of the above. $240,000 X 10% = $24,000. p. 19-11 Correct Marks for this submission: 1/1. Question 21 Marks: 1/1 James, an executive, receives a $600,000 payment under a golden parachute agreement. James' base amount from Silver, Inc., is $140,000. What total tax must James pay, assuming a 35% individual tax rate? Choose one answer. a. $0. b. $92,000. c. $210,000. d. $302,000. Correct. Correct.

e. None of the above. $302,000. James would have a regular income tax liability of $210,000 ($600,000 X .35) plus a nondeductible excise tax of $92,000 ($460,000 X .20). p. 19-31 Correct Marks for this submission: 1/1. Question 22 Marks: 1/1 Which of the following is a tax consequence of a qualified pension plan? Choose one answer. a. Employer contributions are deductible at the time of the contribution.

b. Earnings of the trust are tax-exempt to the trust. c. Employees are not taxed until distributions are received from the trust. d. Only a. and c. e. a., b., and c. p. 19-12 Correct Marks for this submission: 1/1. Question 23 Marks: 1/1 Correct.

A participant, who is age 38, in a cash or deferred arrangement plan [401(k)] may contribute up to what amount in 2011? Choose one answer. a. $12,000. b. $13,000. c. $21,500. d. $16,500. e. None of the other answers are correct. The indexed amount for 2011 is $16,500. A taxpayer who is age 38 is not eligible for the $5,000 catch-up amount in 2011. REF: p. 19-16 Correct Marks for this submission: 1/1. Question 24 Marks: 1/1 Joyce, age 39, and Sam, age 40, who have been married for seven years, are both active participants in qualified retirement plans. Their total AGI for 2010 is $120,000. Each is employed and earns a salary of $65,000. What are their combined deductible contributions to traditional IRAs? Choose one answer. a. $0. b. $3,000. c. $4,000. Correct. Correct.

d. $8,000. e. None of the other answers are correct. Joyce and Sam may contribute a total of $10,000 ($5,000 each) to a traditional IRA, but because their AGI exceeds the phaseout ceiling of $99,000 in 2010, $0 is deductible. Correct Marks for this submission: 1/1. Question 25 Marks: 0/1 Marian receives a $10,000 distribution from her education IRA. The distribution consists of $8,000 of contributions and $2,000 of earnings. Marian pays $7,000 in qualified higher education expenses for the year. Calculate the amount to be included in Marian's gross income. Choose one answer. a. $2,000. b. $1,400. c. $600. d. $-0-. e. None of the other answers are correct. $7,000 X $2,000 earnings = $1,400 exclusion $10,000 Earnings Less: exclusion Include in gross income p. 19-25 Incorrect Marks for this submission: 0/1. $2,000 ( 1,400) $ 600 Incorrect.

You might also like